Recalls 4-5
Recalls 4-5
A. “I would like to listen to your concerns about the 6. Patient Shiela asks the nurse why she has to get up and
prescribed care.” walk the day after surgery. Which of the following is the BEST
B. “It is important that you do. What the physician has response of the nurse? Walking hastens _________.
prescribed.”
C. I can come back after you talk with your spouse about A. Hastens lactation
the care.” B. Relieves pain
D. “Let me review what is needed again, then I’ll talk to you C. Heals wounds
later.” D. Fast recovery from anesthesia
7. Which laboratory finding should the nurse assess on the
patient 24 hours after caesarian section delivery upon
NP2 doctor’s request?
2. Which of the following are the purposes of documentation? 10.On the third postpartum day, Patient Shiela reports that
To ________. she has voided five times that morning. What should the
nurse INITIALLY do?
I. Ensure the development of organized comprehensive care
plan A. Insert a Foley catheter
II. Have a clear and accurate record of what was done to the B. Collect the next voiding and measure the urine amount
patient. C. Catheterize the client to check for residual urine
III. Have an evidence of the health care member’s D. Call the physician
accountability in giving care
IV. Detect patients who are clinically deteriorating Bladder: not in normal tone: give support until regain
normal tone
A. II, III, IV Patient Apple, 19 years old, is in her first trimester of
B. I, II, III, IV pregnancy. Because it is her first
C. I, II, III pregnancy, she went for her prenatal check-up with her
D. I, III, IV mother. She asked a lot of questions which
she expects the nurse to answer her.
3. Which of the following is the CORRECT definition of focus
charting? 11. The nurse asked for the personal data of the patient
which, to some, Patient Apple did not like to answer. And so
A. It is an electronically form of documentation of nursing she asked: “Why do you need to know if I am married?” what
care done to a patient by a registered nurse. should be a good response of the nurse? “I asked your marital
B. It is a note, written or electronically generated, to status because _________.
provide documentation related to a specific focus.
C. It is a nurse-centered way of documentation that A. If you do not have a husband, then that can pose a big
describes the patient status and nursing care rendered. problem for you.”
D. It is a nurse-centered approach to documentation. B. If you are married then your husband will also suffer from
discomforts like you.”
4. In the given situation, which is the FOCUS? C. You need your husband to accompany you every
prenatal check-up.”
A. Patient Natalie as the admitting nurse D. Your husband is your best support system during
B. Lumbo-sacral pains and strong uterine contractions: your pregnancy.”
DATA
C. 15 years old, G0P1, AOG of 36 weeks
D Admission at 6:30 in the morning
12. The patient asked what is the term for signs such as
breast changes, urinary frequency, fatigue, morning sickness A. Assess business facilities if they follow the 10 steps for
and amenorrhea? * successful breastfeeding.
B. Develop a research on the benefits of breastfeeding
A. Probable signs within the institution.
B. Presumptive signs C. Disseminate the benefits of breastfeeding to
C. Possible signs communities.
D. Positive signs D. Follow up mothers if they are exclusively breastfeeding.
13. The patient asked what causes newborn babies with total Heather is 23 years old. She and her boyfriend, Patrick, are
absence of extremities. The nurse answered that the cause planning to get married in a
for Amelia is intake of which of the following medications couple of months. Thereafter, they plan to have three
during pregnancy ___________. * babies. For this reason they sought
reproductive health counseling for their benefit and the
A. Anti-emetics proper growth and development of their
B. Antibiotics future children. Nurse Susane was there to help them.
C. analgesics
D. anti-bacterials 21. Heather asks what she must do in order to be healthy in
14. The patient complained that every morning, she becomes case she becomes pregnant. Which among the answers of
nauseated and oftentimes, she would vomit excessively. Nurse Susane should NOT be followed by Heather?
“What could be the cause of this,” she asked? The nurse’s A. Get support from husband and family.
answer is: “It is due to increase level of ________.” B. May have a massage from a lay midwife.
A. Heart burn C. To readily accept her pregnancy.
B. Estrogen/ increase HCGA D. Early prenatal check-up
C. heart rate
D. Progesterone- DEC gastric motility
22. Patrick asks what possible contribution he could give for
15. The nurse advised patient Patient Apple to report to her the normal development of the baby. Nurse Susane agreed
physician which of the following MOST important sign, if ever that his BEST contribution would be the following EXCEPT
she will suffer from it? __________.
A. Fetal bradycardia
B. Urine output of <30 ml per hour 52.What is the MOST APPROPRIATE response of the nurse to
C. Respiration of <12 per min the mother’s question as to when the child will be ready for a
D. Increase in maternal pulse rate cleft palate repair? Cleft palate repair is usually done __
45.Which of the following hospital environment will be MOST A. When a large-holed feeding bottle is ineffective for his
conducive to Joyce’s condition? A room that is / with feeding
____________. B. When the child is completely weaned from bottle feeding
C. Prior to the development of speech (6-18 months)
A. Bright and well-ventilated D. After the child has been toilet trained
B. 2 or three other patients
C. Quiet and non-stimulating
D. A call button for watcher’s use. 53. Elbow restraints are the choice during the patient’s
operation. When is the right time to introduce the use of these
Mrs. Paula, 20 years old, visited the prenatal clinic with her restraints? It should be during the _________ phase.
husband Christian. Nurse Cynthia noted that the fundus is at
the level of the umbilicus. Being her first pregnancy, the A. Pre-operative
patient asks the clinic nurse about pregnancy and B. Rehabilitative
childbearing. C. intra- operative
D. post- operative: put petroleum jelly to operative site
46.Based from the assessment of the nurse, what would be CL, CP: elbow immobilizer (7-10 days immediately after
the estimated age of gestation, in WEEKS, if then fundus is at surgery)
the level of the umbilicus? 20 weeks
A. 8 54.Before bringing the child to the operating room, what
B. 10 condition of the patient needs immediate notification of the
C. 6 surgeon by the nurse that will warrant suspension of surgery?
D.12 symp pubis
E. None A. Colic
B. Atopic dermatitis
Bonus C. nasopharyngitis
D. eye deviation
36: xiphoid process
55.Include in the post-op plan of care is collaboration and
47.When is the placenta, which is the source of estrogen and referral of the patient with cleft palate to which of the
progesterone, fully developed? It is on the __________ weeks of following APPROPRIATE health care provider?
pregnancy?
A. Speech therapist
A. 8 B. Dentist
B. 20 (18-20) C. dietetian
C. 16 D. gynecologist
D.28
48.Mrs. Paula related that she oftentimes feel nauseated in The giving of medication to a pediatric patient is a serious
the afternoon. Which of the following should Nurse Cynthia responsibility of a nurse. The need for accuracy in giving
advise her to do? medication is greater than with adult patients. Nurse Yuuh
has just been assigned to the Pediatric Wards.
A. Eat sky flakes and follow it up with water
B. Drink iced carbonated drinks 56. Dosage, when giving medicine to pediatric patients, varies
C. Drink hot chocolate or coffee according to which of the following? Select all that apply
D. Eat frequent but small amount of foods.
1. Size
49.Which of the following periods of pregnancy does 3. Age of child
organogenesis occur? 2. Surface area
4. Height
A. Third
B. First A. 3 only
C. Second B. 1 and 2
D. Fourth C. 1, 2, 3
D. 1, 2, 3, 4
C. Gently pull the outer ear
57. Nurse Yuuh is being reviewed by her Headnurse on D. Keep the up for five minutes
administering oral medication on pediatric patients. Which of
the following statements below is CORRECT? 64.Based on her knowledge on otitis media, Nurse Romana
recalls that children are predisposed to AOM due to their
A. The child should be told to place the tablet in the middle Eustachian tubes being, which of the following? Select all that
of his tongue and to drink water, fruit juice, milk in order to apply.
wash down the tablet. 1. Shorter
B. A child’s reaction to a dose ordered by a physician is not 2. More horizontal
less predictable than adult’s reaction. 3. More prone to obstruction by enlarge adenoids
C. When giving oral medication, the child as young as two 4. Longer
years of age CANNOT be taught to swallow drugs. A. 1, 2, 4
D. The possibility of error is greater in the giving of B. 1 and 3
medication to children than to adults. C. 3 and 4
D. 1, 2 and 3
58.In children and infants, which part is often used for
intramuscular injection to reduce the risk of vascular and 65.To promote drainage and reduce pressure from fluid,
peripheral nerve (sciatic) injuries? Nurse Olive’s nursing intervention is to have the child assume
any of the following positions, EXCEPT:
A. Deltoid muscle A. Have the child sit up
B. Gluteus maximus B. Put the pillows behind the head
C. Dorsogluteal C. Lie on the non-affected area
D. Vastus lateralis D. Lie on the affected ear
59.Comprehensive surveys of research reports and case Itchy, 16 years of age, has been ordered to be discharged.
study literature about intramuscular injections revealed that Nurse Worms is faced with
administering medication intramuscularly can produce a a dilemma as Itchy refuse to be discharge.
variety of serious adverse effects. When asked by the
Headnurse what possible complications can arise, Nurse 66.Which of the moral problems is Nurse Worms faced with?
Yuuh should include which of the following?
1. Skin and tissue trauma A. Ethical sensitivity
2. Muscle fibrosis and contracture B. Ethical responsiveness
3. Nerve palsies and paralysis C. Ethical valuing
4. Infectious processes such as abscesses or gangrene D. Ethical reasoning
A. 1, 3, and 4 67.Which of the following are the rights and responsibilities of
B. 1, 2, 3, 4 Nurse Worms’ client while confined in the hospital?
C. 2, 3, 4
D. 1, 2, 3 A. Code of Ethics
B. Bioethical Ethics
60.Prior to administering the drugs ordered by the C. Hospital Client Advocacy
Pediatrician, Nurse Yuuh needs to know if she is giving the D. Patient’s Bill of Rights
ordered medication to the right patient. The FIRST step is
__________. 68.Which of the following principles of bioethics is referred to
when Nurse Worms provides information and supports her
A. Check patient’s identification bracelet. client and family in making a decision?
B. Compare medication to order to identification bracelet
and patient’s stated name and birth date. A. Fidelity
C. Verify patient’s allergies with chart and with patient. B. Nonmaleficence
D. Ask patient to state their name and birth date C. Justice
D. Autonomy
Nicole, 5 years of age, is admitted to the Pediatric Ward due 69.When Nurse Worms values client advocacy, she follows
to severe otalgia, fever and irritability. The mother informed which of the following guidelines?
Nurse Olive, who is assigned to Nicole that patient had upper
respiratory infection three weeks prior to admission. The A. She is more loyal to the hospital than to her client.
admission diagnosis is acute otitis media B. She chooses the client’s well-being over client’s
(AOM). autonomy.
C. She gives priority to the good of the individual client
61.Nurse Olive makes her INITIAL assessment on Nicole. The rather than to the good of society in general.
patient keeps on crying and constantly pulls her right ear. D. She makes decisions for her clients who are not aware
What is her MOST APPROPRIATE action? of their rights and privileges.
A. Assess the description and frequency of pain. 70.Which of the following actions best describe the use of the
B. Take Karen’s vital signs. professional value of altruism?
C. Request parent to carry the child.
D. Refer to the attending physician.
A. Protects the privacy of client regarding the client’s health
62.Nurse Olive is preparing to administer ofloxacin eardrop on condition.
Nicole per Doctor’s order. She needs to hold the bottle with B. Demonstrates an understanding of the culture of her
her hands to warm up the solution to prevent dizziness for _ client.
C. Lobbies for universal access to healthcare.
D. Becomes a mentor to her novice nurses.
A. 10 minutes
B. 5 minutes Miss Greigh an OB Nurse Specialists, was invited in a forum.
C. 3-5 minutes The topic was about
D. 1 to 2 minutes HIV-AIDS and the group was composed of ten pregnant
Cold eardrops: dizziness women, and five newly married women
whose husbands are seafarers
63.After washing her hands and gently cleaning any discharge
that can be removed easily from the outer ear, Nurse Olive 71. Miss Greigh’s first topic was on HIV transmission. After
positions the child. Which of the following steps follows? explaining the different transmission modes, she asked the
group: : “Which among the following is the MAJOR mode of
A. Gently press the tragus of the ear four times in a transmission of the disease?”
pumping motion.
B. Drop the medicine into the ear canal. A. Blood Transfusion
B. Needle pricks
C. Sexual intercourse( sharing of needles) A. I,II, III
D. Kissing B. II, III, IV
C. I, II, III, IV
72.One of the pregnant women asked the BEST way to D. I, III, IV
prevent HIV transmission from a pregnant HIV (+) mother to
child? 78.To erase the doubt of Kylie as to whether she has been
infected with HIV by her BF, what is the BEST advice of the
A. Deliver the child via normal delivery with cervical support nurse?
B. Stop pregnancy by abortion
C. Drink antiviral medicines as prescribed by physician A. Wait until you have access to the diagnosis of your BF.
D. Prenatal check-up more than the standards B. Submit herself for Elisa Test considering window period.
C. Wait until the baby is born.
73.A 20-year-old newly-wed woman, whose husband has D. Go for Elisa Test immediately
recently been diagnosed with HIV, asked: “Will my baby have
HIV too?” What is the BEST answer that Miss Greigh should 79.In case the personal data of Kylie’s BF has been accessed
give the client? accidentally by “somebody”, who has the right to file a
complaint?
A. “No, definitely not because your child has not been
exposed to any type of HIV transmission.” A. Cathlyn’s BF
B. “Yes, because HIV is definitely transmitted to the child A. The other guy
form the mother through the placenta. C. the parents
C. “One way of transmitting HIV from the mother to the D. Cathlyn
baby is through the placenta. However, your child may or
may not have been transmitted with HIV virus.” 80.How long should personal data of an HIV patient be stored
D. “It depends on the viral load of the mother and many on laptops or other portable devices which permits his
other factors. Let us just see you after you give birth.” identification? For as long as __________.
74. Miss Greigh continued with her health teaching. This time A. The patient likes
she describes four main routes of HIV transmission. Which B. His doctor likes
one is NOT included? C. It is needed as part of an international or national
research
A. Childbirth and breastfeeding D. It is necessary for the purpose for which it was
B. Unprotected vaginal and anal or oral sex collected and processed
C. Breathing the same air as someone living with HIV does
D. Sharing unsterilized injecting drug equipment. Rob, 15 years old, has been informed that she is due for
discharge. He has an
75. Miss Greigh gave a few questions to her audience about emergency appendectomy. Nurse Yuuh give him some
HIV transmission to find out if they were able to learn about it. discharge instructions.
“Which of the following is the LEAST among HIV transmission
that can directly enter the body via the blood stream or 81. Rob is noted to be very conversant. To make her
mucous membranes?” This can be through the ___________. discharge instructions clear, Nurse Yuuh should emphasize
which of the following? .Tell Rob that he can clarify or ask
A. Urethra or inside the foreskin of the penis questions.
B. Lining of the vagina, cervix or womb
C. Dermis of the skin I.Tell him that it is alright to interrupt if he had to ask a
D. Lining of the anus question to clarify
● II.Advise him to listen ,well
Kylie is 19 years old, a grade 12 student. She recently III.Ask for the presence of his mother
confessed to Nurse Kris that she had sexual intercourse with
her boyfriend (BF), 18 years old classmate of hers. She A. 4 only
believes that she B. 1 and 2
is pregnant because she missed menstruating for 3 months. C. 1, 2 and 3
She also related that her BF has recently D. 1 only
been confined for pneumonia after an on and off flu-like
symptoms. She suspects he has HIV Elisa- enzymes rapid
because of his “closeness to a guy who has overtly expressed Western: cobfirmatory
his feelings on him.”
82.Nurse Yuuh’s discharge instruction should contain, which
76. Kylie wants to know his BF’s real diagnosis even through a of the following? .Take home medication, if any.
private message in Facebook. The following information that I.Date to follow up
all employees are enjoined to observe strictly. .Operate and II.Activity and exercise
hold diagnosis under strict confidentiality III.Diet
I.Sign a non-disclosure agreement between nurses and their A. 2, 3 and 4
patients. B. 1, 2 and 3
II.Abide with the institution’s privacy and security policies. C. all the options
III.Disclose only the result of the best to the patient himself. In D. 3 and 4
terms of legalities, which of the following should Nurse Patty
explain t Cathlyn? 83.To make Nurse Yuuh’s discharge instructions to be more
effective the following strategies should be used, EXCEPT
A. II, III, IV _____________.
B. I, II, III
C. I, II, III, IV A. Treat him as a child.
D. I, II, IV B. Use simple understandable words
C. Eye to eye contact to show interest on what is being said
77. Nurse Kris can also explain the purpose of security D. Ask for feedback
measures for data protection which is to maintain the
availability, integrity and confidentiality of personal data of all 84.To get feedback on the discharge instructions, which
patients. .Unlawful destruction questions should Nurse Yuuh ask?
I.Alteration
II.Disclosure A. Can you repeat to me what my instructions were?
III.Unlawful processing Which of the following actions of B. If you understood my instructions then you surely be well
anybody is against personal data protection? C. Did you understand my instructions?
D. Are my instructions clear? D. tables and chairs
85.The BEST statement of goodbye that Nurse Yuuh is 93.Based from the studies, which among the following is the
_________. MOST needed resource of nurses in order to attend in-service
training programs?
A. “Bye Rob.”
B. “Bye Rob. See you again in the hospital.” A. Clothes
C. “Bye Rob. I don’t want to see you again in the hospital.” B. Time
D. “Bye Rob. Remember all my instructions. Keep well.” C. Drinks
D. Notebooks
Tiana is the new nurse assigned at Fertility Clinic of an OB 94.Evaluation of the training is an important form of feedback
Specialty Center. She had to in order to improve future offerings. Which of the following
review about the female reproductive cycle, menstrual should be given MOST emphasis?
disorders and appropriate management.
86. Tiana thought that one of her activities would be health A. Taste and amount of food serve.
teaching and counseling. Which of the following statements B. Discipline of participants
by the client regarding premenstrual syndrome would need a C. Convenience of the venue
further explanation? D. Knowledge and skills gained
A. “I will limit my intake of sodas, coffee and tea.”
B. “I will make sure that I will eat small frequent balanced 95.When asking mothers to become real models for
meals.” demonstrating breastfeeding, the nurse’s MOST important
C. “I will stop any physical exercise one week prior to responsibility is which of the following?
menstruation to conserve my energy.”
D. “I will decrease my intake of high sodium containing A. Word of thanks
foods such as canned goods.” B. Stipend/honorarium
87.The nurse read on how to make a nursing diagnosis for C. Priority care
premenstrual syndrome. Which of the following is NOT TRUE D. Free hospitalization
about premenstrual syndrome? BONUS
A. Signs and symptoms of premenstrual period recur in the Nurses Patti and Aika are doing research on, “The common
luteal phase of the menstrual cycle. errors regarding Standard of Nursing Documentation in the
B. Signs and symptoms of premenstrual period recur in the Obstetrics Ward of Medical Center.”
luteal phase of the menstrual cycle.
C. Symptom-free period occurs in the follicular phase and 96.Which could be the MAJOR Purpose of the study? To
must include 7 symptom-free days. __________.
D. Symptoms are severe enough to have an impact on
work, lifestyle and relationship. A. Improve the clarity of documentation
B. Duplicate the study in other hospitals
88.Nurse Tiana read about the nursing management of C. Reduce, if not eliminate, errors in documentation
primary dysmenorrhea. Which of these should Tiana render? D. Protect the hospital and staff from being sued
.Use of cold compress
I.Soft rhythmic rubbing of the abdomen 97. Which of the following could be the BEST Method to
II.Analgesics as ordered gather the research data?
III.Antispasmodics as ordered
A. Nursing Audit
A. II, III, IV B. Focus Group Discussion
B. I, II, III C. Phenomenological
C. I, III, IV D. Survey
D. I, II, IV
98.What is the term used to describe the arrangement of
89.What is the condition wherein menstrual interval is 45-50 statistical data that exhibits the number of times the values of
days? a variable occurs?
A. Metrorrhagia: irregular interval
B. Menorrhagia A. Frequency distribution
C. Dysmenorrhea B. Frequency
D. Oligomenorrhea C. Skewness
D. Survey
90.What is the condition whereby during menstruation, the
patient feels lower abdominal pain that radiates to her back 99.In statistics, what is the definition of percentage? It is
down to her thighs? __________.
A. Hypergonadotrophic amenorrhea A. Whole population
B. Secondary dysmenorrhea B. A value on a scale of 100
C. Hypogonadotrophic amenorrhea
D. Primary dysmenorrhea C. A share of profits
D. Part of a whole expressed in hundredths
Anne, head nurse at the OB-DR Ward, is planning to have an 100. Based from the study’s findings, to whom are the study’s
in-service training for 25 staff nurses. Based from needs recommendations generally addressed? Using the situation a
assessment, the training will be on Lactation for 3 days. forecited, the recommendation should be addressed to
_________.
91.In order to make the activity a success, which of the .Nursing educators
following resources would be MOST important? III. Nursing Service Administrators
I.Staff nurses
A. Number of speakers IV. Hospital Administrators
B. Location of venue
C. Number of Participants A. I, II, III
D. Budget Allocation B. I, III, IV
C. I, II, III, IV
92.Which of the following resources should be requisitioned D. II, III, IV
as TRAINING MATERIAL/ SUPPLY?
A. Bond Paper
B. Sound system NP3
C. food and drinks
https://docs.google.com/forms/d/e/1FAIpQLScGIaRj95_ki of Spironolactone (Aldactone) 25 mg. /day. What adverse
eMAOM_gv1Xma1Ee1vdUxHLoW_IZ50FkL-kBfw/viewscor effect of the drug should you monitor?
e?viewscore=AE0zAgBUzelK_OR8GEOkvvSt0OvytIpfYPzi
AJg7Zhk9PEO4HVwiAIy7niVAjLanjJ2J2Pk A. Hyperkalemia: arrhythmia
B. Palpitation
Mr. GI Joe is 55 year old, married, a car dealer has consulted C. Irregular pulse rate
the ER because of on and fever, indigestion, D. Hypokalemia
weight loss, right abdominal pain, body malaise, and
itchiness of the skin. Based on the health history the patient Juliet, a Pediatric Cardiovascular Nurse is assigned to take
has been a chain smoker and drinks alcoholic beverages care of 3 children in her unit. She just admitted a
almost every day especially when he has clients to entertain. 12- year- old female patient suffering from Rheumatic
His physical examination showed he has suspected liver Fever (RF)
cirrhosis. He was advised for admission for further work-up
and treatment. 6. When doing an assessment, which ONE of the following
conditions is NOT present based on the Jones Criteria?
1. You are the nurse on duty when Mr. GI Joe was admitted in
the pay floor. In your observational data what additional A. Erythema Marginatum
EARLY SIGN of liver cirrhosis do you expect patient to B. Subcutaneous nodules
manifest? C. Chorea
D. Bronchopneumonia
A. Gonadal atrophy
B. Hypotension
C. Splenomegaly 1 major + 2 minor
D. Ankle edema
7. In Rheumatic fever, the factor contributory to the
Liver cirrhosis:alcohol, low testosterone, testicular tissue development of this disease process is brought about by
damage: testicular atropy __________.
Early A. Auto-immune reaction to streptococcal infection
Loss of appetite B. Auto-immune reaction to collagen disease
Fatigue C. Parents had rheumatic heart disease
NV D. Exposure to colds and droplet infection
Fever
Unexpected weight loss 8. Marimar, 6 years old, is receiving digitals for having
Patent Ductus Arteriosus with heart failure with heart
Late failure. Prior to the administration of the medicines, which
Easy bruising,bleeding of the following nursing actions should be done by nurse
Jaundice Rica?
Pruritus
Swelling, edema A. Take the carotid pulse rate for one full minute.
Ascites B. Take the heart rate for one full minute( apical heart rate)
Personality changes C. Take the respiratory rate
D. Take the blood pressure
(+) Inotropic/ pump
2. Which of the following statements is TRUE of liver (-) Chrono topic/ rate
cirrhosis? (-)Dromotropic/ conduction
1. Nutritional deficiency with decreased protein intake 9. While Nurse Juliet was doing her regular rounds, one of
attributes to liver damage. the mothers approached her saying that her child who has
2. Cirrhosis can happen to people with alcohol intake a Tetralogy of Fallot preferred to squat on bed rather than
3. Women are at greater risk for the development of assuming a supine position. The BEST action of the Nurse
alcohol-induced liver disease is ___________.
4. Most patients affected by liver cirrhosis are between 40 to
60 years of age. (30-40) A. Let the mother continue the position of her child if she
feels comfortable with it.
A. 1, 2, 3, and 4 B. Divert the child’s attention by letting her join the nurses’
B. 1 & 2 rounds
C. 1, 2 & 3 C. Put her on bed and start to administer oxygen inhalation
D. 2 & 3 D. Discourage the child and let her lie on bed with 2 pillows
instead.
3. During Mr. GI Joe’s confinement he developed further
itchiness of the skin and jaundice. Which of the following
nursing actions is NOT recommended as this will induce skin 10. Another patient in the Unit is Nikki, 8 years old, waiting to
breakdown? be scheduled for surgery because of a Coarctation of the
Aorta. With this disorder, Nikki will manifest which one of the
A. Add baking soda when bathing the patient. following symptoms?
B. Massage the skin with emollients every 2 hours.
C. Use of commercial soaps and alcohol-based lotions. A. Murmur and bruit of the lungs
D. Rub the itchy skin with knuckles instead of using the B. Central and peripheral edema
nails. C. Hypertension and diminished pulses in the extremities.
D. Hepatomegaly and shortness of breath
4. At early stage of Mr. GI Joe’s disease process, the physician
ordered this SPECIFIC diet for Mr. GI Joe, You emphasized to Mr. Gaudencio Suarez is a surgical nurse in a medical center
the dietitian that he should be served foods that is _________. and he had been practicing for almost five years
in this unit. The staffs go on shift rotation. For that shift, he
A. High carbohydrate and low sodium ( ascites and edema, had two patients for operation. One is for Lobectomy and
lessen fluid retention) high protein okay another is for Nephrectomy
B. High calorie and high carbohydrate
C. Low protein and high fat 11. When is the BEST time for the operative consent to be
D. High protein and high fat signed by his patients?
5. Mr. GI Joe started to develop ascites and complained of A. As Soon as the surgical procedure is explained to the
heaviness of the lower extremities to the physician. An order patient
B. On admission when the relatives are around heart rate. Which of the following outcomes would indicate
C. Aday before the surgery that his medication is achieving its desired effects?
D. Before any pre-op medications are administered
A. Improved appetite
12. Which of the following nursing goals are achieved when B. Increased pedal edema
early ambulation is done by post-surgery patients? C. Increased urine elimination
D. Improved bowel elimination
1. Improved circulation
2. Improved respiratory functions 19. The other patient of Nurse Brilliantina is suffering from
3. Prevent venous stasis Buerger’s disease. He has been complaining of intermittent
4. Prevent emboli formation claudication of the lower extremities which has been giving
him so much discomfort. What is the Best nursing action she
A. 3 and 4 should perform to address the patient’s complaint?
B. 1, 2, 3,4
C. 1, 3, 4 A. Allow the patient to lie flat on bed
D. 1 and 2 B. Teach him foot care and leg exercises.
C. Place affected extremities in a dependent position
13. Which of the following patients is MOST at risk during the D. Apply hot water bag to the affected extremities
induction of anesthesia?
20. Which of the following information is NOT True of
A. 6-year-old with history of allergy Buerger’s disease?
B. 65year old diabetic woman
C. 35 year old TB case A. Small and medium arteries and veins are mostly
D. 25 year old Polio Case affected.
B. Smoking is a major cause of Buerger’s disease.
C. Incidence of Buerger’s disease is high in men than
Risk factors for Anesthesia women.
● Allergies to anesthesia or a history of adverse D. Astrong relationship exists between diabetes and
reactions to anesthesia Buerger’s disease.
● Diabetes
● Heart disease (angina, valve disease, heart failure or Gloria, 58 years old, beautician has undergone bowel
a previous heart attack) obstruction surgery. During his first day post-op. she vomited
● High blood pressure clear liquids about three times during your shift. Her vital
● Kidney problems signs include: temp-37.9 degree C. 138/84 PR-78/min and RR-
● Lung conditions (asthma and chronic obstructive 26/min. her surgical incision is intact, slight bleeding, swelling
pulmonary disease, or COPD) and tenderness with slight pain.
● Obesity
● Obstructive sleep apnea 21. Which of the following manifestations would indicate the
● Stroke development of wound infection?
● Seizures or other neurological disorders
● Age A. Presence of elevated red blood cells count.
B. Profuse and increasing perspiration.
C. Increased blood pressure, respiratory rate and pulse rate.
D. Increasing pain on the surgical incision.
14. As a circulating nurse, which of the following is your
PRIORITY nursing action to promote safety of your patient? 22. If the wound draining is serosanguineous, it means that
the secretion is colored ______.
A. Prevent peri-operative position injury
B. Provide adequate lighting during the procedure A. Yellow-tinged
C. Optimize surgeon’s access to surgical site B. Pinkish-red
D. Maintain surgical aseptic technique during the C. Green-tinged
procedure D. Bright-red
15. Being the nurse in the Post-Anesthesia Care Unit, which of Sero: clear
the following is your PRIORITY concern after surgery? Sanguineous: blood
A. Monitor vital signs
B. Observe surgical site for bleeding 23. Later during the day, Gloria became confused, started to
C. Validate doctor’s orders pull out her IV fluids as well as her wound dressings.
D. Check level of consciousness Complained of severe pain over the wound area. The
PRIORITY action of the NURSE is to ________.
16. There are several factors contributory to the development
of these conditions. Which of the following are the PRIORITY A. Apply sterile dressings over the surgical wound
causative factors that should be emphasized by Nurse Lydia B. Call the surgeon at once for the health status of the
in her health teaching? patient
C. Administer the pain medication at once
A. Cigarette smoking D. Restrain the hands of the patient
B. Aging process and alcohol intake
C. Stress and over acidity 24. The drug of choice of pain relief to patient Gloria
D. Sedentary life and obesity considering score of 8 which was assessed by the nurse
based on the rating scale of 0-10 (0-lowest, 10 is the highest)
17. Mr. Jesse is admitted due to congestive heart failure. is ________.
Nurse Brilliantina would expect that if the failure is on the
right side of the heart, the patient will manifest which of the A. Morphines S04 ( if pancreatitis don’t give)
following. B. Ibuprofen
C. Demerol
A. Jugular vein distention D. Paracetamol
B. Crackles on auscultation
C. Dry productive cough 25. When patient is receiving opioid medication, the nurse
D. Improved bowel elimination should observe complications from the side effects by ______.
Systemic s,sx A. Checking for the presence of loose bowel movement
B. Observing signs and symptoms of addiction
18. The patient has been receiving, Digoxin ( cardiac C. Assessing changes of circulatory status
peripheral, kidney function);0.25 mg per day to regulate his
D. Assessing wakefulness of the patient at least every 2 B. Fast at midnight, but can drink water if he becomes
hours thirsty.
C. Instruct him to void before going to the operating room.
CNS: BRAIN STEM D. Teach him leg and coughing exercises
RESPIRATORY
33. While giving health instructions to Anthony. He looks very
Mr Rodney, a post- operative, was given Amoxycillin at 12 anxious and said to you, “I might not be able to wake from
midnight by the nurse on duty (NOD). The physician asked anesthesia after my operation.” Your BEST response is
the patient during his rounds if he has receive the drug, the _________.
patient answered “NO”. The physician clarified the claim
from the chart and from the chart and from the NOD. A. “Do not worry, everything will be alright.”
B. “Let me tell to your family your fear.”
26. When the physician clarifies the process of Drug C. “I will not let you go to the OR until I have talked to your
Administration, this Quality Improvement Process is known as surgeon.”
D. “Don’t Say that, I will discuss your concerns with your
A. System analysis anaesthesiologist.”
B. Root cause Analysis: SYSTEM, not individual
performance 34. Mr. Anthony returned back to the ward, after recovering
C. Problem identification from anaesthesia. He complain of pain over the scapular
D. Pareto Chart region. Which of the following nursing actions should you do?
27. Which of the following is the primary reason why the A. Apply heating pad over the painful area.
physician wanted to clarify the process? B. Turn client from side to side.
C. Give demerol as ordered.
A. Measures the turnaround time of medication D. Apply cold pack over the affected area.
B. Better communication with patient and result
C. Identify gap in the medication administration 35. After 5 days, Mr. Anthony is ready to go home. His
D. Improve care management in drug administration discharge instructions include observations of the following
complications, EXCEPT:
28. While the medical director is conducting his rounds he A. Palpitation
observed the presence of scattered supplies and equipment B. Loss of appetite
along the corridor as well as 02 tank not properly secured. C. Increased body temperature
The Quality Circle (QC) team was informed about this D. Bleeding
observation. Which of the following is the BEST strategies to
be adopted by the team with this concern? Meg is assigned as charge nurse at the Cardiovascular
Ward in a Medical Center. There are 20 patients,
A. Adopt the 5 S audit. classified as follows: 2 Total Care (TC), Partial Care (PC)
B. Identify and prioritize the problem and 9 Ambulatory Care (AC). The ward’s morning duty are 3
C. Select Alternative Solution. staff nurses and 2 nursing attendants (NA).
D. Enhance Continuous quality improvement
36. How will your distribute the patients to the 3 Registered
29. The approach adopted by quality circle team was Nurses and 2 Nursing Assistants?
presented to top management. Which of the following tools is
APPROPRIATE to be utilized as a guide for their time A. 2 Registered Nurse for Total Care patients, 1 Registered
management? Nurse and 1 Nursing Assistant for Primary Care Patients, 1
Nursing Assistant for Total Care.
A. Pre chart B. 1Registered Nurse and 1 Nursing Assistant for Total
B. Radar chart Care Patients, 1 Registered Nurse and 1 Nursing Assistant
C. Process of flow chart for Primary Care Patient, 1 Registered Nurse for
D. Gantt chart Ambulatory Care patients.
C. 2 Registered Nurse for Total Care patients, 2 Nursing
30. In order to determine if the implementation of the activity Assistant for Primary Care Patients, 1 Registered Nurse for
is successful, the First step to be done by the team is Ambulatory Care Patients.
______________. D. 1 Registered Nurse for Total Care patients, Registered
Nurse and 1 Nursing Assistant for Primary Care patients, 1
A. Prioritize the problem Registered Nurse and 1 Nursing Assistant for Ambulatory
B. Coordinate with the different units Care patients
C. Have a random report to the supervisor
D. Develop a monitoring plan 37. In as much as this is a cardio-ward what basic
equipment/materials should be available at all times?
Nurse Tin is assigned in the Surgical Ward of a Tertiary 1. o2
Medical Center with several patients who are for diagnostic 2. Ambu Bag
tests and for surgery. 3. Suction Apparatus
4. Endotracheal
31. Mr. Anthony, 38 years old, is scheduled for Laparoscopic
Cholecystectomy in the morning the next day. He verbalizes to A. 1, 2, 4
you and says, “Dr. Unido will open me up to remove my C. 1, 2, 3
gallstones?” which of the following is your BEST response? B. 1, 2, 3, 4
D. 2, 3, 4
A. “Let us discuss with the surgeon your operation before
you go to the OR.” 38. Which of the following emergency medicines should be
B. “I know the surgeon will remove your stones with a new available at the emergency cart at the Cardio Ward?
technique.” 1. Epinephrine 3. Valium 2. Demerol 4. Coumadin
C. “Let me call the OR supervisor to discuss the procedure
with you.” A. 1, 3
D. “I will call the anaesthesiologist to explain more the B. 1, 2
procedure.” C. 2, 4
D. 3, 4.
32. A critical component before Mr. Anthony goes for surgery
is the nurse’ preoperative teaching. Which of the following is In every Nursing Unit/Ward, the following should be available
NOT advisable prior to surgery? as mandated by the DOH in a tertiary hospital:
Bag-Valve-Mask, Height & Weight Scale, Defibrillator, E-cart,
A. Teach him deep breathing exercises Opthalmoscope, otoscope, laryngoscope, bed with railings
and lock, bedside table, Nebulizer, Neurologic Hammer,
Oxygen Tank, Sphygmomanometer, Stethoscope, Suction
apparatus and Non-mercurial Thermometer. 45. The following are the practice guidelines in
documentation. Which ONE of these is NOT recommended to
be written in your charting?
Department of Health Guidelines for Emergency Cart needs :
Adenosine, Amiodarone, Ant-tetanus, Aspirin, Atropin A. Had urine output of 600 cc during the whole shift.
Salbutamol, Diazepam, Midazolam, Calcium gluconate, B. “Appears to be comfortable with good night rest and
clopidogrel, D5W, D50W, Digoxin, Diphenhydramine, sleep”
Dobutamin, dopamine, epinephrine, Furosemide, haloperidol, C. Refused to eat her dinner as she feels nauseated at meal
Hydrocortisone, Lidocaine, Magnesium, Sulfate, Mannitol, time
Methylprednisolone, Metoclopromide, morphine, NTG, D. Slept from 11 PM to 6 AM after Valium 5mg. was given.
Noradrenaline, Paracetamol, Phenobarbital, Phenytoin, PLR, The following situations are related to ethical practices in
PNSS, KCl, Vitamin B1/6/12 vial, NaHCO3 and Verapamil the workplace.
39. When stocking fluids in the emergency cart. What should 46. Jillian a registered nurse has signed a contract for two
Nurse Meg instruct the nurses? years in a medical center. A recruitment agency has offered
1. Put the recently- delivered at the back of the old stock her a “fat” salary for foreign employment. If you are in the
2. Put the old stock in front of the newly-delivered ones position of Jillian, what should you do?
3. Check the fluids for sediments and discoloration
4. Keep the fluids inside the cooler A. Discuss your plans with your parents
B. Accept the recruitment offer and go Absence without
A. 1, 2 official leave.
B. 1, 4 C. Write a letter of resignation to your employer 30 days
C. 3, 4 before leaving.
D. 2, 3 D. Accept recruiter’s offer and give an excuse to employer
on behalf her “sick” mother.
40. How will Nurse Meg arrange the patients in the ward? Put
the ______________. 47. Karylle is a BSN graduate but did not take the National
Licensure exams. She is in dire need of money so she applied
A. Primary Care in between the Ambulatory Care patients. and was accepted in a health facility. As a nurse she
B. Primary Care and Ambulatory Care patients together and submitted a fake PRC license in order to be given a job. Riza
Total Care at the corner of the room. is liable to be charged of ___________.
C. Total Care patients near the nurses and the Ambulatory
farthest A. False statement
D. Ambulatory Care patients near the Total Care ones. B. Misrepresentation
C. fraud and deceit
Mr. Nelson, 62 year old, an Executive of a shoe company D. malpractice
was brought to the hospital after having vomited
bright red blood immediately after supper. He claimed he 48. Ms. Kendall an R.N was invited as a resource person in a
had drinking session with his former classmates in college. nursing conference. The curriculum vitae she submitted to the
He was already advised by their family physician not to PRC Continuing Professional Development Council reflects
drink alcohol due to a suspected fatty liver. You are the the word Masters of Arts in Nursing (MAN) after her name.
nurse-on-duty when he was admitted She only completed academic requirements but not her thesis
writing. Which of the following did Ms. Paras violate?
41. Based on your assessment and history taking for Mr.
Nelson which PRIORITY FINDINGS should you document and A. RA 9173 of Nursing Law
report to the physician? B. Code of Professional Ethics for Teachers
C. Magna Carta for Health Workers
A. Use of anti-inflammatory drugs.( GI injury, liver) D. Code of Ethics for Registered Nurse
B. Vital signs BP- 140/90, PR- 88/min.PR24/min.
C. abdominal pain (3 in a scale of 10) 49. It is a hospital policy for nurses in a health facility that
D. Tense, rigid abdomen reporting time is 15 minutes before the official designated
time. Nurse Kh
42. You have formulated your Nursing Diagnosis for the
patient and wrote in the nursing care plan. Which ONE of the A. Dishonesty
following? B. Falsification of document
C. Unprofessional conduct
A. Deficient Fluid Volume R/T vomiting of blood and D. All of the options
gastric secretion.
B. Non-compliance R/T alcohol and medication intake. 50. What ethical justification is observed when administering
C. Fear of death R/T unknown cause of bleeding. Opioids despite the possibility of hastening death.
D. Risk for Aspiration R/T active bleeding.
A. Rule of double effect
43. The physician orders insertion of nasogastric tube with B. Rule of single effect
lavage to Mr. Nelson. What kind of solution will you expect to C. Maleficence
be written in the doctor’s order when a patient will undergo a D. Justice
lavage?
A. Normal Saline Solution 51. Which component of chest physiotherapy involves
B. Distilled Water rhythmic clapping with cupped hands on the chest wall
C. Tap warm water over the area being drained?
D. Dextrose 5% in water
A. Postural drainage ( gravity)
44. When a patient has a gastro-intestinal bleeding and there B. Percussion
is a presence of hematemesis, how should you describe this C. Nebulization- vapor inhalation
in your documentation? D. Vibration- gentle shaking pressure
A. Bloody vomitus appearing as fresh, bright red or 52. Which among these would indicate that coughing was
“Coffee-ground” in appearance. ineffective? ( No expectorate)
B. Brownish vomitus appearing as “Chocolate” in
appearance from a previous Food Intake. A. Decreased normal breath sounds
C. Black, tarry stools in appearance often foul smelling, B. Clearing of adventitious breath sounds
from a previous food intake. C. Sputum expectoration
D. Small amounts of fresh blood observed either through D. Patient’s report of swallowed sputum
gastric secretions or stools.
53. Postural drainage improves clearance and drainage of D. talk with the family members and discuss the demanding
secretions by which principle? behavior of the father.
A. Entropy 60. Nurse Berroya is taking care of Ms. Gonzaga with a
B. Enthalpy multi-organ disorder. The health care team believes that
C. Gravity patient’s survival rate is only 50%. Which of the following
D. Motion nursing actions is MOST APPROPRIATE to be done by Nurse
Berroya?
54. A goal for a post-operative patient using an incentive
spirometry is: (inhale)prevent atelectasis A. Avoid discussion of death when family members are
present.
A. Decrease pulmonary ventilation B. Arrange with the family the available funeral parlors
B. Expand collapsed alveoli incase of death of the patient.
C. Potentiate effects of general anesthesia C. Request the husband to prepare a living will.
D. Thicken respiratory secretions D. Inform the family members that there is a palliative care
available.
55. As safety precautions for clients receiving oxygen,
flammable)which among these should Nurse Tin not do?
The coordinator of the In-Service nursing department
A. Place a “No Smoking” sign on the client’s door and at the prepared an enhancement program for a newly
foot or head of the bead. board passers on Universal Health Precaution as part of their
B. Replace woolen blankets with cotton blankets. personal and professional development. ( SMART)
C. Place and arrange substances like acetone, alcohol, and
oils at the bedside. 61. Which of the following example is a well stated objectives
D. Ensure that all electric devices are in good working order. for this proposal?
A. Utilize the Universal health precaution in dealing with
self and environment.
B. Understand the health precaution in dealing with self and
Mr. Berroya, who has just finished his one (1) year period as a environment.
registered nurse in a medical center, is assigned in the C. Know the value of Universal health precaution in dealing
medical unit on 3-11 shift. He is having a difficulty interacting with self and environment.( Know not measureable)
with the patient’s family so he consulted his senior Nurse D. Acquire knowledge on health precaution in dealing with
regarding this matter. self and environment.
56. The BEST approach to be employed by Mr. Berroya upon 62. The group came up with an effective Strategy in the
the advise of his senior nurse is __________. teaching of Universal Health precaution to the attendees.
Which of the following is the BEST strategy for this program?
A. send the family members home
B. confront with the family regarding their behavior A. Film Showing
C. ignore family’s behavior B. Case Analysis
D. use active-listening technique C. Lecture with interactive discussion
D. demonstration and return demonstration
57. The Senior Resident in the medical unit was making his
regular rounds with the medical interns and Nurse Berroya 63. Before the implementation of the program, it is important
was called to receive his orders for Mr. Peeta who has been to know pertinent information about the participants. Which
confined in the unit for almost a week. Nurse Berroya could of the following INITIAL steps should be undertaken by the
not understand some of the written order orders of the organizing committee?
physician in the chart. Which of the following is the
appropriate action of the nurse? A. Prepare a budget plan for the program.
B. Organize the learners into groups per gender.
A. refer to the nurse supervisor to validate the Physician’s C. Present the detailed program to management.
orders. D. Come up with the learning needs of the participant.
B. approach the physician and clarify his orders
C. call his junior resident to rewrite his orders 64. A Gantt Chart has been devised by the Organizing
D. complete the orders by rewriting what is in the chart. Committee as the program will be conducted for 4 weeks’
time.
58. One of the patients of Mr. Berroya told him that according A. Determining possible budgetary implications.
to his attending physician he would be transferred to the B. Schedule for meeting the objectives for the program.
surgical unit because of a suspected stone in the gall bladder C. Coordination needed between and among the
causing his on and off pain later in the evening. The wife participants.
however, told Mr. Berroya “No, the physician told me, my D. time frame for the completion of the program with set
husband will still be in this unit for 2 more days”. The BEST goals.
action of the Nurse should ____________.
65. Which of the following statements of the participants
A. call the physician to clarify management of care for the indicate that the program is effective?
patient. A. Its good we have earned our CPD unites required.
B. instruct the couple and family members wait for the B. We gain weight after the completion of the program.
decision of the health team. C. There should be more time allotted to the session.
C. inform the patient that sometimes the physician need to D. I am happy that we have met all the objectives set.
have more data available.
D. inform the couple that the decision of the physician is Nurse Kim, a research coordinator is aware of the concept of
sometimes not fixed research as a competency expected as a graduate nurse,
especially in this era of evidence based nursing practice. The
59. The members of the health team in the medical unit has following questions relate to research in
been reacting negatively to a patient who has been branded nursing practice.
as “Very demanding” He has a diagnosis of Emphysema and
had been confined for almost 2 weeks in the ward. Family 66. The ethical codes in research have greatly influenced the
members come and go to visit him. The best approach of the conduct of scientific studies especially among human beings.
health team should be _______________. The declaration of Helsinki( safety) includes which of the
following ethical standards?
A. request a chaplain to regularly visit the patient. 1. Investigator should protect life, privacy and dignity of
B. hold a team conference on how to approach the issue human subjects
C. request for a psychological counseling
2. Investigator should protect from harm in therapeutic A. A standing order is carried out as specified until
studies discontinued for another order.
3. Informed consent is not essential when doing research B. When the physician writes the drug as needed in the
among human subjects order sheet.
4. Investigator should conduct research when objectives C. A single dose of the drug, carried out by the nurse at a
outweigh risks and burdens to the subjects time specified by the physician.
D. An order written by the physician that has to be carried
A. 1, 2, 3 & 4 out immediately.
B. 1 & 2
C. 1, 2, & 4 73. When a patient is in pain, what PRIORITY consideration
D. 2 & 3 should Nurse Liza observe?
67. Nurse Kim, was sent by her chief nurse to attend a Forum A. Goal of treatment of the health professionals.
conducted by the Philippine Nursing Research Society on the B. Perception of pain of the individual.
topic “Effectiveness and Efficacy of the wound dressing to C. Harmful effects of the drugs.
D. Cost of the pain medication.
patients with bed injury”. She would like to adopt intervention Pain- subjective( pain scale)
in their unit. What is the MOST appropriate action to be done
by Nurse Kim when she returns to their hospital? 74. Prior to the administration of pain medication, Nurse Liza
assesses the degree of the pain of patient before and after
A. Pilot the study to determine if results will be the same the drug administration. Using the numeric scale of 0-10 (0 is
B. Incorporate the intervention into their hospital manual the lowest and 10 is the highest) on pain, when the patient
C. Convince the chief nurse to change their wound has rated her pain as 7 it means it is ___________
management practices ,0 no pain
D. Request investigator if she can adopt the research 1-3 mild
protocol in their unit 4-6 moderate
7-10 severe
A. severe
B. tolerated
C. Slight
68. The quality assurance nurse who works closely with nurse D. moderate
Kim request her to conduct a study by comparing the average
length of hospitalization of cardiac patient in a regular ward 75. In educating effectively the patient and family on how to
with that of patients with same disorders in an intensive care use a pain rating scale, the following are the STEPS to be
unit. What statistical test is MOST appropriate to use in this followed by the nurse __________
study? . 1. discuss pain as a broad concepts
2. set goals for comfort and function
A. Analysis of variance( 3 or more variables ratio interval)( 3. explain parts of the rating scale
Annova) 4. show pain rating scale and purpose
B. T-test - (two variables that are ratio or interval) 5. verify if patient understands broad concepts of pain. 6. ask
C. Chi-square( frequency/ categorical: likert scale) patient to practice the pain rating scale
D. Coefficient of correlation ( relationship)
A. 1, 2, 3, 4, 6 & 5
69. The research team together with the gastro-intestinal B. 4, 3, 1, 5, 6 & 2
department also conducted a study on the alcohol C. 2, 4, 3, 1, 6 & 5
consumption of patients as heavy drinkers and developed D. 3, 1, 2, 5, 4 & 6
liver cirrhosis over a 20 year period.
Communication is a basic component of person to person
A. Follow-up( retrospective: coast efficient relationship. In a clinical setting nurses are evaluated by
B. Cross-sectional( now)( study the present patient to be efficient in carrying out activities of daily living
C. Trend but sometimes limited in employing routine
D. Panel bedside care.
70. Jenny with her team would like to conduct a research on 76. Nurse Helen is taking of an older person and has been
“Perceptions of professional Health practitioners on the waiting for almost two hours for her son to arrive. She is
competence and level of acceptance on Nurse-Led Clinic”. ready for discharge. Which of the following statements is an
What type of research design should nurses Kim and her example of a therapeutic communication?
team adopt? A. “I can call your son by cellphone to remind him that you
are waiting.”
A. Experimental B. “Your son must be in a traffic jam right now.”
B. Correlational C. “I’ll stay with you until your son arrives.”
C. Descriptive- level perspective D. “Would you want to watch the television while waiting?”
D. Quasi-experimental
77. You are teaching a 66 year old male diabetic client about
the effects of insulin. He is ready for discharge the next week.
Nurse Liza is assigned on a 3-11 shift in the female medical Which of the following is a statement of a helping relationship
ward of a Tertiary Hospital. Patient Alma has been with the patient?
complaining of excruciating abdominal pain and was asking
for pain medication. She was referred to the physician on duty A. “In a few days I’ll review what you have learned
(POD) for an immediate relief of pain about actions and effects of your insulin.”
B. “You keep on forgetting what I said but will try to repeat
71. In addition to the identification bracelet worn by the some of them.”
patient, how can the nurse on duty (NOD) verify the identity of C. “I already mentioned those actions and effects of insulin,
her patient when administering the drugs? I think you forgot some of them.
D. “Would you like me to discuss the information to your
A. Ask the question, “Are you Ms. Alma?” wife? She can assist you?”
B. Read the patient’s name at the foot of the bed
C. Ask the patient’s name from the watcher. 78. A patient is scheduled for cholecystectomy. She
D. Ask the patient’s husband’s name- immediate verbalizes to the nurse in an anxious manner “Will my surgery
relative be too long?” The BEST response of the nurse is __________.
A. “I have undergone the same surgery, its a very quick
72. The physician orders a PRN pain medication to patient operation.”
Alma. When will Nurse Liza administer a PRN drug? ( As B. “Your doctor is an expert, don’t worry about how long.”
needed: not single dose)
C. “Your surgery is at 9AM tomorrow. And you’ll stay in Len, a 35 year old dressmaker has been experiencing
the recovery room for an hour. You will be back in the recurrent episodes of abdominal pain, nausea and vomiting
room before 12 noon.” and feels her stomach is bloated. She has been taking
D. “The operation will not last long. By the time you are contraceptive pills in the past. She is married with three
awake you will be already in your room.” children. She consulted the OPD and was advised by the
physician to be admitted for suspected Pancreatitis.
79. When the nurse explores thoughts and feelings about the
patient’s surgery, this phase in the helping-relationship is in 86. Nurse Gladys started her admission care to Ms. Len.
_______________. Which of the following laboratory examinations do you expect
A. introductory phase the physician to order for the patient?
B. termination phase
C. resolution phase A. Serum lipase and Amylase- liver
D. working phase (peplau) B. Creatinine & Phospotase- kidney function
C. Serum Transaminase
80. Which of the following major types of learning strategies D. Urea Nitrogen Substance
can you offer for a Geriartric patient who has difficulty in
hearing? 87. As a nurse, which of the following assessment data you
A. practice will MOST likely NOT to find on patient Len?
B. demonstration
C. printed learning materials A. Abdominal and back pain with tenderness
D. role playing B. Cramping pains before intake of heavy meals.
C. Pain unrelieved by ke of antacids
Nurse Zanjoe is assigned in the Medical Ward for 7 years. He D. Mid-epigastric pain acute in onset after heavy meals
is well appreciated by his colleagues and patients because he
intercedes with patient’s need, he has good decision making 88. The MOST useful diagnostic test is to validate whether
ability, and respect the patients’ rights and cultural practices. Ms. Len is suffering from Pancreatitis is for her to undergo
He helps colleagues beyond call of duty. ___________.
81. A patient who has craniotomy was referred by Nurse Greg A. Endoscopic Retrograde Cholangio Pancreatography
to Department of Social Welfare (DSWD) because he has B. Endoscopic Ultrasonography
problem on paying his unsettled hospital bill amounting to 80 C. Percutaneous Transhepatic Cholangiography
thousand pesos. The relatives are afraid the patient will not be D. Cholesterol Serum level
discharged soon. Which of the following function BEST 89. When managing a patient with acute pancreatitis the first
describe the action of Nurse Greg? priority is to ____________.
A. Advocate A. insert urinary catheter for adequate elimination.
B. Caregiver B. encourage oral fluids to improve elimination.
C. Educator C. insert nasogastric tube to decompress stomach
D. Clinicia D. administer IV fluids to replace electrolytes lost.
82. Nurse Zanjoe notice a colleague taking frequent breaks, 90. One of the SAFETY alerts that the Nurse-on-duty (NOD)
working over-time, inaccurate drug accounts and displaying will have to watch for patient with acute pancreatitis is
inappropriate behavior during meetings and conferences. ___________.
Nurse Zanjoe suspects her colleague is ___________.
A. Diabetes insipidus
A. a victim of domestic violence B. Respiratory distress
B. with personality disorder C. Hypercalcemia
C. on substance abuse D. Pericarditis
D. not mindful of mindful work output
Norilee, an accountant, 29 years was rushed by her
83. Nurse Zanjoeis assigned on night shift and noted a husband in the ER because of body weakness, palpitation,
co-worker with a tourniquet wrapped around the upper arm. confusion and diaphoresis. Her blood glucose is 450 mg./dl
The co-worker is about to insert needle attached to a syringe with fruity acetone smell on her breath. She was diagnosed
containing a clear liquid into the antecubital area. Zanjoe’s to be suffering from type I diabetes mellitus 6 months ago.
initial action is to call the ______________.
91. Which of the following the clinical characteristics are of
A. nurse supervisor Type I diabetes mellitus EXCEPT _________________.
B. physician
C. security guard A. often have islet cell antibodies
D. watcher B. Ketosis prone when insulin is absent.
C. Onset any age, above 30 year old, usually obese
84. Nurse Zanjoe made a conference with a preoperative D. Onset any age, below 30 year old, usually thin
patient on important data related to her illness. To practice
confidentiality of information, Nurse Zanjoe MUST consider 92. Patient Norilee upon admission has a breath
which of the following? characteristics of fruity acetone, this is brought about by
the presence of ______________.
A. Documents the process of conservation to serve as
proof for future reference. A. lactic acids
B. Apply technology for easier and faster communication B. uric acid
and documentation. C. nitric acid
C. Conducts the conference in a private place where D. ketoacids
conversation is not heard.
D. Invites others to interpret the discussion if patient is deaf
to unfamiliar with the language 93. Based on the presenting manifestations of patient
Norilee you expect that the physician will likely order which
85. A patient refuses to undergo surgery after a conference of the following treatment?
with her doctor. Which of the following ethical principle
applies when Nurse Zanjoe and doctor respect the decision of A. 50% dextrose
the patient in the spite encouragement and pieces of advise B. 5% Lactated ringers
given? C. Dextrose 10% in water
D. normal Saline solution
A. Autonomy
B. Beneficence 94. When caring for patient Norilee, which of the following
C. Veracity should be included in your teaching plan?
D. Self-Determination 1. Signs and symptoms of hypoglycemia
2. Self-glucose monitoring
3. Administration of insulin A. Anger due to pain experience
4. Meal planning and exercise B. Feeling of excessive guilt
C. Anorexia and weight loss
A. 2, 3, 4 D. Inability to care for one’s physical self
B. 2, 3
C. 1, 2 2. The client has difficulty sleeping. Which of the following
D. 1, 2, 3, 4 interventions is LEAST helpful for Nurse Yolan to incorporate
in her care plan?
IV- REGULAR
A. Instruct the client to drink herbal tea.
95. When formulating a Nursing diagnosis for Ms. Norilee B. Give warm milk at bedtime
who is suffering from type I diabetes mellitus with C. Perform relaxation routine such as massage, imagery or
ketoacidosis, which of the following will be the top priority? music
D. Instruct the client to drink black tea
A. impaired tissue integrity
B. deficient fluid volume 3. The care plan for the client includes family support. Which
C. Imbalance nutrition of the following is MOST appropriate for the family to
D. Risk for infection establish a relationship with the health care team? Nurse
Yolan should ___________:
Ms. Arce is the Research Coordinator of the Medical Center
who is concerned about the increased incidence of A. Give permission to the family to take time to maintain
nosocomial infection in the hospital and has convinced the friendship with the health care team. ( Professional)
management to allot a budget for the conduct of research B. Discuss the roles of the family members to the health
on the clinical problem care team.
C. Explain the roles of all members of the interdisciplinary
96. What type of research is appropriate for this study? team.
D. Provide a brief explanation to the family member about
A. Quasi-experimental( NO INTERVENTION the care being delivered to the client.
B. Experimental
C. Historical
D. Descriptive-correlational
14. You are aware that there is a need for you to understand 21. The family decides to wean the patient from the ventilator
how to manage the cost of client care as it relates to clinical support. The family talks to the nurse about their decision to
practice. Which of the following are nurses accountable for? get the nurses’ support. Which of the following actions is NOT
appropriate? The Nurse ___________
A. Decision regarding cost effective practices.
B. The Client’s hospital charges A. Checks the physician’s orders for sedation and
C. Distribution and consumption of resources such as analgesia and make sure that the anticipated death is
time, supplies, drugs, staff and personnel. comfortable and dignified.
D. Financial viability of nursing department. B. Explains to the family what will happen each phase of
the weaning and offer support.
15. While touring the department where you are assigned, you C. Tells the family that death will occur almost
immediately after the patient is removed from the
noticed that the supply room is stacked with medical supplies ventilator support.
D. Participates in the decision-making process by offering D. 1, 3, 4, 5
the family information
28. The nurse assesses the patient to determine the extent of
22. Two hours after the ventilator support was discontinued, injury. Which of the following signs is a CARDINAL sign of
the patient dies. The nurse discusses with the family the renal trauma?
possibility of donating the deceased person’s organs. The A. Shock
following are guidelines in organ or tissue donation. B. Lumbar pain
C. Abdominal pain
1. Religious beliefs in organ donation and transplantation D. Hematuria- dugo in ihi
must be respected.
2. Donors must be free of infectious disease and cancer. 29. The nurse writes a nursing diagnosis for the patient with
3. Consent or written orders by the physician are necessary stab wound. The MOST appropriate nursing diagnosis is
for referral to an organ procurement organization. ____________.
4. The family of the deceased should be offered an A. Nutrition imbalance, less than body requirements, related
opportunity to speak with a knowledge organ procurement to nausea from renal trauma
coordinator. B. Deficient fluid volume related to blood in the urine
5. The person requesting for organ donation does not have to C. Acute pain in the abdominal area related to renal trauma
believe in the benefits of organ donation but should support D. Acute pain in the lumbar area related to renal trauma
the process with a positive attitude.
30. The physician prescribes Magnetic Resonance Imaging
Which of the guidelines should the nurse observe? (MRI) of both kidneys to confirm clinical suspicion and
determine the severity of the injury. Which of the following
A. 1, 2, 3, 4, 5 activities is a PRIMARY nursing consideration in preparing the
B. 1, 2, 4 patient for MRI?
C. 2, 3, 4 A. Administer all medications scheduled before the test.
D. 1, 3, 5 B. Report findings of metal screening ; sedate the patient
before sending him for MRI.
23. The legal definition of death that facilitate organ donation C. Coordinate the MRI with other patient care activities
is the cessation of ________ : and inform the patient about the test.
A. Function of the entire brain D. Ensure the patient is on NPO and hold all medications
B. Pulse until test is completed.
C. Circulatory and respiratory functions ( RA 71710)
D. Respiration Nurse Ashley is a staff nurse in the oncology unit of a tertiary
9173: new hospital. She reads
7164: old literature on antineoplastic medications.
24. The patient is pronounced dead by the physician. Which of 31. Nurse Ashley understands the importance of continuing
the following nursing actions VIOLATES the standards of care professional development. Which of the following is the MAIN
for a dead person? purpose of continuing professional development? To
A. Removing soiled dressing and tubes. ____________.
B. Keeping the dead person in a sitting position until A. Update one’s professional knowledge and competence
the family has arrived and said their goodbyes. B. Acquire a certificate of attendance to add to one’s
C. Placing identification tags on both the shroud and ankle. curriculum vitae
D. Preparing to transfer the body to the morgue. C. Establish networking within the nursing profession
D. Fulfill requirements for an advanced degree in nursing
25. The family goes through the stages of grieving. What are
the stages in the grieving process? 32. Nurse Ashley reads that the drug Cyclophosphamide (
1. Acceptance Cytoxan ) is given to patients with breast cancer. Nurse
2. Depression Ashley understand that this drug is ______:
3. Denial
4. Bargaining A. Cell cycle phase-non-specific
5. Anger B. A hormonal medication
A. 3, 5, 1, 4, 2 C. An antimetabolite
B. 3, 5, 4, 2, 1 D. Cell cycle phase-specific
C. 1, 5, 3, 4, 2
D. 1, 2, 5, 4, 3 33. Nurse Ashley reads in the literature that a patient with
DABDA breast cancer taking Cytoxan( hemorrhagic cystitis) should
observe the following. Given a case what should nurse Ashley
A male teenager was wheeled in the Emergency Department instruct a patient to do?
(ED) for injured. A. Decrease sodium intake while on medication.
26. The nurse assesses the patient for complications. Which B. Take the medication with food.
are the MOST COMMON complications? C. Increase potassium intake while on medication.
1. Urinary leakage D. Increase fluid intake 2000 to 3000 mL daily.
2. Delayed bleeding from damage
3. Abscess formation 34. Nurse Ashley understands that patients receiving
4. Paralytic ileus antineoplastic medications should do which of the following?
5. Renal failure
A. 4 & 5 1. Drinks beverages containing alcohol in moderate amounts.
B. 3 & 4 2. Consult with the physician before receiving immunizations.
C. 1 & 2 3. Be sure to receive flu and pneumonia immunizations.
D. 2 & 3 4. Take aspirin (Acetylsalicylic Acid, ASA) as for headache.
27. The nurses knows that with renal trauma, further A. 2 only
complications may occur such as: . B. 3 & 4
1. Secondary hemorrhage usually due to infection C. All of the options
2. Renal artery stenosis D. 1 & 2
3. Renal atrophy
4. Hypotension 35. An incident was described in the literature where a patient
5. Hydronephrosis developed stomatitis after receiving a course of
antineoplastic medications. Which of the following actions
Which are the POSSIBLE complications? would be BEST for a nurse to do?
A. 2, 3, 4, 5
B. 1, 2, 3, 4, 5 A. Swab the mouth daily with lemon and glycerine.
C. 1, 2, 3, 5 B. Avoid foods and fluids for the next 24 hours.
C. Brush the teeth and use waxed dental floss 3x a day. 43. Nurse Mirasol writes a nursing diagnosis for the patient.
D. Rinse the mouth with diluted baking soda or saline. Which of the following is a PRIORITY nursing diagnosis?
The head nurse of a trauma unit introduce changes to A. Social isolation
improve the quality of care B. Impaired skin integrity
of trauma patients. C. Disturbed body image
D. Low self-esteem
36. The head nurse presented a set of goals to the staff
nurses. Which of the following goals is NOT relevant to 44. Nurse Mirasol assists the patient in coping with the
improving quality of care? No_______: disorder. During the early stages of a chronic disease,
patients tend to focus on which of the following behaviors?
A. Legal suits.
B. Needless deaths. A. Understanding the disease process
C. Waste of resources. B. Impact on lifestyle changes
D. Needless pain or suffering. C. Interpretations of symptoms
D. Schedule of medications
37. The head nurse reviews reports on nurse staffing. The
following findings result to better patient outcomes EXCEPT: 45. Nurse Mirasol prepares a discharge plan of care for the
A higher _______: patient. Which of the following objectives are MOST
appropriate? The patient should _______________.
A. Nurse to patient ratio shortens lengths of patient stay in
the hospital. 1. Try to prevent breakdown of the skin and ulceration
B. Nurse to patient ratio results to reduced patient 2. Avoid activities that trigger pain
mortality. 3. Modify diet to include legumes
C. Number of nurses, infection rates fall. 4. Avoid exposure to extreme cold temperature
D. Nurse to patient ratio increases costs.
A. 1, 2, 3, 4
38. The head nurse determines to reduce medication errors in B. 1, 2, 3
the trauma unit. She recognizes that medication errors often C. 1, 2, 4
occur in relation to the following EXCEPT: D. 2, 3, 4
A. Preparing the wrong concentration and administering the Nurse Bessie is a nurse manager of trauma unit. She
medication via the correct route. supervises the staff nurses and regularly holds conferences
B. Failure to question unclear medication errors. with them and other unit personnel. In one meeting she
C. Lack of knowledge about medication. reorients the staff nurses on their various functions. She cites
D. Failure to identify non-therapeutic client responses. clinical situations related to a nurses dependent,
interdependent, and collaborative functions.
39. The head nurse suggests that to reduce medication errors,
several measures will be instituted. Which of the following is 46. An interdependent function of nurse is when the nurse
MOST appropriate? _______:
Independent- nurse
A. Use point-of-care technology to access drug reference Dependent- MD/nurse
information. Collab- team
B. Use of drug index Inter- MD/nurse/ hcp/ allied
C. Nurses must help educate patients and their families
regarding proper medication A. Irrigates a feeding tube that appears obstructed.:I
administration. B. Gives ice chips to a client who has an order of NPO.: D
D. Patients must become more involved in managing their C. Applies a dry sterile dressing to an abdominal incision.: I
care. D. Helps a client choose foods rich in protein from an
ordered diet.ID
40. The head nurse is aware that managing and improving 47. A nurse decides to give a partial bath to a client instead of
quality care in the trauma unit requires which of the following? a complete bath. The nurse is working __________:
A. Independently
A. Personalized attention to patient’s needs and their B. Interdependently
families. C. Dependently
B. A blame – free environment. D. Collaboratively
C. All of the choices.
D. A clean and orderly trauma unit. 48. A nurse works with a skin care team. The nurse is
functioning _________:
Nurse Mirasol is the attending nurse of a 40-year old female A. Dependently
admitted in the medical unit B. Interdependently
with a probable diagnosis of Scleroderma. C. Collaboratively- team
D. Independently
41. The patient complains of pain in her fingertips and pallor
followed by blanching of the extremities and redness. Nurse 49. A nurse initiates a visit from member of the clergy for a
Mirasol knows that these symptoms are characteristic of terminally ill client. The nurse is functioning ___________:
which of the following disorders?
A. Interdependently
A. Swan-neck deformity B. Collegially
B. Raynaud’s phenomenon C. Independently
C. Joint swelling and effusion D. Dependently
D. Symmetric Polyarthritis
50. When a nurse uses a straight catheter to obtain a urine
specimen for laboratory test, the nurse is functioning ________:
42. Nurse Mirasol assesses the skin of the patient. Which
phase of skin changes occur FIRST and are usually painless A. Dependently
and symmetrical? B. Interdependently
C. Independently
A. Indurative D. Collegially
B. Primary
C. Curative Straight catheter
D. Edematous
Marie, an oncology nurse assists in the care of patients B. Local rash that occurs as a result of allergy.
with cancer. C. inflammatory disease of collagen contained in
connective tissues.
51. One of her patients is a 50-year old female named Marcela D. disease caused by the continuous release of histamine
is in the terminal stage of breast cancer. She tells Nurse in the body.
Marie. “I have given responses of Nurse Marie is MOST
therapeutic? 57.The nurse includes in the care plan dietary instructions.
Which of the food items should the nurse instruct the client to
A. “You have given up hope?” AVOID?
B. “You should talk to your physician about your fears of
dying.” A. steak
C. “You should talk about dying with your spiritual adviser.” B. broccoli
D. “You should not give up hope. There are research studies C. legumes
being done to cure cancer.” D. fish
52. Marcela says to Nurse Marie. “ I don’t like to spend my 58.The nurse is aware that fatigue is experienced by patients
final days on earth in a hospital.” The BEST response of Nurse with SLE. Which of the following activities should be a
Marie would be : component in the care plan for the client to manage fatigue?
To ________.
A. “Can you please tell me more how you are feeling right
now?” 1. sit whenever possible
B. “I know how you feel. It must be hard to know that you 2. take a hot shower in the morning
are dying.” 3. avoid long periods of rest
C. “If I were in your place, I should have refused being 4. engage in moderate low impact exercise when not
admitted to the hospital knowing that I will die soon. fatigued
D. “What is it that you don’t like being in the hospital?” 5. maintain a balance diet
53. Marcela tells Nurse Marie that her younger sister was A. 2, 3, 5
recently diagnosed with cancer. She is concerned because B. 1, 2, 3
she is aware that breast cancer “ runs in the family” but she C. 1, 2, 3, 4, 5
could not recall any family member diagnosed with bone or D. 1, 4, 5
lung cancer. Nurse Marie’s BEST response would be:
59.The physician schedules the client for plasmapheresis
A. “ I am sorry to hear about your sister. I think you should (blood and plasma). The client asks the nurse what is
meet with all of your family members and share with them plasmapheresis. The nurse explains that it is a method that
their increased risk for developing lung and bone cancer.” will __________.
B. “ Apparently your sister is so unfortunate . it is rare to
have three such unrelated cancers at one time.” A. prevent foreign antibodies from damaging various body
C. “ I think it is important for you to be tested for lung tissues
cancer as soon as possible , because it has hereditary link.” B. decrease the damage to organs caused by attacking
D. “ I am sorry to hear about your sister’s recant diagnosis. T-lymphocytes
Most probably your sister has a breast cancer that has C. eliminate eosinophils and basophils from the blood
metastasized or spread to the bone and lungs.” D. remove antibody-antigen complexes from circulation
54. Nurse Marie has another patient, Cena who was recently 60.The nurse monitors the client undergoing plasmapheresis.
diagnosed with ductal cell carcinoma of the breast. Her Which of the following reactions should the nurse observe?
oncologist described Cena’s cancer as T2, N1, Mx. Cena
asked Nurse Marie to repeat to her what “ all those letters and A. shortness of breath
numbers mean.” Nurse Marie replies that it means the B. numbness and tingling
following: C. transfusion reactions
D. high blood pressure
A. Two tumors present, one lymph node involved, and many
sites of metastasis. The nurse assists in the care of clients with chronic
B. One large tumor present, nodal involvement in one obstructi pulmonary disease
region, and metastasis was present. (COPD).
C. Two tumors present, one lymph node involved and
metastasis was present. 61.The nurse is aware that clients with COPD are at risk for
D. One tumor present, which is larger than 2.5 centimeters, ineffective respirations EXCEPT which of the following
nodal involvement in one region, and metastasis was _________?
unable to be determined.
A. Clients undergoing thoracic or abdominal surgery
55.Patient Cena tells Nurse Marie “How did I acquire breast B. Clients with rib fractures and kyphosis
cancer?” Nurse Marie explains that there are risk factors that C. Clients with neuromuscular disorders such as
may have contributed to her condition. Which of the following Guillain-Barre’ syndrome
statements is TRUE concerning the risk factors for breast D. Clients with fluid volume deficit
cancer?
62.Nursing interventions for clients with respiratory acidosis
A. Hormones are not a risk factor for breast cancer. include the following EXCEPT to __________.
B. Other types of cancer history have no correlation with
breast cancer. A. monitor arterial blood gases (ABGs), pH, PCO2, and
C. Ethnicity is a risk factor. HCO3
D. Environment is not a risk factor for breast cancer. B. administer oxygen and medication as ordered
C. monitor hourly vital signs and respiratory status
A 35-year old female client presents herself in the D. administer sedation as ordered by the physician to relax
outpatient Department with complaints of rashes the client
particularly on the face, across the bridge of the nose and
on the cheeks. The client is suspected of having systematic 63.The nurse understands that excess acid in the body acts
lupus erythematous (SLE). She is admitted to the female as CNS depressant. Clients with acidosis may exhibit which
medical unit. of the following symptoms:
56.The nurse writes a care plan for the client. The Nurse is 1. reduced level of consciousness
aware that this disorder is a/an ________. 2. confusion
3. lethargy
A. disease caused by over exposure to sunlight 4. coma
D. Congress should pass a bill banning all cancer-producing
A. all of the options foods and beverages.
B. 1, 3, & 4
C. 1, 2, 3 The nurses cares for a 30 year old patient who is admitted
D. 1 & 3 for severe vomiting. The
diagnosis of the patient is hypernatremia.
64.The goal for treatment for respiratory acidosis is to
improve ventilation. Which of the following measures is 71.The nurse reads the laboratory results. Which of the
appropriate for clients with COPD experiencing respiratory following values indicate that the patient is experiencing
acidosis? hypernatremia?
65.The nurse understands that respiratory acidosis occurs Osmolality: concentration of particles in serum
when __________: High osmo: viscous/ malapot
Low osmo: malabnaw
A. the body retains too much carbon dioxide 72.The nurse monitors the patient for signs and symptoms of
B. the client is unable to exhale carbon dioxide complications. The nurse knows that one of the PRIMARY
C. the client hyperventilates risks when treating hypernatremia is ___________:
D. there is loss of acid or retention of base in the body
A. renal shutdown
Nurse Mark is assigned in the oncology unit of a tertiary B. cerebral edema
hospital. He is aware of the C. cellular dehydration
increase in the number of colorectal cancer patient in his D. RBC destruction
unit. He and a colleague plan to conduct a study
of the incidence of colorectal cancer in the Philippines. 73.In planning the care for this patient the nurse includes the
following interventions. Which of the following actions should
66.Nurse Mark formulates a possible title for the study. Which the nurse NOT include in the plan of care?
of the following is the MOST appropriate title?
A. observe for possible increase in temperature
A. “Incidence of Colorectal Cancer in the Philippines” B. observe and prepare for possible seizure attack.
B. “Perceptions of the Filipinos on Colorectal Cancer” C. monitor intake and output
C. “Colorectal Cancer in the Philippines: It’s Risk Factors D. restrict fluids to 1,200 mL per day. If sodium is high,
and Interventions” temp is high
D. “A Comparative Study of Gastrointestinal Cancer Cases
among Filipinos” 74.The nurse understands that a patient with hyperatremia is
at high risk for seizure. Which of the following safety
67.What research design is the MOST suitable to gather data measures is MOST appropriate? Use of _________.
for the study?
A. pillows placed at the head
A. Quasi Experimental B. padded tongue blades
B. Correlational Study C. padded restraints
C. Descriptive study D. padded side rails
D. Developmental study
75.The nurse formulates a nursing diagnosis for the patient.
68.In gathering data for the study, ethical guidelines on basic Which of the following nursing diagnoses is NOT appropriate
human rights will be observed? Which of the ethical principles for this patient?
is applicable?
A. Impaired Electrolyte, Sodium related vomiting.
1. Justice B. imbalanced Nutrition, more that body requirements,
2. Privacy and dignity related to excess intake of foods rich in sodium.
3. Respect C. Risk for injury, bleeding, related to the interference with
4. Confidentiality blood coagulation secondary to sodium excess.
D. impaired skin integrity, related to peripheral edema
A. 1, 2, 3, 4 secondary to sodium and water excess.
B. 2, 3, 4 You are a nurse manager of a tertiary hospital. One of your
C. 1, 2, 3 responsibilities is to keep a
D. 2, 3 record of all patients admitted in the hospital.
69.Nurse Mark formulates an assumption for the study. 76.You are aware of the importance of keeping hospital
Which of the following is MOST acceptable? records. Which of the following statements is NOT rue about
hospital records? Hospital records _____________.
A. Liver cancer cases have decreased in 2017 due to
intensive public awareness A. provide data on health information system
B. More Filipinos regardless of gender are diagnosed with B. are a key source of data for medical research on
colorectal cancer. statistical reports.
C. Male Filipinos are prone to colorectal cancer than liver C. provide personal information about the physicians and
cancer. nurses assigned to care for the patients.
D. The leading cause of colorectal cancer among Filipinos D. provide evidence of a hospital’s accountability
is high consumption of alcohol.
77.You are oriented on the hospital policy that when a patient
70. Which of the following is the MOST appropriate is readmitted, the patient’s file maybe retrieved from the
recommendation Nurse Mark should propose? hospital records department. From which file may be a
readmitted patient’s record be retrieved?
A. Warning signs of the effects of alcohol should be printed
on bottles and cans of alcoholic beverages. A. physician’s ledger
B. Health professionals should educate the public on the B. master patient index file.
risk factors of CRC. C. civil service file
C. DOH only should intensify its campaign on colorectal D. hospital library records
cancer awareness.
78.You orient your staff on the common system used in 85.Nurse Gina is a potential team leader of the health team.
recording nursing interventions. The system used is a nursing Which of the following skills should she develop?
index card or Kardex. What information is NOT included in the
Kardex? ( Nurses notes A. Collaborative skills
B. Management
A. Drug regimen of the patient. C. Supervisory skills
B. Allergies if any of the patient. D. Patient advocacy
C. Progress notes of the physician.( Med order sheet)
D. Dietary requirements of the patient. Nurse Rolly, a triage nurse admits clients in the Emergency
Department (ED) of X
79.A patient’s record contains information of the medications hospital. The following are situations in the ED Nurse Rolly
and treatments administered, and observations of the encounters.
patient’s condition. Which of the following data MUST be filled
up in the patient’s chart when he/she is discharged from the 86.Four victims are brought to the ED after a motor vehicle
hospital? crash. Who among the following victims require the HIGHEST
priority for the treatment?
A. Religion
B. Nursing Diagnosis A. 21 year-old male with fracture of the face jaw.
C. Final medical diagnosis B. 20 year-old female with misaligned right leg.
D. Educational attainment C. 35 year-old male complaining of abdominal pain.
D. 62 year-old female with palpitation and chest pain.
80.You are familiar with the ethical aspects of patients and
hospital records. Which of the following statements is NOT 87.Four victims of a car crash are brought to the ED. Nurse
true? Rolly assesses the victims. Select who among the following
has the HIGHEST priority for treatment.
A. Health records are the property of the locality where the
patient is treated. A. Absence of peripheral pulses
B. Hospital records maybe released without the patient’s B. A suckling chest wound
consent when required in investigation for serious criminal C. Severe bleeding of facial and head lacerations
offense. D. An open femur fracture with profuse bleeding
C. Confidential records must be protected against loss,
damage, unauthorized access, modification and disclosure. 88.Nurse Rolly performs primary assessment on one of the
D. Patients have the right to confidential treatment of trauma victims, and determines that the client has a patent
information they provide to health professionals. airway. The NEXT assessment by Nurse Rolly should be to
___________.
Record: hospi
Info: patient A. palpate for the presence of peripheral pulses
B. check the level of consciousness
Ms. Gina is a staff nurse in a medical unit of x hospital. She C. examine the client for any external bleeding
collaborate with other D. observe/assess client’s breathing or respiratory effort
members of the health team to provide safe and quality
patient care. 89.A 45 year-old male client was brought in the ED with head
and neck trauma sustained in a motorcycle accident. The
81.Which of the following statements BEST explains the role FIRST action of Nurse Rolly is to ___________.
of the nurse in collaborating with others to plan for the
patient’s care? The nurse _____________. A. suction of the mouth and oropharynx
B. obtain venous access
A. collaborates with colleagues and the patient’s family to C. immobilize the cervical spine
provide combined expertise in planning care. D. administer supplemental oxygen
B. works independently to plan and deliver care and does
not depend on other staff for assistance. 90.Jerome, 65 years old who works as a carpenter fell from a
C. consults the physician for direction in establishing goals ladder while fixing the roof of a neighbor. He was brought to
for clients. the ED by family members. He is unconscious. Nurse Rolly
D. depends on the latest literature to complete an excellent does a primary assessment on client Jerome which is to :
plan of care.
A. Ask the family about Jerome’s medical condition
82.Nurse Gina is aware that collaborative interventions are B. Assess the vital signs
therapies that require the following: Which of the collaborative C. Attach a cardiac ECG monitor
interventions is the MOST therapeutic? D. Obtain a Glasgow Coma Scale Score
A. Nurse and patient intervention A 30 year-old female is admitted for fever, fatigue,
B. Multiple health care professionals lymphadenopathy, thrush, diarrhea
C. Physician and nurse intervention and muscle and joint pains. She also has a rash in her torso
D. Patient and physician intervention and arms.
83.To initiate an intervention in collaboration with the health 91.The nurse assesses the client. What question should she
team, Nurse Gina must be competent in which of the ask to determine the client’s possible exposure to HIV?
following areas?
A. “Do you use public toilet seats?”
A. Leadership, autonomy, and skills B. Did you shake hands with a person infected with HIV?”
B. Experience, advanced education, and skills C. “Did you receive blood transfusion recently?”
C. Knowledge, function, and specific skills D. “Do you practice safe sex?”
D. Leadership, finances, and skills
92.The nurse writes a care plan for the client. Included in the
84.Nurse Gina is aware that there are nursing activities that care plan is to provide health teachings. Before the nurse
may be delegated to other health care team members. Which performs any teaching, what should the nurse do FIRST?
principle should guide the nurse in delegating tasks?
A. Evaluate the client’s existing level of knowledge about
A. Delegation occurs only upon a physician’s order. HIV infection.
B. The delegated personnel is accountable for the care. B. Assess the client’s immediate clinical status.
C. Delegation may reduce the patient’s cost of care. C. Assess the emotional status of the client.
D. The nurse has the primary responsibility for the quality D. Focus on potential problems the client may encounter
of patient care. during the illness.
93.To determine whether the client is infected with HIV, the C. Lower back pain, increased blood pressure, decreased
physician writes an order for HIV antibody testing. What test RBC count, increased WBC count.
would confirm a positive ELISA test? D. Severe lower back pain, decreased blood pressure,
decreased RBC count, increased WBC count.
A. CD4 cell count
B. Easter Blot test Lower back pain: kidney problem
C. HIV antigen test
D. Western Blot test
94.The client is being treated for thrush. The patient asks if NP5
there are any side effects of the medication she is receiving https://docs.google.com/forms/d/e/1FAIpQLSdeGH-pegpJ
for thrush. Which of the following should the nurse include in LpxJG8pJc-68_dFEu_AVKWTam49uPNFaB8RheQ/viewsc
her teaching? ore?pli=1&pli=1&viewscore=AE0zAgALIKst-N0ZU_hUS2V
xAiisD_u7avpoACEbBYgylmAxc6a7zEym9CJg5Cfk9B2C
A. “There are few side effects associated with the gRQ
medications to treat thrush”
B. “Hepatitis can develop as a side effect.”
C. “Nausea, vomiting, and diarrhea are common side You’re assigned as a staff nurse in the Stroke Unit of the
effects.” Medical Ward. One of your responsibilities is to provide health
teachings to the stroke patients and to the other members of
D. “Skin discoloration is a common side effect.” the family. The following questions refer to these statements.
95.The client complains of increasing pain in her feet and 1. You are teaching a patient about taking prophylactic
legs. The nurse realizes that the client is demonstrating a/an warfarin sodium (Coumadin).( Vitamin K)Which of these
_________: statement indicate that the patient understands how to take
the drug. SELECT all that apply:
A. nervous system manifestation of the disease
B. reaction to a medication 1) “The drug’s action peak in two hours”
C. opportunistic infection 2) “Maximum dosage is not achieved until 3 to 4 days after
D. secondary cancer starting the medication”
3) “Effects of the drug continue for 4 to 5 days after
An adult male is wheeled in the Emergency Department discontinuing the medication
with complaints of nausea and 4) “Protamine Sulfate is the antidote for Warfarin”
vomiting, abdominal pain and lower back pain. The 5) “I should have my blood levels tested periodically”
physician writes a medical diagnosis of abdominal
aortic aneurysm (AAA) A. 2, 4, 5
B. 2, 3, 5
96.The nurse assesses the patient with AAA. Which of the C. 1, 4, 5
assessment findings is related to the aneurysm? D. 1, 3, 4
1. Pulsatile abdominal mass 2. Maintaining oral hygiene for a stroke patient is important in
2. Hyperactive bowel sounds his care. You are teaching a family member how to give oral
3. Systolic bruit over the area of the mass care to the stroke patient. Which of the following nursing
4. Subjective sensation of “heart beating” in the abdomen measure is NOT APPROPRIATE when giving oral care to such
patient? ( Risk for aspiration)( Side lying)
A. 1, 3, 4
B. 1, 2, 3, 4 A. Cleaning the patient’s mouth and teeth with a
C. 2, 3, 4 soft-bristled toothbrush
D. 1, 2, 3 B. Keeping portable suctioning equipment at the bedside
C. Opening the patient’s mouth with a padded tongue
97.The nurse auscultates the abdominal area of the patient depressor
with AAA. Which of the following sounds can be DISTINCTLY D. Placing the patient on his back with a small pillow
heard over the area? under the head
A. Dullness 3. Your patient has paraplegia. ( Lower extremities paralysize,
B. Bruit at risk for pressure ulcers) In giving instruction to the watcher
C. Friction rubs when changing the patient’s position in bed, which of the
D. Crackles following is NOT APPROPRIATE?
98.The nurse recalls specific anatomic sites for aneurysm. A. Lifting the patient when moving him up in bed
The most common sites are the aortic arch, thoracic aorta B. Rolling the patient unto the side
and abdominal aorta. Which of the following areas is an AAA C. Using a trapeze to help the patient lift off the bed
most commonly located? D. Sliding the patient to move up in bed( at risk for
injury)
A. Proximal to the renal arteries
B. Distal to the iliac arteries 4. A patient is experiencing mood swings after stroke. Often
C. Distal to the renal arteries times she has episodes of crying that are distressed to the
D. Adjacent to the aortic arch family members. What is the BEST TECHNIQUE that you will
advise the family members when the patient is having crying
99.The patient complains of severe lower back pain. Which of spells?
the following is the PRIORITY action by the nurse?
A. Ignore the patient during this time
A. take the vital signs and document results B. Sit quietly with the patient until the crying episode is
B. administer pain medication as prescribed over
C. notify the physician C. Tell the patient that the behavior is unacceptable
D. observe for signs of abdominal distention. D. Attempt to divert the patient’s attention
5. To prevent joint deformities of the arm and hands of a
100. The nurse is aware that rupture of the aneurysm is a hemiplegic patients what are the APPROPRIATE position will
life-threatening emergency. Which of the following groups of you include in your health teachings of the family member?
symptoms indicates a ruptured AAA? SELECT all that apply.
A. Intermittent lower back pain, decreased blood pressure, 1) Placing a pillow in the axilla so the arm is way from the
decreased RBC count, increased WBC count. body ( should not be away)
B. Severe back pain, decreased blood pressure, decreased
RBC count, increased WBC count.
2) Inserting a pillow under the slightly flexed arm so the hand
is higher than the elbow 12. Which of the following sampling techniques is the most
3) Immobilizing the extremity with a sling ( can cause appropriate to answer a research question on the
contractions) effectiveness of active exercise on the mobility of post stroke
4) Placing a hand cone so the finger are barely flexed patients?
5) Keeping the arm at the sides using a pillow
A. 2, 3, 4 A. convenience sampling
B. 1, 3, 5 B. purposive sampling
C. 1, 2, 4 C. quota sampling
D. 2, 4, 5 D. random sampling: fishball method
To avoid footdrop wear rubber shoes 13. You want to develop a tool for measuring the level of
You are a community health nurse assisting a family in the consciousness of stroke patients. This is what type of
care of their daughter suffering from research?
the past traumatic stress disorder.
A. Historical study: at the past
6. While caring for this client, the family notices that loud B. Case study: one
noises cause a serious anxiety response. Which of the C. Comparative study
following explanations by you would help the family D. Methodological study
understand the client’s response?
14. You want to examine the difference in the eating
A. After a trauma, the client cannot respond to stimuli in an behaviors between two groups of high school students.
appropriate manner Which of the following research designs would you use to
B. Clients often experience extreme fear about normal answer the research question?
environmental stimuli
C. Environmental triggers can cause the client to react A. Case study
emotionally B. Correlation study
D. The response indicates that another emotion problem C. Comparative design
needs investigation D. Survey study
7.Which of the following instructions should you include 15. In a study on the relationship between personality
about relationships for this client with post-traumatic stress make-up and obesity, a Pearson r=.8 would mean: (0.1-0.3
disorder? low, 0.3-0.5 moderate, o.51- 1 high)
A. Assess the client’s discomfort when talking about A. a high, negative correlation
feelings to family member. B. a moderate, positive correlation
B. Explain that avoiding emotional attachment protects C. a high, positive correlation
against anxiety. D. a low, positive correlation
C. Warn the client that she will have a tendency to be
overdependent in relationships 16. Which of the following is the precipitating factor for Ella’s
D. Encourage the client to resume former roles as soon as feeling of wanting to die? ( Provoking)
possible
A. The death of Lory
8. Which of the following nursing intervention would best help B. Her age
this client and her family handle interpersonal conflict at C. Her authoritative mother
home? Have the family: D. Her fear of death
A. Discuss how to change dysfunctional family patterns 17. Ella’s feeling of joining Lory in heaven is manifestation of:
B. Teach the client to identify defensive behavior.
C. Agree not to tell the client what to do about problem A. Suicidal intent( pananakot)( plan)
D. Arrange for the client to participate in social activities. B. Suicidal threat( failed)
C. Paranoia
9. Which of the following nursing actions would be more D. Suicidal ideation
appropriate when speaking with this client about the trauma
she experienced? 18. Ella state “I will hang myself”. This is a manifestation of:
A. Listen attentively A. Denial
B. Request the client write what is being said B. Suicidal threat
C. Ask question to convey an interest in the details C. Suicidal intent
D. Obtain validation of what the client says from another D. Paranoia
party
19. Which of the following is the best predictor of adolescents
10. Which of the following nursing actions would you include attempting suicide?
in the plan of care for this client who keeps saying that the
trauma she experienced was just a case of bad luck? A. Depressed mood
B. Joyful mood
A. Assist the client in defining the experience as a trauma C. Feeling of euphoria
B. Encourage the client to verbalize the experience D. feeling of hopelessness
C. Work with the client to accept positive and negative
feelings 20. Ella states “I wish I were dead. I cannot stand anymore not
having Lory around.” Your most appropriate nursing action
One of the competencies expected of you as a beginning would be:
professional nurse is to have a positive
attitude toward research. A. Do nothing because Ella will not do it
B. Refer Ella to your supervisor
11. You plan to study the effectiveness of films in teaching C. Stay with Ella
active exercises to post stroke patients. Which of the D. Explore Ella’s feeling
following research designs will give you the most confidence
to answer the research problem? You are a staff nurse in the Out-Patient Department. Mrs. D
brought her 9-year old daughter Bianca for consultation.
A. time- series design Bianca is complaining of sore throat, muscle tenderness,
B. non-equivalent control group design weakness of the arms and a general feeling of not being well.
C. one-group pre-test, post-test design The attending physician’s impression is Infectious
D. post-test only, control group design Polyneuritis. ( Guillain Barre syndrome)
21. Initially, what assessment data would be the BEST A. Normal carbohydrate, high protein, high potassium
IMPORTANT for you to make? B. Low carbohydrate, high protein, high potassium
C. Low carbohydrate, high protein, low potassium
A. Exposure to an infectious disease D. High carbohydrate, low protein, low potassium
B. Difficulty in swallowing
C. Difficulty in urinating 30. Jennylyn was on Ibuprofen which has been stopped when
D. Diet intake for the last 24 hours the steroid therapy was started. Which was the main reason
for stopping the Ibuprofen once the steroid therapy was
22. Bianca was admitted to the Pediatric ward. What would be started?
your PRIORITY nursing action when caring for her?
A. The two drugs taken together increase the risk of GI
A. Providing a diversional activity for Bianca bleeding
B. Evaluating Bianca’s bilateral muscle strength B. The Ibuprofen would interfere with the action of the
C. Making a game of range-of motion exercise steroid
D. Assessing Bianca’s ability to follow simple commands C. The Ibuprofen was not needed with the steroid
D. The two drugs together would cause severe liver
23. The pediatric ward nurse ask Bianca to cough every now damage
and then. She also assesses Bianca’s speech for decreased
volume and clarity, the underlying rationale for these After meal
assessments is to determine the ________.
You are a staff nurse in the Musculo-skeletal Unit of the
A. regression to an earlier developmental phase hospital. You are taking care of the
B. involvement of facial and cranial nerves patients with varied musc
C. increased intracranial pressure ulo-skeletal conditions.
D. inflammation of the larynx and epiglottis
31. which of the following inflammatory conditions commonly
24. Assessment findings revealed that Bianca has weak gag occurs in the shoulder?
and cough reflexes. During this acute phase of Bianca’s
illness, which of the following problems should receive the 1) Bursitis
HIGHEST PRIORITY? 2) Carpal Tunnel Syndrome
3) Tendonitis
A. Impaired swallowing related to neuromuscular 4) Dupuytrens
impairment. A. 1 and 2
B. Risk for infection due to an altered immune system B. 2 and 3
C. Ineffective breathing pattern to neuromuscular C. 3 and 4
impairment D. 1 and 3
D. Total urinary incontinence related to fluid losses
32. Which of the following is NOT considered a conservative
25. Bianca was placed on a medical ventilator. Which of the treatment for inflammatory conditions of the shoulder?
following nursing actions should the ward nurse give
emphasis to? A. Non-steroid Anti-inflammatory Drugs
B. Intermittent ice and heat application to the joint
A. Turning Bianca slowly and gently from side to side to C. Laser photo-therapy
prevent respiratory complication D. Resting of the extremity
B. Engaging Bianca in vigorous passive range-of-motion
C. Maintaining Bianca in a supine position to prevent 33. Impaired movement of the rotator cuff of the shoulder is
unnecessary nerve stimulation generally termed as Impingement Syndrome. Edema, which is
D. Transferring Bianca to a bedside chair three times a day an early manifestation of this syndrome, is due to ________.
to prevent posture hypotension
A. acute trauma of the shoulder
Jennylyn Abad, 40-years old woman, has a history of B. muscle spasm
Rheumatoid Arthritis ( autoimmune, systemic, bilateral) for C. hemorrhage of the structures involved
the last ten years. The following questions refer to this D. atrophy due to disuse
situation.
34. For a persistent shoulder pain and weakness, this usual
26. Which of the following is a common sign/symptom of treatment modality performed is _______.
rheumatoid arthritis?
A. Pulse-magnetic field
A. Presence of crepitus over joints: OA B. Laser photo-therapy
B. Symptoms aggravated by humidity C. Radio-frequency ablation
C. Deformity and displacement of proximal joints D. Arthroscopic Synovectomy
D. Presence of Heberden’s nodes: OA
35. One of the most common foot problems is Callus. This
27. which of the following would NOT be a common condition precedes the formation of a Callus _____.
laboratory findings in rheumatoid arthritis?
A. External pressure on the foot
A. Low hemoglobin B. Faulty foot mechanics
B. Positive RA factor C. Flexion deformity of the interphalangeal joint
C. Positive Lupus Erythematosus (LE) prep D. Penetrating nail plate in the surrounding skin of the foot
D. Increase white blood count
You are a staff nurse working in a hospital and encounter
28. Jennylyn has been taking steroids for the last five years to situations with ethico-moral implications.
control her arthritis. Which of the following is a common side
effects of steroid therapy? 36. Which of the following nursing actions will you do if the
STEROID: TRIPLE SALT SEX patient is unconscious and unable to make decisions? This
fact should be:
A. Hyponatremia
B. Hyperkalemia A. reported to the nurse supervisor
C. Increase blood glucose B. reported to the attending physician
D. Protein anabolism C. documented in the patient’s chart
D. reported to the police
29. Which of the following diet is best suited to decrease the
side effects of Jennylyn’s steroid therapy? Paternalism: surgery
1. Unable to consent
2. Unconscious A. Feeding the infant just before doing any procedures
3. Emergency care B. Giving the infant small, frequent feedings
4. Unable to reach next of kin C. Scheduling the feeding every six hours
D. Feeding the infant in a horizontal position
37. A patient came in from prison for emergency treatment. 44. The parent of a school-age child with ventriculo-peritoneal
Which of the following precautionary measures will you shunt were given discharge teaching. Which of the following
institute to ensure the safety of the hospital staff and the signs of a blocked shunt would the parents be able to identify
other patients? that the discharge teaching is successful?
A. Provide the same assessment and care to the client A. Irritability and increase difficulty with eating
B. Use restraints B. Elevated temperature and reddened incisional site
C. Lock the ER door against entry if security is in question C. Decreased urine output with stable intake
D. Never release the hand or ankle restraint D. Tensed fontanel and increased head circumference
38. In using restraints to patients, which of the following 45. After a six-year old child underwent placement of
guidelines will you follow? ventriculo-peritoneal shunt, he was put on I.V. medication. The
head nurse observed that many of the staff nurses are
A. Apply restraints because the patient needs it disconnecting the flush syringe first and then clamping the
B. Apply restraints according to the hospital policy intermittent infusion device. The staff nurse seem not to
C. Restraint is the most effective nursing intervention to understand the benefits of positive pressure techniques.
minimize aggressive behavior Which of the following strategies would be the MOST
D. Maintain respect for the client when applying restraints EFFECTIVE way to improve the staff nurses technique in
giving I.V. medication?
39. You have observed that a group of students are
implementing their research project on the effect of hot and A. Post an evidence-based article on the topic in the unit
cold application to febrile patients without informing the B. Create a poster presentation on the topic with a required
participants properly. What ethical principle was violated by post test
the students? C. Send a memorandum on the importance of clamping the
device first
A. Nonmaleficence D. Ask each nurse if they are aware that their practice is not
B. Autonomy current
C. Justice
D. Respect for person You are a staff nurse in the Stroke Unit of the Medical ward.
You have a stroke patient with a tracheostomy. In the unit’s
last meeting it was discussed that there has been increasing
40. A researcher should pay particular attention in protecting complaints on the nurse incompetence in caring for a patient
the rights of certain vulnerable groups. Which of the following with tracheostomy. The following questions related to this
is NOT considered vulnerable? situation.
A. Prisoners 46. In providing tracheostomy care which of the following is
B. Children the nurse’s PRIORITY nursing action? The nurse _____.
C. Mentally-challenged persons
D. Postpartum mother with normal delivery A. cut the dressing using sterile scissors
B. cleans the incision with iodine-based antiseptic
C. secure clean ties before removing soiled ones
D. used clean technique
You are a staff nurse in the Neurologic Unit of the Pediatric
Department. Several problems have been reported in relation Colostomy: clean
to ventriculo-peritoneal shunt for hydrocephalus patients. The Dressing: clean
initial investigation on the care of patient with this shunt GI tract: clean
revealed that majority of the staff nurses have little
experience on caring for such patients. It was decided to look 47. Which of the following nursing actions should the nurse
into their level of knowledge and skills to serve as baseline TEACH the patient’s care giver regarding tracheostomy care?
data for improving the nursing care of hydrocephalic patients
on ventriculo-peritoneal shunt. The following questions refer A. Remove the tracheostomy tube if obstruction occurs
to this situation. B. Leave the tracheostomy ties in place until the physician
change them
41. a four-year old hydrocephalic preschooler is scheduled to C. Remove and clean the inner cannula daily
have a ventriculo-peritoneal shunt at the right side of the D. Never put a covering over the tracheostomy stoma
head. Which of the following positions should the child be
placed immediately after surgery? ( Opposite side)
48. After a tracheostomy procedure, the nurse documents
A. Supine, with the head of the bed flat important observations made during the procedures such as
B. On the left side with the head of the flat _______.
C. Prone, with the head of the bed elevated
D. On the right side, with the foot of the bed elevated A. response of the patient after the procedure
B. vital signs of the patient during the procedure
WOF inc ICP C. amount, color, and consistency of sputum and
appearance of the incision
42. What nursing action should the nurse do when providing D. behavior of the patient during the procedure
post-operative nursing care to a child after insertion of a
ventriculo-peritoneal shunt? 49. What is the purpose of a tracheostomy? To _____.
A. Monitoring for the increased temperature A. establish an airway
B. Check the urine for glucose and protein B. establish a pathway for nutrition
C. Administer narcotics for pain control C. monitor respiratory functions
D. Test cerebrospinal fluid leakage for protein D. maintain the patency of the airway
43. An infant who has hydrocephalus is irritable, lethargic, and 50. which of the following can be a MAJOR PROBLEM for a
difficult to feed before the ventriculo-peritoneal shunt is patient with tracheostomy?
placed. To maintain the infant’s nutritional status, which of the
following nursing action is the MOST APPROPRIATE? A. Breathing
B. Ambulating 59. Which of the following is the GOAL of nursing intervention
C. Swallowing for a patient who is hallucination.
D. Singing
A. To help the patient to have an increased awareness that
You are staff nurse in the Psychiatric Unit of a private in symptoms she is experiencing is not real
hospital. Lorena, a 20-year old stage actress was admitted B. To help the patient to identify what triggers her
with the chief complaints of getting angry easily and inability hallucination
to tolerate being alone. She claimed that she has also the C. To encourage the patient to verbalized her fear, anxiety,
tendency to manipulate people and feels unhappy most of the and anger
time. She was diagnosed to be suffering from Borderline D. To help the patient to go back to her real world
Personality Disorder (BPD).
60. What communication technique would be Most effective
51. You have observed that Lorena is manifesting “spitting”. to achieve the nursing goal for patient with hallucination?
This characteristic of BPD is BEST defined as ______.
A. viewing people and objects as parts, either good or bad A. Listening
B. having two personalities : dissosiative B. Facilitative communication
C. talking about other people behind their back C. One-on-one discussion
D. literally spitting in other people’s face D. Social interaction
52. When assessing a patient with BPD which of the following Charting is part of the professional nurse’s responsibility
information would you focus on? related to record management. The following questions refer
A. Ability to get people on his/her side to this statement.
B. Disruption in some aspect of his/her life
C. desire for intimate relationship 61. One of the characteristics of charting is brevity. Which of
D. Increase acceptance from other people the following example of charting shows this characteristics?
53. Lorena revealed she has a tendency to manipulate others. A. Nurse brought the patient to OR via stretcher at 10:15
What would be the MOST APPROPRIATE short-term goal for B. Patient left for surgery via stretcher at 10:15 am
her? For Lorena to _____. C. To surgery via stretcher at 10:15
D. patient brought by the nurse to OR via stretcher at 10:15
A. have an intimate relationship accompanied by the “bantay”
B. acknowledge her own behavior
C, stop arguing with other people 62.Which of the following is NOT a requirement for a late
D. express her feelings verbally entry in charting?
54. Patient with BPD manifest transient psychotic symptoms. A. nurse’s initials above the time
What is the drug of choice to treat these symptoms? B. may be made if something has been forgotten
C. circle it and write “late entry”
A. Mood stabilizer D. reason for the late entry
B. Benzodiazepines
C. Lithium
D. Antipsychotics 63. There is a black space after your last entry in your
charting. You will ______.
55. Lorena has a history of alcohol abuse so she was started A. draw a double line after your last entry
on Antabuse. Which of the following is a COMMON side effect B. sign your full name after your last entry
of this drug? C. draw a horizontal line through the center of an empty
line
A. Hypertension D. draw a perpendicular line across the empty space
B. Bradycardia
C. Depression: low mood level 64. Which of the following should the nurse’s notes focus on?
D. Elation A. Immediate past and the present
B. The present only
Angela, a 40-year old, single mom of two preschool children, C. The future
aged two (2) and five (5) years old respectively was admitted D. The recent past
to the Psychiatric Unit for attempting to kill her two children.
65. Which of the following sample charting would show the
56. Angela says “My dead mother ordered me to kill my two characteristic of ACCURACY?
children before the devil could get them”. What behavior is
being manifested by Angela? A. intake from 700-1000 ml: 80 ml of coffee; 240 ml of
orange juice; 500ml of water
A. Auditory hallucination B. Patient on forced fluid but refused to take it most of the
B. Visual hallucination time
C. Tactile stimulation C. Patient on forced fluids observed
D. Olfactory hallucination D. Given fluid at frequent intervals but takes only a few sips
57. You ask Angela to touch a 25-centavo coin and a During summer season incidence of heat stroke rises. As a
one-peso coin. You have observed that Angels was having community health nurse you should disseminate information
difficulty differentiating one coin from the other. This is on this as part of your role as a health educator.
manifestation of _____.
66. A patient came in with sign of heat stroke. Which of the
A. astereognosis following are signs/symptoms of profound CNS dysfunction?
B. kinesthetic
C. agraphestesia A. Elevated temperature
D. cenesthetic B. Hot, dry skin
C. Tachypnea, hypotension, tachycardia
58. You have observed that Angela is manifesting persistent D. Confusion, delirium, bizarre behavior
hallucination. What is the MOST EFFECTIVE therapy in
treating patient with a condition like that of Angela’s? 67. Which of the following is the primary goal in the care of
patient with a heat stroke?
A. Cognitive-behavior therapy
B. Crisis intervention A. To maintain cardiac functions
C. Psycho-pharmacologic treatment B. To reduce immediately the high temperature
D. Anti-psychotic drug C. To restore normal body temperature
D. To prevent further complication
C. “I’m so glad Valiums won’t affect my driving skills”
D. “I’m so glad I will only have to take this until I learn to be
68. Which of the following is the best advice you would give to less anxious”
an athlete to prevent heat stroke?
As a professional nurse, one of your roles is an advocate for
A. To monitor fluid losses and weight lost during workout the patient’s rights. The following
activities and to replace fluids. questions refer to this statement.
B. To avoid immediate exposure to high temperature
C. To maintain adequate fluid intake 76. You were invited by a women’s organization to be the
D. To avoid planning outdoor activities between 10 a.m. and resource speaker on “Violence against women and children”.
2 p.m. Women and their children are protected against violence
under Republic Act No.__.
69. to prevent heat stroke, the following pieces of advice are A. 9173
given to the community, EXCEPT: B.9211
C.9262
A. Avoid direct exposure to the sun especially late in the D.9160
morning
B. Always drink plenty of water 77. One of the violence acts is sexual violence. Which of the
C. Use a hat or an umbrella when going out of the house following does NOT constitute sexual violence?
D. Just stay home and relax
A. Acts of lasciviousness
B. Rape
70. Which of the following would your best advice to the C. Treating a child as a sex object
community people during summer time to prevent D. Economic abuse
dehydration?
A. Use a hat or an umbrella when going out of the house 78. What would be the nurse’s MAIN concern when a case of
B. Just stay home and relax abuse, maltreatment, or neglect is suspected in a patient?
C. drink plenty of fluids
D. wear light-colored clothing A. Reporting the incident to proper authorities
B. Safety and welfare of the patient
Lizbeth, a 30-year old registered nurse with two children, C. Referring the patient to a mental health worker
legally separated from her husband was admitted to the D. Just keep quiet about the matter
psychiatric unit three weeks ago.
79. What is the MOST common clinical manifestation of child
71. You are the nurse attending to Lizbeth. You have observed neglect?
that she has a habit of washing her hands repeatedly for a
long period of time. This is a manifestation of what kind of A. Missed appointment with health care provider
behavior? B. Fracture
C. Unexplained bruises
A. Negative D. Malnutrition and dehydration
B. Hyperactive
C. Ritualistic: to relieve anxiety
D. Nonconformist 80. What should be the FOCUS of care for patients who are
suspected to be abused, maltreated, or neglected?
72. Lizbeth engages in this behavior to _____.
A. Immediate treatment of any injury
A. protect herself from undesirable people B. Consequences of the act
B. relieve her anxiety C. Prevention of further injury
C. occupy herself with purposeful activity D. Referral to legal authorities
D. call the attention of other
You are a staff nurse at the Ear Unit of the Medical Ward of a
73. A new nurse introduce herself to Lizbeth and asks her government hospital. Clarita, a 25-year old stage actress was
name. Lizbeth responds “I am an obsessive compulsive admitted with the chief complaint of on and off tinnitus at the
neurotic. I have had psychoanalysis for ten years. What do right ear. The audiogram confirms conductive hearing loss or
you think can you do for me?” Your BEST response would be mixed loss especially in the low frequencies. Clarita was
_____. diagnosed with Otosclerosis.
A. “Can we talk about that Lizbeth?” 81. Otosclerosis is a common cause of what condition?
B.” I need to know you better Lizbeth”
C.” You seem to feel hopeless” A. Premature labor
D.” who was your psychoanalyst?” B. Meningitis Auditory nerve
C. Sensori-neural hearing loss: inner ear
74. Lizbeth tells you “That the new nurse makes me angry. D. Conductive hearing loss
Like you, she does not understand what my real problem is”.
Your BEST reply would be _____. 82. The audiogram also confirms a sensori-neural hearing
loss. What ear structure is damaged when this condition is
A. “You seems to be upset. I will come back later” present?
B. “You have the right to be upset when people don’t seem
to understand” A. Outer and/or middle ear
C. “That’s a common feeling. I understand. Let’s talk about B. Tympanic membrane: middle ear
it” C. Cochlear nerve
D. “I know what your problem is. You are an D. Stapes: middle
obsessive-compulsive personality”
83. Clarita was given Sodium Flouride. What is the rationale
75. Diazepam (Valium) was prescribe for Lizbeth. You gave for giving this to her?
her instructions on effects of the drug. What statement would
indicate that Lizbeth needs further health teaching about the A. To mature the spongy bone growth
medication? B. To remove the diseased stapes
C. To prevent further complications
A. “I’m so glad I can still eat chocolate while I’m taking this.” D. To restore hearing
B. “I’m so glad no blood tests are necessary while I’m
taking this.”
84. A surgical procedure was recommended for Clarita. Which
of the following surgical procedures is useful in correcting her 91. What are the effects of drinking coffee two to three cups a
condition? day on the central nervous system?
A. Myringotomy A. Feeling motivated and energized, less fatigue
B. Ossicular reconstruction B. Feeling motivated and energized, headache
C. Cochlear transplant C. Feeling motivated and drowsy, headache
D. to restore hearing D. Headache, less fatigue
85. After her surgery, Clarita was placed on her left side with 92. Which of the following reinforces a person to continue
the head of the bed elevated. What is the rationale for placing drinking coffee?
her in this position?
A. Feeling of unpleasant symptoms when drinking is
A. To minimize the pressure in the middle ear abruptly stopped.
B. To prevent complication of bleeding B. Drinking coffee at Starbucks is the “in-thing
C. To ensure patient safety C. Feeling of well-being
D. To prevent vomiting D. The price of coffee is getting lower
Ms. Ligaya Co is a chief nurse in a secondary level hospital 93. Ana was given Luminal barbiturates 60 mg./day p.o. this
with nursing staff consisting of registered staff nurses, drugs is a/an _____/
nursing assistants and aides.
a. intermediate-acting sedative
86. Ms. Co stresses the importance of promoting ‘esprit d b. short-acting sedative
corps’ among the nursing staff. Which of the following c. hypnotic drug
statement indicates that they understand the meaning of the d. long-acting sedative
term?
94. While doing your assessment Ana complained of nausea.
A. “In order that we achieve the goals of the institute, we Which of the following would be you PRIORITY nursing
must follow the directives coming from above.” action?
B. “We will ensure that all resources we need are available
when needed” A. Refer Ana to the attending physician immediately
C. “Let’s work together in harmony; we need to be B. Let Ana put her head in-between her legs
supportive of one another.” C. Administer an antiemetic drug
D. “We need to show our competence to the higher D. Offering Ana some ice chips
management level.”
87. She assert the importance of promoting a positive 95. you have observed Ana to be pacing in the room, restless,
organizational culture among the nursing staff. Which of the and stutters when your speaking to her. You would assess
following behaviors indicate that this has been attained? Ana as manifesting which of the following global anxiety
response?
A. Obedient and uncomplaining
B. Powerful and oppositional A. Cognitive
C. Competitive and perfectionist B. Behavioral
D. Caring and nurturing with one another C. Motor
D. Biological
88. Ms. Co is a visionary strategist, and desires to be a You are a staff nurse in a psychiatric unit, use of therapeutic
committed leader. Which of the following types of leadership communication is one of your nursing responsibilities.
do these behaviors reflect?
96. Verbal communication is the use of words when talking to
A. Servant your patient.
B. Transactional
C. Connective A. understandable phase
D. Transformational B. content
C. the circumstances
D. understandable sentences
89. She knows that as a leader, she has to strategize in order
to create followership in response to authority. This capacity 97. Context of a verbal communication is the:
to act or the strength to accomplish a goal is referred to as
_____. A. use of understandable sentences
B. use of literal words
A. power C. use of clear sentence
B. accountability D. environmental where communication occurs
C. responsibility
D. authority 98. Non-verbal communication is the behavior that
accompanies verbal communication. Which of the following
90. Ms. Co gathered the nursing staff and other health care is NOT an indicator of this?
worker in the hospital who support the idea of having a
meeting to discuss problems that affect the delivery of care A. Eye Contact
and to agree to speak with one voice. This power mechanism B. Word representing an object
is called ______. C. Grunts and groans
D. Bochy language
A. coalition
B. upward appeal 99. Which of the following give meaning and context to the
C. rationality message?
D. assertiveness
A. Process
Ana, a 28-year old supervisor in a business processing office, B. Phrases and sentences
consult at the Out-patient department of the hospital with the C. Context
chief complaints of headache, drowsiness, nausea, and D. Thoughts and feelings
vomiting. She revealed that one week before consult she
stopped drinking coffee which she used to take three to five
times a day. She was diagnosed with Caffeine 100. Which of the following situation is an example of
Discontinuation Syndrome (CDS). incongruent message?
7. Nurse Kang Tae-moo also identifies that dengue cases
A. When the nurse means what she says peak during the rainy season. This type of pattern of
B. When the words and behavior of the nurse agree occurrence over time is known as:
C. When what the nurse says and does do not agree
D. When content and process agree A. Point source
B. Continuous source
C. Seasonal trend
D. Cyclical trend
Other:
RECALLS 5 8. With regards to the steps in outbreak investigation, which
should be followed after preparing for field work?
NP1
A. Establish the existence of an outbreak
B. Define and Identify cases
C. Verify Diagnosis
https://docs.google.com/forms/d/e/1FAIpQLScRA1_-a3otk D. Developing Hypotheses
JkZ2Qx7VRhQY-NfsxjpsWorz65_xvaAjjMnMA/viewscore? 9. Which of the following correctly defines the term "sporadic"
viewscore=AE0zAgBe2G8G6kC3HyHiwO57X_YtetJF-xElL as it relates to the occurrence of rabies?
zUMXmdeMfWciXlp-RWHJ1jOYOjmhNDr9-M
A. A simultaneous occurrence of epidemic of the same
Situation: Juliana, a registered nurse in the Philippines with 10 disease in several countries.
years of experience, has been promoted to a supervisory role B. An intermittent occurrence of a few isolated and
in a large hospital. To excel in her new position, she reviews unrelated cases in a given locality.
Republic Act 9173, the Philippine Nursing Act of 2002, C. An intermittent occurence of a few isolated and
focusing on qualifications, prohibitions, penalties, and unrelated cases in a given locality.
procedures in nursing. D. A continuous occurrence throughout a period of time of
the usual number of cases in a given locality.
1. According to Republic Act 9173, after how many years can 10. What is the role of an epidemiology nurse during an
the Board re-issue a revoked certificate of outbreak? Select all that apply.
registration/professional license, provided that the cause for 1. Monitor local health personnel conducting disease
revocation has been resolved surveillance
A. 1 year 2. Implement public health surveillance
B. 2 years 3. Assist in the conduct of training course in epidemiology
C. 3 years 4. Responsible for inventory and maintenance of
D. 4 years epidemiology and surveillance unit (ESU) equipment
2. According to Republic Act 9173, what penalty may be
imposed upon individuals found guilty of violating A. 1, 3, 4
prohibitions in the practice of nursing? B. 2, 3, 4
C. 1, 2, 3
A. Fine of not less than P20,000.00 and imprisonment of D. All of the above
not less than 6 months Situation: Fatmah is a community health nurse working in a
B. Fine of not less than P50,000.00 and imprisonment of rural health unit. She is responsible for implementing various
not less than 1 year health programs and educating the community on how to
C. Fine of not less than P100,000.00 and imprisonment of prevent health problems. During her daily activities, she
not less than 2 years encounters different situations that require applying the
D. Fine of not less than P200,000.00 and imprisonment of levels of prevention.
not less than 3 years 11. Which of the following activities is an example of primary
3. According to Republic Act 9173, what is the passing grade prevention in Nurse Fatmah’s work?
for the licensure examination for nurses in the Philippines?
A. Rehabilitation for stroke patients
A. 70% B. Blood pressure screenings
B. 75% C. Immunization programs
C. 80% D. Physical therapy sessions
D. 85% 12. Nurse Fatmah is informed that a child in the community
4. What is the minimum required number of units in has been diagnosed with measles. To prevent the spread of
management and administration courses at the graduate level the infection to other children, Nurse Fatmah arranges for the
for a nurse to qualify for a supervisory position as per RA child to be isolated at home until they are no longer
9173? contagious. Isolation of a child with measles belongs to what
level of prevention?
A. 6 units
B. 9 units A. Primary
C. 12 units B. Secondary
D. 15 units C. Intermediate
5. According to RA 9173, what is the minimum number of D. Tertiary
years of experience required in general nursing service 13. At a local health clinic, Fatmah provides screening
administration for a nurse to qualify for a supervisory or services for sexually transmitted infections (STIs) to sexually
managerial position? active adults. Which activity is an example of secondary
prevention?
A. 1 year
B. 2 years A. Providing dietary counseling to all community members
C. 3 years B. Conducting annual mammograms
D. 4 years C. Teaching stress management techniques
D. Screening services for STIs
14. In a rural community, Fatmah, together with her
colleagues, conducts Operation Timbang, a program aimed at
6. During his assessment, Nurse Kang Tae-moo notes a assessing the nutritional status of children under five years
sudden increase in the number of dengue cases in Barangay old through regular weighing and monitoring. Operation
Malinis. This type of disease occurrence is best described as: Timbang is _____ prevention.
A. Endemic A. Primary
B. Epidemic B. Secondary
C. Sporadic C. Intermediate
D. Pandemic D. Tertiary
15. Nurse Fatmah is providing care for a patient who has 23. Which law in the Philippines mandates the creation of a
recently undergone surgery for advanced-stage lung cancer. comprehensive mental health program and aims to improve
She is focused on pain management, symptom control, and mental health services nationwide?
improving the patient's quality of life. Tertiary prevention is *
needed at which stage of the natural history of disease? 1 point
A. RA 11036
A. Prodromal B. RA 9275
B. Pre-pathogenesis C. RA 11223
C. Terminal D. RA 10173
D. Pathogenesis 24. Which legislation in the Philippines provides for the
16. Lalane, a public health nurse, organizes a support group expansion of health care coverage and the integration of
for residents diagnosed with chronic conditions like diabetes health services to ensure access to quality care for all
and hypertension to promote self-management and enhance Filipinos?
their quality of life. What constitutes tertiary prevention in *
Lalane’s community health initiatives? 1 point
A. RA 11036
A. Health education on lifestyle changes B. RA 7836
B. Routine screenings for chronic diseases C. RA 11223
C. Support groups for chronic illness patients D. RA 8551
D. Immunization drives for children 25. Which law in the Philippines is responsible for transferring
17. In a rural community, Nurse Lalane conducts a community the control and responsibility of delivering basic health
workshop on promoting heart health to residents of all ages. services to local government units?
Which of the following represents a primary prevention *
strategy aimed at reducing the incidence of cardiovascular 1 point
diseases? A. RA 7875
A. Angioplasty B. RA 7160
B. Blood pressure monitoring C. RA 8423
C. Health education on diet and exercise D. RA 7164
D. Cardiac rehabilitation Situation: Nurse Taeyeon is working in a community health
18. At a community health fair, Nurse Lalane offers diabetes clinic where the staff is facing a challenging ethical dilemma.
screenings to residents to identify those at risk of developing The clinic has a limited budget for various essential
the disease. This activity is an example of: resources and services. Taeyeon is involved in the
* discussion on how to make these decisions ethically,
1 point ensuring that all patients' needs are considered fairly and
A. Primary prevention compassionately.
B. Secondary prevention 26. Nurse Taeyeon is faced with a situation where a patient,
C. Tertiary prevention Mr. Kim, who has a terminal illness, wishes to discontinue all
D. Quaternary prevention treatments despite his family’s desire to continue treatment.
19. Nurse Lalane provides a rehabilitation program for stroke What principle should guide Taeyeon in respecting Mr. Kim’s
survivors to help them regain their physical and cognitive decision?
abilities. Offering rehabilitation programs for stroke survivors *
is an example of which level of prevention? 1 point
* A. Autonomy
1 point B. Beneficence
A. Primary prevention C. Justice
B. Secondary prevention D. Nonmaleficence
C. Tertiary prevention 27. Nurse Taeyeon discovers that a mistake was made in the
D. Quaternary prevention administration of medication, which did not affect patient
20. In a barangay health center, nurses conduct a health outcomes. The mistake was not reported, and Taeyeon is
promotion campaign to encourage regular handwashing with considering whether to disclose it to maintain transparency
soap and water among children to prevent the spread of and trust. Which principle should guide her decision?
gastrointestinal infections in the community. Promoting *
regular handwashing with soap and water among children is 1 point
an example of which level of prevention in community health A. Veracity
nursing in the Philippines? B. Beneficence
* C. Justice
1 point D. Autonomy
A. Primary prevention 28. If Nurse Taeyeon needs to decide whether to allocate the
B. Secondary prevention medication to a patient who has recently arrived but is in
C. Tertiary prevention critical condition, which principle should guide her decision?
D. Quaternary prevention *
Situation: Nurse Jaehyun is studying for his licensure exam 1 point
and needs to review the health laws in the Philippines that A. Beneficence
impact nursing practice. He focuses on understanding the B. Autonomy
key legislation to ensure he provides care within legal C. Justice
guidelines. D. Nonmaleficence
21. Which law in the Philippines focuses on integrating 29. If Nurse Taeyeon faces a situation where adhering to the
traditional and alternative medicine into the primary health ethical principles might lead to conflicts with the community’s
care system? expectations, which principle should guide her decision?
* *
1 point 1 point
A. RA 7875 A. Beneficence
B. RA 8423 B. Autonomy
C. RA 9211 C. Justice
D. RA 11332 D. Nonmaleficence
22. Which law in the Philippines governs the regulation and 30. During a resource allocation meeting, Nurse Taeyeon is
control of tobacco use? concerned that a proposed budget cut might lead to reduced
* staffing levels, potentially increasing patient wait times and
1 point compromising care quality. Which ethical principle should
A. RA 9211 Taeyeon emphasize when arguing against the budget cut?
B. RA 7394 *
C. RA 8749 1 point
D. RA 9165 A. Beneficence
B. Nonmaleficence
C. Veracity A. Veracity
D. Justice B. Beneficence
Situation: A community health nurse, Nurse Nayeon, is C. Non-maleficence
preparing for a local health campaign aimed at addressing a D. Justice
recent outbreak of measles in an urban barangay. The 38. Nurse Joy is asked to participate in a research study that
campaign involves educating the community about the signs involves administering an experimental drug to patients
and symptoms of measles, how it is transmitted, the benefits without their explicit consent, as the study sponsor claims it
of vaccination, and specific advice for vulnerable groups is harmless. Which ethical principle should Nurse Joy
such as pregnant women. In addition, Nurse Nayeon is prioritize?
tasked with conducting contact tracing to prevent further *
spread of the disease. 1 point
31. Nurse Nayeon is preparing educational materials about A. Autonomy
the signs and symptoms of measles for a local health B. Justice
campaign. What are the common signs and symptoms of C. Beneficence
measles? D. Non-maleficence
* 39. A terminally ill patient in severe pain requests that Nurse
1 point Joy administer a higher dose of pain medication than
A. High fever, rash, cough, runny nose, red eyes prescribed, knowing it could potentially hasten their death.
B. Muscle aches, joint pain, diarrhea, sore throat Which ethical principle should Nurse Joy consider in making
C. Headache, vomiting, abdominal pain, fatigue her decision?
D. Chest pain, shortness of breath, palpitations, dizziness *
32. Nurse Nayeon is preparing educational materials about 1 point
measles transmission for a local school health program. How A. Beneficence
is measles transmitted from person to person? B. Autonomy
* C. Non-maleficence
1 point D. Justice
A. Through respiratory droplets when an infected person 40. During a hospital staffing meeting, Nurse Joy is faced with
coughs or sneezes a decision about how to fairly distribute shifts among staff
B. Through contaminated food or water members to cover a sudden influx of patients. Some staff
C. Through contact with infected animals members have requested time off, while others are willing to
D. Through sexual contact with an infected person take on extra shifts. Which ethical principle should guide
33. Which virus is the causative agent of measles? Nurse Joy in making the staffing decisions?
* *
1 point 1 point
A. Rubella virus A. Beneficence
B. Varicella-zoster virus B. Autonomy
C. Paramyxovirus C. Non-maleficence
D. Epstein-Barr virus D. Justice
34. A pregnant woman visits a health center, concerned about Situation: Nurse IU is responsible for implementing proper
her potential exposure to measles. What advice should Nurse waste segregation practices in a hospital to ensure safety
Nayeon provide to the pregnant woman regarding measles? and compliance with environmental regulations.
* 41. Nurse IU encounters used needles, syringes, and scalpel
1 point blades in the hospital ward. Which color-coded bin should she
A. Measles infection during pregnancy can lead to use for disposing of these items?
complications such as premature birth or low birth weight. *
B. The measles vaccine is safe and recommended during 1 point
pregnancy to protect both the mother and the baby. A. Black
C. If symptoms of measles develop, she should seek B. Green
medical attention immediately. C. Yellow
D. All of the above D. Red
35. Nurse Nayeon is conducting contact tracing for a 42. Nurse IU collects used gloves, used foley catheters, and
confirmed measles case in a community. What actions should used test strips from patient care activities. Which
the nurse take in conducting contact tracing for measles? color-coded bin should she designate for disposing of these
* items?
1 point *
A. Identify and notify individuals who may have been 1 point
exposed to measles. A. Black
B. Provide information about measles symptoms and B. Green
vaccination to contacts. C. Yellow
C. Recommend isolation or quarantine for unvaccinated D. Yellow with black band
contacts. 43. Nurse IU identifies empty bottles of laboratory reagents
D. All of the above and expired medicines. Which color-coded bin should she use
Situation: Nurse Joy is a senior nurse at a busy metropolitan for disposing of these items?
hospital. She often encounters complex ethical dilemmas *
involving patient care, staff management, and resource 1 point
allocation. She must apply her knowledge of ethical A. Pink
principles to navigate these challenges effectively. B. Green
36. Nurse Joy is faced with a situation where a patient C. Yellow
refuses a recommended treatment due to personal beliefs, D. Yellow with black band
but the treatment is critical for their recovery. Which ethical 44. Nurse IU collects used paper products and packaging
principle should Nurse Joy consider when respecting the materials from administrative areas. Which color-coded bin
patient's decision? should she designate for disposing of these items?
* *
1 point 1 point
A. Beneficence A. Black
B. Autonomy B. Green
C. Non-maleficence C. Pink
D. Justice D. Orange
37. Nurse Joy is considering whether to report discrepancies 45. Nurse IU identifies radioactive materials used in
in a research study funded by the hospital, which could affect diagnostic procedures, such as used x-ray films and test
patient care outcomes. Which ethical principle should guide tubes containing radioactive substances. Which color-coded
her decision? bin should she use for disposing of these items?
* *
1 point 1 point
A. Black 54. Nurse Wonwoo needs to recommend a plant for a patient
B. Green with a high uric acid level. Which plant should he choose for
C. Orange this condition?
D. Red *
Situation: Nurse Hoshi is tasked with educating a community 1 point
about the Expanded Program on Immunization (EPI) of the A. Lagundi
Department of Health of the Philippines. B. Sambong
46. Which of the following diseases is included in the C. Ulasimang Bato
Expanded Program on Immunization (EPI) schedule in the D. Bawang
Philippines for infants? 55. Nurse Wonwoo is preparing to advise a patient on a
* remedy for cough and cold. Which plant's preparation should
1 point he recommend?
A. Tuberculosis *
B. Dengue 1 point
C. Malaria A. Ampalaya
D. HIV/AIDS B. Yerba Buena
47. At what age is the first dose of the Hepatitis B vaccine C. Akapulko
administered under the Expanded Program on Immunization D. Niyog-Niyogan
(EPI) schedule in the Philippines? Situation: Nurse Baekhyun is working in a busy hospital ward
* and needs to ensure the safe administration of medications
1 point to patients.
A. Birth 56. Nurse Baekhyun is preparing to insert an intravenous (IV)
B. 6 weeks catheter. What is the correct angle for inserting the catheter
C. 10 weeks into the vein to ensure proper placement?
D. 14 weeks *
48. Which of the following is a normal progression of the skin 1 point
reaction after receiving the BCG vaccine? A. 10-15 degrees
* B. 25-30 degrees
1 point C. 45-60 degrees
A. Immediate swelling followed by an ulcer D. 75-90 degrees
B. A small bump that becomes a larger ulcer before healing 57. What is the correct method for verifying a patient’s identity
C. Rapid disappearance of redness and swelling before administering medication?
D. Formation of a large, hard lump with significant pain *
49. Which vaccine is NOT included in the routine childhood 1 point
immunization schedule of the Expanded Program on A. Checking the patient’s chart
Immunization (EPI) in the Philippines? B. Asking the patient’s roommate
* C. Using at least two patient identifiers
1 point D. Looking at the patient’s bed number
A. Hepatitis B vaccine 58. Which of the following is the primary reason for using
B. MMR vaccine aseptic technique during a procedure?
C. Rotavirus vaccine *
D. Yellow fever vaccine 1 point
50. Which disease is targeted for elimination through A. To enhance the healing process
vaccination efforts under the Expanded Program on B. To prevent the spread of infection
Immunization (EPI) in the Philippines? C. To improve patient comfort
* D. To reduce the cost of healthcare
1 point 59. Nurse Baekhyun needs to check a patient's blood
A. Tuberculosis pressure. Which of the following steps should be performed
B. Poliomyelitis first?
C. Malaria *
D. HIV/AIDS 1 point
Situation: Nurse Wonwoo is leading a community health A. Apply the blood pressure cuff to the patient's arm
workshop focused on the use of traditional medicinal plants B. Ensure the patient has not smoked or exercised within
endorsed by the DOH. To evaluate the attendees' the last 30 minutes
understanding of the uses and preparations of these plants, C. Inflate the cuff to the appropriate level
he presents the following question. D. Position the patient’s arm at heart level
51. Nurse Wonwoo is advising a patient with a persistent 60. Nurse Baekhyun has just finished administering a series
cough and fever. Which of the following traditional medicinal of injections. What is the correct procedure for disposing of
plants should he recommend? used needles and syringes?
* *
1 point 1 point
A. Lagundi A. Place them in a sharps container
B. Yerba Buena B. Submerge them in a bleach solution and discard
C. Sambong C. Dispose of them in a regular trash bin
D. Guava D. Rinse them and put them in a recycling bin
52. Nurse Wonwoo is advising a patient with severe insect Situation: Nurse Hong Hae-in is working on developing a
bites. Which plant should he recommend for topical comprehensive care plan for a newly admitted patient, Mr.
application? Baek Hyun-woo, who is a 65-year-old male with a history of
* diabetes mellitus and hypertension. Mr. Baek Hyun-woo was
1 point admitted to the hospital with complaints of chest pain and
A. Lagundi shortness of breath. Upon admission, Nurse Hong needs to
B. Yerba Buena gather detailed health information, identify the patient's
C. Guava problems, set measurable goals, implement appropriate
D. Akapulko interventions, and evaluate the outcomes of the care
53. Nurse Wonwoo is helping a patient who has been provided.
suffering from constipation and digestive issues. Which plant 61. What is the first step in the nursing process that Nurse
should he recommend for its anti-diarrheal properties? Hong Hae-in should perform when developing a care plan for
* Mr. Baek Hyun-woo?
1 point *
A. Blumea balsamifera 1 point
B. Carmona retusa A. Diagnosis
C. Quisqualis indica L. B. Planning
D. Mamordica Charantia C. Assessment
D. Implementation
62. During the assessment phase, Nurse Hong Hae-in B. To verify that all necessary preoperative preparations
identifies that Mr. Baek Hyun-woo has elevated blood have been completed
pressure and glucose levels. Which of the following activities C. To administer preoperative medications
is part of the diagnosis step in the nursing process? D. To prepare the operating room
* Situation: Nurse Cha Eun Jae is currently pursuing her
1 point Master's degree in Nursing and is working on her thesis
A. Setting goals for patient care related to patient outcomes in postoperative care. She is
B. Analyzing patient data to identify health problems required to design a research study, select appropriate
C. Implementing nursing interventions methodologies, collect data, and analyze the results. She
D. Collecting patient health history needs to ensure her study is rigorous and adheres to ethical
63. After identifying Mr. Baek Hyun-woo's health problems, guidelines.
Nurse Hong Hae-in sets measurable and achievable goals for 71. What is the first step Nurse Cha Eun Jae should take when
his care. Which step of the nursing process is being beginning her research study?
performed? *
* 1 point
1 point A. Conduct a literature review
A. Assessment B. Formulate a research question
B. Diagnosis C. Select a research design
C. Planning D. Collect data
D. Implementation 72. After formulating her research question, what should
64. Nurse Hong Hae-in begins to administer medications and Nurse Cha Eun Jae do next?
educate Mr. Baek Hyun-woo on lifestyle changes to manage *
his conditions. Which of the following describes the 1 point
implementation step of the nursing process? A. Develop a hypothesis
* B. Conduct a literature review
1 point C. Collect pilot data
A. Evaluating the effectiveness of the nursing interventions D. Write the research proposal
B. Carrying out the planned nursing interventions 73. Nurse Cha Eun Jae needs to decide on a research design
C. Developing a comprehensive care plan for her study. Which of the following should she consider
D. Diagnosing patient health problems during this step?
65. In the evaluation step of the nursing process, what is *
Nurse Hong Hae-in primarily focused on for Mr. Baek 1 point
Hyun-woo? A. Ethical approval
* B. Data collection methods
1 point C. Formulating a hypothesis
A. Creating a nursing diagnosis D. Writing the literature review
B. Collecting patient health data 74. When Nurse Cha Eun Jae is ready to collect data, what
C. Determining if patient goals have been met should be her primary focus to ensure data quality?
D. Performing nursing interventions *
Situation: Nurse Park Shin Hye is working in the 1 point
perioperative unit of a large urban hospital. She is A. Speed of data collection
responsible for preparing patients for surgery, monitoring B. Consistency and accuracy of data collection
them during procedures, and providing postoperative care. C. Number of participants
Today, her patients include a 45-year-old female for a D. Length of the questionnaire
laparoscopic cholecystectomy, a 60-year-old male for hernia 75. Nurse Cha Eun Jae is analyzing her collected data. What
repair, and a 30-year-old female for a cesarean section. is the main purpose of this step in the research process?
66. What is the primary goal of preoperative nursing care? *
* 1 point
1 point A. To finalize the research question
A. To assess the patient's vital signs B. To test the hypothesis and draw conclusions
B. To prepare the patient physically and emotionally for C. To select the sample population
surgery D. To obtain ethical approval
C. To administer medications Situation: You are a public health consultant tasked with
D. To discharge the patient implementing a comprehensive community development
67. During the intraoperative phase, what is Nurse Park Shin program in a rural area with various social, economic, and
Hye's primary responsibility? environmental challenges. Your goal is to ensure that your
* strategies align with the 17 Sustainable Development Goals
1 point (SDGs) adopted by the United Nations.
A. Administering anesthesia 76. Your program aims to empower women and girls in the
B. Monitoring the patient's vital signs and maintaining a community. Which SDG aligns with this objective?
sterile environment *
C. Preparing the surgical instruments 1 point
D. Discharging the patient after surgery A. SDG 1
68. What is the purpose of the postoperative phase in B. SDG 4
perioperative nursing care? C. SDG 5
* D. SDG 10
1 point 77. If your program aims to promote sustained, inclusive, and
A. To prepare the patient for surgery sustainable economic growth, full and productive
B. To provide care and facilitate recovery after surgery employment, and decent work for all, which SDG should it
C. To administer preoperative medications align with?
D. To educate the patient about the surgical procedure *
69. Which of the following is a common postoperative 1 point
complication that Nurse Park Shin Hye should monitor for? A. SDG 7
* B. SDG 8
1 point C. SDG 9
A. Hypertension D. SDG 12
B. Deep vein thrombosis (DVT) 78. You want to ensure the community has access to clean
C. Hyperglycemia water and sanitation. Which SDG should guide your efforts?
D. Hypokalemia *
70. What is the main purpose of the preoperative checklist 1 point
used by Nurse Park Shin Hye? A. SDG 4
* B. SDG 5
1 point C. SDG 6
A. To ensure the patient has fasted appropriately D. SDG 13
79. To improve the community’s resilience and capacity to A. Direct contact with respiratory droplets
recover from natural disasters, which SDG should be your B. Airborne particles
priority? C. Fecal-oral route
* D. Vector-borne transmission
1 point 88. To confirm a diagnosis of poliomyelitis, specific
A. SDG 1 diagnostic tests are used to detect the presence of the virus.
B. SDG 9 What is one of the primary diagnostic methods used to
C. SDG 11 identify poliovirus?
D. SDG 15 *
80. Your program includes initiatives to combat climate 1 point
change. Which SDG aligns with this focus? A. Blood smear
* B. Urinalysis
1 point C. Stool culture
A. SDG 7 D. Chest X-ray
B. SDG 12 89. The Oral Polio Vaccine (OPV) is given in a specific dosage
C. SDG 13 to ensure its effectiveness. What is the standard dosage for
D. SDG 15 OPV?
Situation: Key, a 58-year-old man with chronic obstructive *
pulmonary disease (COPD), was recently admitted to the 1 point
hospital after a severe exacerbation. His respiratory A. 0.5 mL
condition deteriorated, leading to an emergency B. 1 mL
tracheostomy to establish a secure airway. The nursing staff C. 2 drops
is now tasked with managing Key's tracheostomy care to D. 5 drops
ensure he maintains a stable respiratory status and avoids 90. The nurse is educating a parent about the poliomyelitis
complications. vaccination schedule for their child. Which statements about
81. Immediately after Key's tracheostomy procedure, the the Oral Polio Vaccine (OPV) and Inactivated Polio Vaccine
nurse should prioritize which of the following interventions? (IPV) are correct?
* 1. OPV is administered as 2 drops orally.
1 point 2. IPV should be administered intramuscularly in the
A. Encouraging the patient to try speaking deltoid muscle for infants.
B. Administering prescribed antibiotics 3. The first dose of IPV is given at 3 ½ months of age.
C. Assessing for bilateral breath sounds and proper tube 4. OPV should be administered at 2 months, 4
placement months, and 6-18 months.
D. Initiating enteral feedings as ordered 5. IPV is administered intramuscularly in the
82. Why is hyperoxygenation essential before tracheal anterolateral thigh for infants.
suctioning in Key's case? *
* 1 point
1 point A. 1, 3, 4
A. To prevent mucus from becoming too thick B. 1, 2, 5
B. To eliminate the need for suctioning C. 2, 3, 5
C. To minimize the risk of hypoxia during suctioning D. 1, 3, 5
D. To ensure the cuff is properly inflated Situation: Ryu Sun Jae, a 35-year-old male, has recently
83. Key presents with subcutaneous emphysema around his returned from a vacation in Palawan, Philippines, where
tracheostomy site. This finding indicates which complication? malaria is endemic. He presents at the clinic with complaints
* of fever, chills, headaches, and muscle aches for the past
1 point three days. He did not take any preventive measures against
A. Infection malaria during his stay.
B. Hemorrhage 91. Which species of Plasmodium is most commonly
C. Pneumothorax associated with severe malaria in the Philippines?
D. Tracheoesophageal fistula *
84. Which type of tracheostomy tube would allow Key to 1 point
speak? A. Plasmodium vivax
* B. Plasmodium ovale
1 point C. Plasmodium malariae
A. Double-lumen tube D. Plasmodium falciparum
B. Single-lumen tube 92. Which type of mosquito is the primary vector for
C. Fenestrated tube transmitting malaria?
D. Cuffed tube *
85. Key is anxious about his tracheostomy and asks if he will 1 point
ever be able to eat normally again. What should the nurse A. Aedes
explain to him? B. Culex
* C. Anopheles
1 point D. Mansonia
A. "You will not be able to eat normally with a 93. Which blood smear finding is most indicative of malaria?
tracheostomy." *
B. "You can eat normally once the tracheostomy tube is 1 point
removed." A. Presence of parasites
C. "You can eat, but the cuff must be inflated during meals B. Elevated white blood cell count
to prevent aspiration." C. Increased hemoglobin levels
D. "You will need to rely on enteral feeding indefinitely." D. Decreased platelet count
86. Gu Jun-pyo is suspected of having poliomyelitis after 94. Which drug is commonly used for chemoprophylaxis
recent travel to an endemic area. What is the causative agent against malaria and should be started 1-2 weeks before
of poliomyelitis? entering an endemic area?
* *
1 point 1 point
A. Bacteria A. Quinine
B. Fungus B. Chloroquine
C. Virus C. Primaquine
D. Parasite D. Artemisinin-based combination therapy (ACT)
87. Poliomyelitis is known for its specific modes of 95. What is the typical incubation period for Plasmodium
transmission. Which of the following is the primary mode of falciparum malaria?
transmission for poliomyelitis? *
* 1 point
1 point A. 1-7 days
B. 9-14 days D. Ordinal
C. 12-18 days 2. Nurse Chae Song-hwa records the pain levels of patients on
D. 18-40 days a scale from 1 to 10. What type of data is this?
Situation: Do Da-hee, a 35-year-old woman, presents to the *
clinic with complaints of persistent diarrhea, abdominal pain, 1 point
and occasional bloody stools for the past three weeks. She A. Nominal
mentions recent travel to an area with poor sanitation. On B. Ordinal
examination, she appears fatigued and has mild abdominal C. Interval
tenderness in the right upper quadrant. Stool tests are D. Ratio
ordered to investigate the cause of her symptoms. 3. Nurse Chae Song-hwa measures the weight of patients
96. Which diagnostic test would most likely confirm the before and after implementing the new wound care protocol.
presence of Entamoeba histolytica? What type of data is ‘weight’?
* *
1 point 1 point
A. Blood culture A. Nominal
B. Stool examination B. Ordinal
C. Urinalysis C. Interval
D. Chest X-ray D. Ratio
97. What is the primary mode of transmission of Entamoeba 4. Nurse Chae Song-hwa collects data on patients' ages to
histolytica? determine the average age of those participating in the study.
* What type of data is age?
1 point *
A. Direct contact with infected individuals 1 point
B. Contaminated food and water A. Nominal
C. Inhalation of spores B. Ordinal
D. Vector-borne (e.g., mosquitoes) C. Interval
98. What is the drug of choice for treating the intestinal phase D. Ratio
of amoebiasis? 5. Nurse Chae Song-hwa uses a thermometer to measure
* patients' body temperatures. What type of data is body
1 point temperature in Celsius?
A. Amoxicillin *
B. Metronidazole 1 point
C. Fluconazole A. Nominal
D. Loratadine B. Ordinal
98. What is the typical incubation period for symptoms of C. Interval
amoebiasis after exposure? D. Ratio
* Situation: Nurse Song Hye-kyo has recently been promoted
1 point to a nurse manager position at her hospital. She is
A. 1 to 2 days responsible for overseeing a team of nurses and ensuring
B. 1 to 2 weeks efficient and effective patient care.
C. 2 to 4 weeks 6. Nurse Song Hye-kyo is setting specific goals for her nursing
D. 1 to 3 months team and developing strategies to achieve them. Which
99. What is the most common extraintestinal complication of function of management is she performing?
amoebiasis? *
* 1 point
1 point A. Controlling
A. Skin rash B. Planning
B. Liver abscess C. Directing
C. Pneumonia D. Organizing
D. Myocarditis 7. Nurse Song Hye-kyo is allocating tasks and coordinating
100. Which preventive measure is crucial for reducing the risk the activities of her team to ensure that patient care is
of amoebiasis? delivered efficiently. Which function of management is she
* performing?
1 point *
A. Regular use of insect repellent 1 point
B. Safe drinking water and proper sanitation A. Planning
C. Wearing protective clothing B. Controlling
D. Vaccination C. Organizing
D. Directing
8. Nurse Song Hye-kyo is inspiring her team to adhere to the
new wound care protocol and providing support to ensure
they are motivated. Which function of management is she
performing?
NP2 *
1 point
A. Planning
B. Controlling
https://docs.google.com/forms/d/e/1FAIpQLSdRQBKwRjJuQP C. Organizing
FsyDOcIvTpi421QZggr7NF0AIVpHcoqlon7w/viewscore?view D. Directing
score=AE0zAgAjznsnJXULF80ThQ42mkXOte9t9fwFVksXTR 9. Nurse Song Hye-kyo is reviewing patient care reports and
azEm0FqzQzwkEx_0TUfvEt9hUHm6Y evaluating the performance of her nursing team to identify
Situation: Nurse Chae Song-hwa is conducting research on areas for improvement. Which function of management is she
the effectiveness of a new wound care protocol in a hospital performing?
setting. She needs to collect various types of data to analyze *
the results effectively. 1 point
1. Nurse Chae Song-hwa is collecting data on the different A. Planning
categories of wounds (e.g., surgical, traumatic, pressure B. Organizing
ulcers). What type of data is this? C. Staffing
* D. Controlling
1 point 10. Nurse Song Hye-kyo is recruiting new nurses to join her
A. Interval team, ensuring they are properly trained and competent in
B. Ratio their roles. Which function of management is she performing?
C. Nominal *
1 point
A. Planning 18. During the prenatal visit, Mrs. Do Bong-soon asks about
B. Organizing the risks associated with gestational hypertension. Nurse
C. Staffing Nam Ha-nuel explains that untreated gestational hypertension
D. Controlling can lead to which of the following complications?
Situation: Nurse Son Ye-jin has recently been promoted to *
the position of head nurse in a busy emergency department. 1 point
She faces the challenge of managing a diverse team of A. Anemia
nurses, each with different levels of experience and B. Placental abruption
expertise. Nurse Son Ye-jin values open communication and C. Gestational diabetes
seeks to create an environment where every team member D. Low birth weight
feels empowered to contribute to decision-making 19. Nurse Nam Ha-nuel assesses Mrs. Do Bong-soon’s
processes. understanding of her condition. Which statement by Mrs. Do
11. Which type of leadership is Nurse Son Ye-jin Bong-soon indicates a need for further teaching?
demonstrating by involving her team in decision-making? *
* 1 point
1 point A. "I need to monitor my blood pressure regularly at home."
A. Democratic leadership B. "I can continue with my usual salt intake without any
B. Autocratic leadership changes."
C. Transactional leadership C. "I should rest frequently, especially in the lateral position."
D. Laissez-faire leadership D. "I need to watch for any signs of severe headache or
12. What leadership style is Nurse Son Ye-jin likely employing visual changes."
during critical situations when she motivates her team to 20. Mrs. Do Bong-soon is curious about the follow-up and
perform at their best? management of gestational hypertension. What intervention
* does Nurse Nam Ha-nuel emphasize as crucial in managing
1 point gestational hypertension?
A. Transactional leadership *
B. Transformational leadership 1 point
C. Servant leadership A. Weekly visits for fetal heart rate monitoring
D. Charismatic leadership B. Increased physical activity to lower blood pressure
13. Which type of leadership is characterized by Nurse Son C. Routine use of diuretics to reduce blood pressure
Ye-jin's emphasis on creating an environment where D. Regular monitoring of blood pressure and weight
collaboration and trust thrive? Situation: Nurse Moon Gang-tae is providing education to
* Ms. Ko Moon-young, a pregnant woman diagnosed with
1 point gestational diabetes mellitus (GDM) and requiring insulin
A. Laissez-faire leadership therapy to manage her blood glucose levels during
B. Transactional leadership pregnancy.
C. Transformational leadership 21. Nurse Moon Gang-tae explains to Ms. Ko Moon-young the
D. Servant leadership importance of managing gestational diabetes. Why is it
14. In what leadership style does Nurse Son Ye-jin empower critical for Ms. Ko Moon-young to maintain tight glycemic
her team by providing support and opportunities for growth? control during pregnancy?
* *
1 point 1 point
A. Transactional leadership A. To prevent fetal macrosomia
B. Transformational leadership B. To monitor the effectiveness of insulin therapy
C. Servant leadership C. To prevent premature labor
D. Charismatic leadership D. To adjust dietary intake during pregnancy
15. Which leadership style might Nurse Son Ye-jin adopt 22. Nurse Moon Gang-tae discusses insulin therapy options
during routine operations when she allows her team members with Ms. Ko Moon-young for managing her gestational
to make decisions independently within established diabetes. Which type of insulin is typically used for managing
guidelines? mealtime blood glucose spikes in pregnant women with
* GDM?*
1 point 1 point
A. Laissez-faire leadership A. Lispro (Humalog)
B. Autocratic leadership B. Regular (Novolin R)
C. Democratic leadership C. Glargine (Lantus)
D. Transformational leadership D. NPH (Humulin N)
Situation: Nurse Nam Ha-nuel is caring for Mrs. Do 23. Nurse Moon Gang-tae reviews with Ms. Ko Moon-young
Bong-soon, who is 32 weeks pregnant and has been the signs of hypoglycemia and their implications during
diagnosed with gestational hypertension. Mrs. Do Bong-soon pregnancy. Which symptom is a common indicator of
has come to the clinic for her routine prenatal visit, and hypoglycemia and requires immediate intervention?
Nurse Nam Ha-nuel is assessing her blood pressure and *
educating her on the condition and its potential 1 point
complications. A. Bradycardia
16. Nurse Nam Ha-nuel is explaining the diagnostic criteria for B. Visual disturbances
gestational hypertension to Mrs. Do Bong-soon. Which blood C. Increased urination
pressure reading meets the criteria for gestational D. Muscle cramps
hypertension according to AHA guidelines? 24. Nurse Moon Gang-tae educates Ms. Ko Moon-young on
* the importance of insulin storage during pregnancy. How long
1 point can Ms. Ko Moon-young safely store her insulin, and where
A. 120/80 mm Hg should she keep it to maintain its effectiveness?
B. 130/85 mm Hg *
C. 140/90 mm Hg 1 point
D. 150/95 mm Hg A. Up to 1 month in the refrigerator door
17. Nurse Nam Ha-nuel educates Mrs. Do Bong-soon on the B. Up to 2 weeks in direct sunlight
potential progression of gestational hypertension. Which of C. Up to 3 months in a cool, dark place
the following is a key characteristic of preeclampsia? D. Up to 6 months in the bathroom cabinet
* 25. Nurse Moon Gang-tae reviews proper insulin injection
1 point techniques with Ms. Ko Moon-young to ensure optimal
A. Blood pressure less than 120/80 mm Hg absorption. What precaution should Ms. Ko Moon-young take
B. Absence of proteinuria to minimize discomfort during insulin injections?
C. Presence of seizures *
D. Blood pressure greater than or equal to 140/90 mm Hg 1 point
with proteinuria A. Inject quickly to minimize pain
B. Rotate injection sites within the same area
C. Inject into muscle for faster action D. Refers to the capacity for self-evaluation and criticism.
D. Use the same needle for multiple injections 33. Nurse Park is discussing the development of the
Situation: Nurse Cha Eun-woo is educating Ms. Moon Superego. What statement accurately describes the
Ga-young, a 25-year-old woman, about her menstrual cycle. Superego?
Ms. Moon is concerned about the regularity, duration, and *
flow of her periods. She wants to understand what is 1 point
considered normal and when she should seek medical A. Functions include reality testing and problem solving.
advice. B. Emerges out of the ID and acts as an intermediary
26. Nurse Cha Eun-woo is educating Ms. Moon Ga-young on between the ID and the external world.
the normal duration of a menstrual cycle. What is the average C. Includes internalization of the values, ideals, and moral
length of a menstrual cycle in most women? standards of parents and society.
* D. Consists of the primary process psychological activity.
1 point 34. Alex was aware that she could steal materials from her
A. 21 days workplace without anybody noticing. Even though she would
B. 28 days most likely never be caught, she chose not to steal as she
C. 35 days knew it was wrong to do so. Which of the following actions
D. 40 days fits the psychoanalytic theory of Freud?
27. Nurse Cha Eun-woo explains to Ms. Moon Ga-young that *
the duration of menstrual bleeding can vary. What is the 1 point
average range for the number of days a woman may A. The ID
experience menstrual bleeding? B. The Ego
* C. The Libido
1 point D. The Superego
A. 1-3 days 35. Christine was thirsty. She stretched across the table and,
B. 4-6 days much to Mr. Castro's surprise, drank from his water glass
C. 7-10 days instead of waiting for the service to refill hers. According to
D. 11-14 days Freud's psychoanalytic theory, which of the following
28. Nurse Cha Eun-woo reviews the typical amount of behaviors is it?
menstrual flow with Ms. Moon Ga-young. What is the average *
amount of menstrual flow during a period? 1 point
* A. The ID
1 point B. The Ego
A. 10-30 mL C. The Libido
B. 30-80 mL D. The Superego
C. 80-100 mL Situation: Jhoanna, a primigravida at 39 weeks gestation,
D. 100-150 mL arrives at the hospital in active labor. She is experiencing
29. Nurse Cha Eun-woo discusses with Ms. Moon Ga-young regular contractions every 3-4 minutes, lasting 60 seconds
the normal characteristics of menstrual odor. What should each. Her cervix is dilated to 5 cm upon admission.
Ms. Moon Ga-young understand if her menstrual blood has an 36. Nurse Mikha explains the stages of labor to Jhoanna and
odor similar to marigold? her partner. Which of the following best describes the first
* stage of labor?
1 point *
A. Presence of bacterial infection 1 point
B. Normal hormonal changes A. Begins with full cervical dilation to 10 cm and ends with
C. Excessive menstrual flow the birth of the baby.
D. Recent dietary changes B. Involves the descent of the presenting part through the
30. Nurse Cha Eun-woo educates Ms. Moon Ga-young about birth canal.
normal characteristics of menstrual blood. Which of the C. Begins with the onset of true labor contractions and
following statements about the color of menstrual blood is ends with full cervical dilation.
accurate? D. Is characterized by the delivery of the placenta after the
* birth of the baby.
1 point 37. During Jhoanna's labor, Nurse Mikha observes her
A. Menstrual blood is typically bright red in color. experiencing contractions approximately every 3-5 minutes,
B. Menstrual blood is usually dark red or brown in color. lasting 40-60 seconds. What stage of labor is Jhoanna likely
C. Menstrual blood is yellowish in color. in?
D. Menstrual blood color varies from white to pink. *
Situation: In a university psychology seminar, Dr. Park leads 1 point
a discussion on Freud's psychoanalytic theory. Students from A. First stage, transition phase
various disciplines attend, eager to learn about the ID, Ego, B. First stage, active phase
and Superego. Dr. Park uses scenarios to illustrate how C. Second stage
these components influence human behavior and D. Third stage
decision-making, encouraging active participation and 38. Nurse Mikha is discussing the sequence of cardinal
application of Freudian concepts in real-life situations. movements of labor with Jhoanna. Which of the following
31. As they went along, they talked about the components of represents the correct chronological order of these
personality according to Freudian theory. Which of the movements?
following best describes the ID? *
* 1 point
1 point A. Engagement, flexion, descent, internal rotation,
A. Begins its development during the fourth or fifth month extension, external rotation, expulsion.
of life. B. Engagement, descent, flexion, internal rotation,
B. Functions include reality testing and problem solving. extension, external rotation, expulsion.
C. Acts immediately in an impulsive, irrational way without C. Descent, engagement, flexion, internal rotation,
considering consequences. extension, external rotation, expulsion.
D. Consists of the conscience and the ego ideal. D. Descent, engagement, internal rotation, flexion,
32. Dr. Park is explaining the role of the Ego in personality extension, external rotation, expulsion.
development. Which statement accurately describes the Ego? 39. Jhoanna is now in the second stage of labor, actively
* pushing with each contraction. What is the primary goal of
1 point this stage?
A. Necessary part of socialization that develops during the *
phallic stage at 3 to 6 years of age. 1 point
B. Emerges out of the ID and acts as an intermediary A. Cervical dilation from 0 to 10 cm
between the ID and the external world. B. Descent of the presenting part through the birth canal
C. Consists of internalized values, ideals, and moral C. Birth of the baby
standards learned during childhood. D. Delivery of the placenta
40. Nurse Mikha is assessing Jhoanna during the third stage clothes. Which of Erikson’s stages is Nurse Sunghoon
of labor. What is the typical duration of the third stage of addressing with this intervention?
labor, which begins with the birth of the baby and ends with *
the delivery of the placenta? 1 point
* A. Initiative vs. Guilt
1 point B. Industry vs. Inferiority
A. 5-10 minutes C. Autonomy vs. Shame and Doubt
B. 15-30 minutes D. Trust vs. Mistrust
C. 45-60 minutes 48. Nurse Sunghoon is working with a school-aged child who
D. 90-120 minutes is hospitalized. To support the child's development, which of
Situation: Nurse Jake is preparing a group of nursing the following interventions is most appropriate?
students for their upcoming exam on Erik Erikson's theories *
of growth and development. He uses complex real-life 1 point
scenarios to explain each stage and its corresponding A. Allow self-feeding opportunities.
psychosocial crisis, emphasizing the nuances and common B. Encourage the child to continue schoolwork while
misconceptions that students might encounter. hospitalized.
41. Nurse Jake presents a case study of a 5-year-old child C. Respect the child's choices and expressions of feelings.
who is eager to take on new challenges at home and in D. Take the health history and perform examinations
preschool. Which of Erikson's stages is the child most likely without the parents present.
navigating, and what is the psychosocial crisis involved? 49. Nurse Sunghoon is performing a health assessment on an
* adolescent. Which intervention is most suitable for this stage
1 point of Erikson's development?
A. Initiative vs. Guilt *
B. Autonomy vs. Shame and Doubt 1 point
C. Industry vs. Inferiority A. Encourage volunteer activities.
D. Trust vs. Mistrust B. Include support from the client’s partner or significant
42. During a seminar, Nurse Jake asks the students to identify other.
the primary challenge for adolescents according to Erikson's C. Take the health history and perform examinations
stages. Which of the following best describes this challenge? without the parents present.
* D. Assist with making changes to living arrangements.
1 point 50. Nurse Sunghoon is planning discharge for a middle-aged
A. Developing a sense of trust in caregivers client. Which of the following interventions aligns with
B. Establishing intimate relationships Erikson's stage of "Generativity vs. Stagnation"?
C. Forming a stable identity *
D. Achieving a sense of autonomy 1 point
43. Nurse Jake describes a scenario where a middle-aged A. Assist with making changes to living arrangements.
adult is focused on contributing to society through career and B. Listen attentively to reminiscent stories about their life’s
community involvement. Which of Erikson's stages is this accomplishments.
individual likely experiencing? C. Encourage volunteer activities.
* D. Take the health history and perform examinations
1 point without the parents present.
A. Integrity vs. Despair Option 5
B. Intimacy vs. Isolation Situation: Nurse Jungwon is leading a seminar on Kohlberg's
C. Generativity vs. Stagnation stages of moral development. He presents complex cases to
D. Industry vs. Inferiority help nursing students understand how to apply these
44. A nursing student asks Nurse Jake about the potential theories in their practice. The session focuses on
outcomes of failing to achieve the psychosocial task of recognizing moral reasoning in different age groups and
gaining some basic control over self and environment. Which understanding appropriate interventions.
of the following best represents a consequence of this 51. Nurse Jungwon is observing a toddler in a pediatric unit.
failure? According to Kohlberg's stages of moral development, which
* of the following behaviors is most likely demonstrated by a
1 point child at the Punishment-Obedience Orientation stage?
A. Developing a sense of inferiority *
B. Experiencing feelings of shame and doubt 1 point
C. Struggling with identity confusion A. The child follows rules to obtain rewards.
D. Facing isolation and loneliness B. The child obeys rules to avoid punishment.
45. Nurse Jake is teaching about the psychosocial crisis of C. The child makes judgments based on social order.
"Integrity vs. Despair." Which age group is most likely to face D. The child follows self-chosen ethical principles.
this crisis, and what is a key characteristic of successfully 52. During a session, Nurse Jungwon describes a
navigating it? preschooler's behavior of sharing toys to receive praise from
* teachers. Which Kohlberg's stage of moral development does
1 point this behavior exemplify?
A. Adolescents, developing a sense of identity *
B. Young adults, forming intimate relationships 1 point
C. Middle-aged adults, contributing to society A. Stage 1: Punishment-Obedience Orientation
D. Older adults, reflecting on life with a sense of fulfillment B. Stage 2: Instrumental Relativist Orientation
Situation: Nurse Sunghoon is conducting a training session C. Stage 3: Good Boy or Nice Girl Orientation
for new nurses on interventions that can assist clients in D. Stage 4: Law and Order Orientation
achieving Erikson’s stages of development. He emphasizes 53. Nurse Jungwon explains that a 10-year-old child feels
practical examples and real-life applications to help the guilty when behaving in a way that displeases their parents.
nurses understand how to support patients at different Which stage of moral development does this illustrate?
stages of their lives. *
46. Nurse Sunghoon is caring for an infant in the pediatric 1 point
ward. Which intervention is most appropriate to help the A. Stage 1: Punishment-Obedience Orientation
infant achieve Erikson's stage of development for infancy? B. Stage 2: Instrumental Relativist Orientation
* C. Stage 3: Good Boy or Nice Girl Orientation
1 point D. Stage 4: Law and Order Orientation
A. Allow the infant to make choices, ensuring safety. 54. A school-age child insists on following classroom rules
B. Hold and touch the infant frequently. even when no one is watching. According to Nurse Jungwon,
C. Encourage self-feeding opportunities. which stage of Kohlberg's moral development is the child
D. Offer medical equipment for play. likely in?
47. During a developmental assessment, Nurse Sunghoon *
encourages a 3-year-old child to remove and put on their own 1 point
A. Stage 1: Punishment-Obedience Orientation
B. Stage 2: Instrumental Relativist Orientation 61. Nurse Julie is assessing Natty, a 24-week pregnant
C. Stage 3: Good Boy or Nice Girl Orientation woman, who complains of shortness of breath. What is the
D. Stage 4: Law and Order Orientation most likely cause of this symptom?
55. An adolescent challenges a school policy they believe is *
unfair. Nurse Jungwon identifies this behavior as indicative of 1 point
which stage of Kohlberg's moral development? A. Decreased red blood cell volume
* B. Increased oxygen consumption and elevated diaphragm
1 point C. Increased levels of human chorionic gonadotropin (hCG)
A. Stage 2: Instrumental Relativist Orientation D. Elevated blood pressure
B. Stage 3: Good Boy or Nice Girl Orientation 62. Natty expresses concerns about occasional dizziness
C. Stage 4: Law and Order Orientation when standing up quickly. Nurse Julie explains that this is due
D. Stage 5: Social Contract and Legalistic Orientation to physiological changes during pregnancy. What change is
56. Nurse Jay is observing an infant who constantly seeks most likely causing this symptom?
comfort by sucking on a pacifier, even when not hungry. While *
this might suggest the oral stage, which additional behavior 1 point
would most strongly confirm this stage according to Freud's A. Increased blood volume
psychosexual theory? B. Elevated estrogen levels
* C. Postural hypotension due to vascular changes
1 point D. Increased progesterone levels
A. Demonstrating control over toilet training. 63. Natty reports constipation as another issue she is facing.
B. Seeking physical comfort and security from a caregiver. Which physiological change during pregnancy is likely
C. Exhibiting curiosity about their genital organs. contributing to this problem?
D. Engaging in intellectual activities and forming social *
bonds. 1 point
57. During rounds, Nurse Jay notices a toddler showing A. Increased levels of estrogen
resistance to toilet training and experiencing frustration. How B. Increased levels of progesterone
might this resistance reflect a possible conflict in Freud's C. Increased blood volume
psychosexual stages, and what long-term personality trait D. Increased heart size
could develop if this stage is not successfully resolved? 64. Nurse Julie advises Natty to increase her dietary iron
* intake. What physiological change during pregnancy
1 point increases the need for iron?
A. Oral fixation leading to issues with dependency and trust. *
B. Anal-retentive traits such as obsessiveness and 1 point
stinginess. A. Decreased red blood cell volume
C. Phallic fixation resulting in confusion about gender roles. B. Increased plasma volume
D. Latency stage struggles causing social withdrawal and C. Increased oxygen consumption
intellectual inhibition. D. Increased metabolic rate
58. A preschool-aged child in the hospital expresses intense 65. Which health teaching will Nurse Julie give to Natty to
jealousy when their same-sex parent spends time with their help manage her constipation during pregnancy? Select all
opposite-sex parent. While this could indicate the phallic that apply.
stage, what additional behavior would most confirm this 1. "Increase your fiber intake by eating more fruits, vegetables,
diagnosis? and whole grains."
* 2. "Drink at least 8-10 glasses of water daily to stay hydrated."
1 point 3. "Take a daily laxative to ensure regular bowel movements."
A. Seeking immediate gratification of oral needs. 4. "Engage in regular moderate exercise, such as walking."
B. Displaying curiosity and exploration of their genitalia. 5. "Avoid consuming dairy products, as they can worsen
C. Preferring to play alone and avoiding social interactions. constipation."
D. Showing a keen interest in academic pursuits. *
59. Nurse Jay is interacting with a 9-year-old child who 1 point
prioritizes schoolwork and expresses a strong desire to A. 1, 2, 4
please their teachers and parents. While these behaviors fit B. 2, 3, 5
the latency stage, which conflicting behavior would challenge C. 1, 4
this assumption and suggest a different stage? D. All of the above
* Situation: Nurse Hanni is conducting a prenatal check-up for
1 point Danielle, a 30-year-old woman who is 22 weeks pregnant
A. Showing dependency and seeking constant reassurance with her second child. Danielle is experiencing various
from caregivers. cardiovascular changes and has some concerns about her
B. Demonstrating intense sibling rivalry and aggressive heart and circulation. She reports feeling occasional
behavior towards peers. palpitations, mild swelling in her feet, and dizziness when
C. Developing a keen interest in forming romantic she stands up too quickly. Danielle wants to understand
relationships. these changes and how she can manage them.
D. Exhibiting regression to earlier stages, like bed-wetting. 66. Nurse Hanni explains to Danielle that her circulating blood
60. Nurse Jay is evaluating a 15-year-old patient who is volume has increased during pregnancy. What is the primary
exploring their sexuality and forming a distinct personal reason for this increase?
identity. Although this aligns with the genital stage, what *
behavior might indicate unresolved conflicts from an earlier 1 point
stage? A. To provide adequate nutrition to the fetus
* B. To compensate for blood loss during delivery
1 point C. To meet the increased metabolic demands of pregnancy
A. Excessive neatness and stubbornness. D. To improve renal function during pregnancy
B. Frequent nail-biting and thumb-sucking. 67. Danielle asks how her blood pressure might change
C. Avoidance of social interactions and activities. during pregnancy. What is the most accurate description of
D. Showing extreme jealousy of the same-sex parent. these changes?
Situation: Nurse Julie is working in the maternity ward of a *
hospital and is conducting an antenatal check-up for Natty, a 1 point
28-year-old woman who is 24 weeks pregnant with her first A. Blood pressure increases steadily throughout pregnancy.
child. Natty expresses concerns about the changes she is B. Blood pressure decreases slightly during the first
experiencing, including shortness of breath, occasional trimester, increases during the second trimester, and then
swelling in her ankles, and constipation. She also mentions returns to baseline by the third trimester.
feeling more tired than usual and experiencing some C. Blood pressure decreases slightly during the first
dizziness when standing up quickly. Natty is eager to trimester, remains low in the second trimester, and then
understand these changes and what she can do to manage increases in the third trimester.
them effectively. D. Blood pressure remains consistently high throughout
pregnancy.
68. Danielle is concerned about the mild swelling in her feet. Situation: Nurse Risa is attending to a newborn, Alice, just
Nurse Hanni explains that this is due to retention of sodium delivered vaginally at 39 weeks gestation. During the initial
and water. What additional advice should Nurse Hanni give to assessments at 1 and 5 minutes after birth, Nurse Risa
help manage this symptom? observes and documents the following:
*
1 point ● 1-Minute APGAR Assessment:
A. Increase salt intake ○ Heart rate: 85 beats per minute
B. Avoid drinking water before bedtime ○ Respiratory effort: Weak, irregular breathing
C. Elevate the feet when sitting ○ Muscle tone: Some flexion of extremities
D. Wear tight-fitting shoes ○ Reflex irritability: Grimace with stimulation
69. Nurse Hanni informs Danielle that physiological anemia ○ Color: Body is pink, but hands and feet are blue
occurs during pregnancy. What is the primary cause of this ● 5-Minute APGAR Assessment:
condition? ○ Heart rate: 120 beats per minute
* ○ Respiratory effort: Strong cry
1 point ○ Muscle tone: Active movement
A. Increased destruction of red blood cells ○ Reflex irritability: Vigorous cry with stimulation
B. Decreased production of red blood cells ○ Color: Pink all over
C. Plasma volume increasing more than red blood cell 76. Based on Nurse Risa's observations, what is the APGAR
volume score for Alice at 1 minute after birth?
D. Increased iron requirements *
70. Nurse Hanni advises Danielle on ways to prevent 1 point
dizziness and maintain good cardiovascular health during A. 5
pregnancy. Which of the following should be included in her B. 6
advice? Select all that apply. C. 7
1. Change positions slowly to avoid dizziness. D. 8
2. Lie down on your back to rest. 77. What is the APGAR score for Alice at 5 minutes after
3. Drink plenty of fluids throughout the day. birth?
4. Avoid standing for long periods. *
5. Increase salt intake to maintain blood pressure. 1 point
* A. 7
1 point B. 8
A. 1, 3, 4 C. 9
B. 2, 3, 5 D. 10
C. 1, 4 78. Given Alice's 1-minute APGAR score, which of the
D. All of the above following interventions should Nurse Risa consider? Select all
Situation: Nurse Winter is conducting a developmental that apply.
assessment on various pediatric patients at a community 1. Provide positive pressure ventilation.
health clinic. She is reviewing their growth and 2. Administer supplemental oxygen.
developmental milestones to ensure they are on track and 3. Initiate chest compressions.
providing guidance to their parents. 4. Keep the newborn warm and dry.
71. Nurse Winter is assessing a 6-month-old infant. Which 5. Stimulate the newborn to breathe.
finding is not typical for an infant of this age? *
* 1 point
1 point A. 1, 2, 4, 5
A. Height increases by 1 inch per month B. 2, 3, 5
B. Birth weight has tripled C. 1, 3, 4
C. Posterior fontanel has closed D. All of the above
D. Nocturnal sleep pattern lasts 9 to 11 hours 79. Which statement correctly describes the interventions for
72. During a well-child visit, Nurse Winter notes that a an Apgar score of 0-3 at 1 minute?
2-year-old toddler has a head circumference of 19 inches. *
What other physical characteristics are expected in a child of 1 point
this age? A. No intervention is needed; the newborn will stabilize on
* their own.
1 point B. Only provide supportive care with no additional
A. Weight doubled from birth weight resuscitation measures.
B. Height increase of 1 inch per month C. Newborn requires full resuscitation efforts, including
C. Anterior fontanel closed chest compressions if necessary.
D. No need for daytime naps D. Administer routine newborn care without additional
73. Nurse Winter is reviewing the growth of a 2-year-old monitoring.
toddler. Which statement is not accurate for this age? 80. Nurse Risa is preparing a report on Alice’s Apgar scores.
* Which of the following is accurate regarding the need for
1 point resuscitation and interventions based on an Apgar score of
A. Weight gain slows down compared to infancy 8-10?
B. Head circumference increases about 1 inch *
C. Anterior fontanel is still open 1 point
D. Height increase is about 3 inches per year A. Immediate resuscitation is required; no further
74. Nurse Winter is monitoring a 10-year-old child's growth interventions are needed.
and development. What physical growth pattern is expected B. The newborn needs continued assessment but no
for this age group? specific interventions beyond supportive care.
* C. The newborn requires advanced resuscitation measures,
1 point including medication.
A. Growth spurts of 6 inches per year D. Continuous monitoring is unnecessary if the score is
B. Weight gain of 10 pounds per year 8-10.
C. Height increase of 2 inches per year Situation: Nurse Karina is assessing a 6-month-old infant
D. Loss of all primary teeth by age 10 named Sam during a routine well-child visit. Sam’s mother is
75. Nurse Winter is discussing puberty with a group of concerned about his development and is seeking guidance
adolescents. Which statement about puberty is not true? on what to expect as he continues to grow. Nurse Karina
* provides information on typical developmental milestones,
1 point growth patterns, and appropriate types of play for Sam’s age.
A. Menstrual periods occur about 2.5 years after the onset 81. Which age brackets represent the most rapid periods of
of puberty growth in terms of height and weight for children?
B. Body mass reaches adult size *
C. Puberty starts at the same age for everyone 1 point
D. Sebaceous and sweat glands become fully functional A. Infancy (0-12 months) and Adolescence (12-18 years)
B. Toddlerhood (1-3 years) and Preschool age (3-6 years) Situation: Nurse Woo Young-woo is performing a prenatal
C. School age (6-12 years) and Infancy (0-12 months) assessment for Mrs. Choi Soo Yeon, who is 30 weeks
D. Adolescence (12-18 years) and Toddlerhood (1-3 years) pregnant. Woo Young-woo uses McDonald's rule to assess
82. Which of the following types of development describes the fundal height and determine the estimated gestational
the progression from the center of the body outward, as age. Accurate interpretation of these measurements is
observed in infants? crucial for monitoring fetal growth and development.
* 91. At 30 weeks of gestation, Nurse Woo Young-woo
1 point measures the fundal height of Mrs. Choi Soo Yeon. Where is
A. Cephalocaudal development the fundus expected to be located?
B. Proximodistal development *
C. Gross motor development 1 point
D. Fine motor development A. At the level of the symphysis pubis
83. Which developmental milestone is characteristic of B. At the umbilicus
proximodistal development in a 6-month-old infant? C. Between the umbilicus and the xiphoid process
* D. At the level of the xiphoid process
1 point 92. When Mrs. Choi Soo Yeon is 36 weeks pregnant, where
A. Reaching for toys with the entire hand would Nurse Woo Young-woo expect the fundal height to be
B. Grasping small objects with thumb and forefinger located?
C. Rolling over from back to stomach *
D. Sitting without support 1 point
84. Which type of play is most appropriate for a 6-month-old A. At the level of the symphysis pubis
infant? B. At the umbilicus
* C. Between the umbilicus and the xiphoid process
1 point D. At the level of the xiphoid process
A. Parallel play 93. What is the expected location of the fundus about an hour
B. Solitary play after Mrs. Choi Soo Yeon delivers her baby?
C. Cooperative play *
D. Dramatic play 1 point
85. Which of the following activities would best support a A. At the level of the umbilicus
6-month-old’s development of fine motor skills? B. At the level of the symphysis pubis
* C. At the level of the xiphoid process
1 point D. Slightly above the pubic bone
A. Playing with a large ball 94. If Nurse Woo Young-woo measures a fundal height of 24
B. Stacking large blocks cm in Mrs. Choi Soo Yeon, what would this indicate about her
C. Shaking a rattle gestational age?
D. Pulling a wagon *
Situation: Nurse Suzy is providing prenatal education to a 1 point
group of expectant mothers at a community health clinic. A. 20 weeks gestation
She discusses the different signs of pregnancy, categorizing B. 24 weeks gestation
them into presumptive, probable, and positive signs. The C. 28 weeks gestation
mothers in the group ask various questions to clarify their D. 32 weeks gestation
understanding, and Nurse Suzy provides detailed 95. If Nurse Woo Young-woo is assessing Mrs. Choi Soo
explanations. Yeon's fundal height at 18 weeks of pregnancy. What is the
86. Which of the following is not a presumptive skin change expected location of the fundus?
during pregnancy? *
* 1 point
1 point A. Above the symphysis pubis
A. Linea nigra B. At the level of the umbilicus
B. Melasma/Chloasma C. Near the xiphoid process
C. Striae Gravidarum D. Below the symphysis pubis
D. Palmar Erythema Situation: Nurse Im Sol is working with a new graduate nurse
87. Which of the following is not a probable sign of who is transitioning from a novice to a competent
pregnancy? practitioner. Nurse Im Sol is providing mentorship and
* guidance on how to navigate through the stages of skill
1 point acquisition as described in Benner’s Novice to Expert Model.
A. Positive serum hCG test The new graduate nurse is keen to understand the
B. Hegar's sign differences between the stages and how to apply them in
C. Quickening clinical practice.
D. Chadwick's sign 96. According to Benner’s Novice to Expert Model, which of
88. Which of the following would Nurse Suzy identify as not a the following best describes a "novice" nurse?
presumptive sign of pregnancy? *
* 1 point
1 point A. A nurse who relies on rules and guidelines to provide
A. Breast tenderness care and lacks experience.
B. Uterine enlargement B. A nurse who has a deep understanding of patient care
C. Frequent urination and anticipates needs.
D. Excessive nausea and vomiting C. A nurse who has extensive clinical experience and
89. Which of the following is considered a positive sign of intuitively understands complex situations.
pregnancy? D. A nurse who performs tasks efficiently and uses past
* experiences to make decisions.
1 point 97. Which stage of Benner’s Novice to Expert Model is
A. Linea nigra characterized by the ability to prioritize patient care and
B. Braxton Hicks contractions recognize patterns in clinical situations?
C. Detection of fetal heartbeat by Doppler *
D. Abdominal enlargement 1 point
90. Nurse Suzy explains that which of the following signs is A. Novice
not classified as a presumptive sign of pregnancy? B. Advanced Beginner
* C. Competent
1 point D. Expert
A. Amenorrhea 98. Nurse Im Sol is mentoring a new graduate nurse who has
B. Quickening difficulty adapting to unexpected changes in patient
C. Positive urine pregnancy test conditions. At which stage of Benner’s Novice to Expert Model
D. Breast tenderness is the nurse likely to be?
*
1 point D. Official Gazette or in any two (2) newspapers of general
A. Novice circulation in the Philippines
B. Advanced Beginner E. Immediately after being signed by the President
C. Competent Situation: You are Nurse Sung Deok-sun, providing
D. Proficient education to Choi Taek, a 16-year-old with Type 1
99. In Benner’s Novice to Expert Model, which stage is Diabetes Mellitus. Your focus includes understanding the
characterized by an intuitive grasp of situations and a deep etiology of diabetes, assessing self-care skills, managing
understanding of the clinical context? complications, and appropriate emergency interventions.
* 6. Choi Taek, newly admitted, asks about the cause of his
1 point Type 1 diabetes. What process should you describe to him?
A. Advanced Beginner *
B. Competent 1 point
C. Proficient A. The tissues in your body are resistant to the action of
D. Expert insulin, making the glucose levels in your blood increase.
100. Which statement best describes a nurse who is in the B. Damage to your pancreas causes an increase in the
"proficient" stage of Benner’s Novice to Expert Model? amount of glucose that it releases, and there is not enough
insulin to control it.
* C. The amount of glucose that your body makes
1 point overwhelms your pancreas and decreases your production
A. The nurse relies heavily on rules and guidelines to of insulin.
provide care. D. Destruction of special cells in the pancreas causes a
B. The nurse can recognize patterns in patient care but still decrease in insulin production. Glucose levels rise because
needs to follow strict protocols. insulin normally breaks it down.
C. The nurse has an intuitive grasp of situations and can 7. Choi Taek has reported a positive urine ketone test. What is
prioritize care effectively. the most appropriate nurse's response based on this finding?
D. The nurse has limited clinical experience and struggles *
with handling unexpected situations. 1 point
A. The patient should withhold his next scheduled dose of
insulin.
B. The patient should promptly eat some protein and
NP3 carbohydrates.
C. The patient's insulin levels are inadequate.
D. The patient would benefit from a dose of metformin
(Glucophage).
8. Which criterion supports the diagnosis of diabetes and
warrants checking blood glucose levels?
Situation: A newly graduated nurse is preparing to apply *
for licensure in the Philippines. They are studying 1 point
Republic Act 9173 to understand the qualifications, A. Fasting plasma glucose greater than or equal to 126
responsibilities, and ethical standards required for mg/dL
professional practice in the country. B. Random plasma glucose greater than 150 mg/dL
1. What additional qualifications must a nurse possess to C. Fasting plasma glucose greater than 116 mg/dL on 2
occupy the position of chief nurse or director of nursing separate occasions
service as per RA 9173? D. Random plasma glucose greater than 126 mg/dL
* 9. Choi Taek reports abdominal pain, appears weak, and is
1 point displaying Kussmaul respirations at 30 breaths/minute. What
A. A master's degree major in nursing and at least five (5) prescriptions should you implement in his care? (Select all
years of experience in a supervisory or managerial position that apply.)
in nursing 1. Dextrose 50% by intravenous (IV) push
B. A master's degree in any field and at least three (3) 2. Regular insulin by IV infusion
years of experience in nursing service administration 3. Obtain a STAT capillary blood glucose measurement
C. A bachelor's degree in nursing and at least ten (10) 4. Start an IV line and begin an infusion of 0.9% sodium
years of experience in a managerial position chloride
D. A bachelor's degree in nursing and at least five (5) years 5. Draw blood for the prescribed serum potassium level
of experience in general nursing *
2. According to RA 9173, what is the term of office for each
member of the Board of Nursing? 1 point
* A. 1, 3, 4
B. 1, 2, 3, 4
1 point C. 2, 3, 4, 5
A. 2 years D. All of the above
B. 3 years 10. The nurse is reviewing medication orders for Choi Taek.
C. 4 years Which prescription should the nurse clarify with the
D. 5 years healthcare provider (HCP)?
3. According to RA 9173, how many members are there in the
Board of Nursing? *
* 1 point
1 point A. 12 units of subcutaneous detemir insulin daily at 2000
A. 3 members B. 8 units of regular insulin by IV infusion for serum glucose
B. 5 members greater than 300 mg/dL
C. 7 members C. 16 units of subcutaneous lispro insulin daily at 1000
D. 9 members before breakfast
4. Under RA 9173, which of the following is NOT a ground for D. 18 units of NPH insulin PO daily at 0700 for blood
glucose greater than 80 mg/dL
the revocation or suspension of a nurse's certificate of Situation: You are Nurse Sung Deok-sun, providing
registration/professional license? education to Kim Jung Hwan, a 55-year-old recently
* diagnosed with Type 2 Diabetes Mellitus. Your focus
1 point includes understanding the etiology of diabetes,
A. Unprofessional and unethical conduct assessing self-care skills, managing complications, and
B. Gross incompetence or serious ignorance appropriate emergency interventions.
C. Refusal to participate in continuing education 11. Kim Jung Hwan, newly diagnosed with Type 2 diabetes,
D. Malpractice or negligence in the practice of nursing asks about the primary treatment approach. What should you
5. When does RA 9173, the Philippine Nursing Act of 2002, emphasize?
take effect? *
* 1 point
1 point A. “A pancreas transplant will cure my diabetes.”
A. Thirty (30) days after signing by the President B. “I will take my oral antidiabetic agents only when my
B. Fifteen (15) days after approval by Congress blood sugar is high.”
C. Fifteen (15) days after publication in the
C. “I will follow the weight loss plan provided by the 1 point
dietitian.” A. Rapid infusion of hypotonic saline
D. “I will call the diabetes educator whenever I have B. Slow infusion of isotonic saline
questions about my insulin.” C. Rapid infusion of hypertonic saline
12. Kim Jung Hwan, who has been managing his Type 2 D. Administration of diuretics
diabetes with oral medications and lifestyle changes, 18. Cha Eun-jae is prescribed demeclocycline for her SIADH.
presents to the emergency department with a blood glucose Nurse Seo Woo-jin knows that the purpose of this medication
level of 623 mg/dL and confusion following a recent is to:
gastrointestinal virus. What is the priority nursing action? *
* 1 point
1 point A. Increase ADH secretion
A. Administer antihypertensive medications. B. Decrease renal response to ADH
B. Administer sodium bicarbonate intravenously. C. Replace deficient ADH
C. Reverse acidosis by administering insulin. D. Promote water retention
D. Perform fluid and electrolyte replacement. 19. During the follow-up visit, Nurse Seo Woo-jin reviews Cha
13. Kim Jung Hwan is at risk for Hyperglycemic Hyperosmolar Eun-jae’s sodium levels. Which range of serum sodium is
Syndrome (HHS). In which population is HHS most commonly considered critical and requires immediate intervention?
observed? *
* 1 point
1 point A. 130-135 mEq/L
A. Obese patients with no known history of diabetes. B. 125-130 mEq/L
B. Patients with Type 1 diabetes and poor dietary control. C. 115-120 mEq/L
C. Adolescents with Type 2 diabetes and sporadic use of D. 140-145 mEq/L
antihyperglycemics. 20. What is a priority nursing diagnosis for Cha Eun-jae who
D. Middle-aged or older individuals with Type 2 diabetes or has been diagnosed with SIADH?
no known history of diabetes. *
14. Kim Jung Hwan asks how long he can store a vial of 1 point
insulin at room temperature before it goes bad. What is the A. Risk for peripheral neurovascular dysfunction
best response? B. Excess fluid volume
* C. Hypothermia
1 point D. Ineffective airway clearance
A. “If you use the vial within 1 month, it can be kept at room Situation: Nurse Anneliese is assigned to Erika, a
temperature.” 35-year-old teacher diagnosed with diabetes insipidus
B. “If you use the vial within 21 days, it may be kept at room (DI) after a car accident caused head trauma. Erika has
temperature.” been experiencing excessive thirst and urination. Nurse
C. “If you use the vial within 2 weeks, it may be kept at Anneliese needs to manage Erika's symptoms and
room temperature.” provide education on her condition and treatment plan.
D. “If you use the vial within 1 week, it may be kept at room 21. Erika complains of severe thirst. What is the underlying
temperature.” cause of this symptom in diabetes insipidus?
15. Kim Jung Hwan, who has been diagnosed with Type 2 *
diabetes-related peripheral neuropathy, is receiving health 1 point
education. Which of the following recommendations should A. Elevated blood glucose levels
the registered nurse (RN) provide? Select all that apply. B. Increased renal reabsorption of water
1. “Every day, check your feet, especially in the spaces C. Deficient antidiuretic hormone (ADH) production
between your toes, and report any changes in skin right away.” D. High serum calcium levels
2. “Avoid wearing socks to reduce the amount of perspiration 22. Nurse Anneliese notices that Erika's serum sodium levels
and irritation on your skin.” are elevated. What does this indicate in the context of
3. “Before stepping into the tub, use a thermometer to check diabetes insipidus?
the water's temperature.” *
4. “Prevent nail breakage by keeping toenails clipped close to 1 point
the skin.” A. Effective management of fluid balance
5. “Wear well-fitting, closed-toe shoes at all times, even when B. Dehydration due to excessive urine output
indoors.” C. Overhydration from excessive fluid intake
* D. Normal renal function
1 point 23. Erika's urine output is being monitored. Which of the
A. 1, 3, 5 following would indicate effective management of her DI?
B. 2, 4, 5 *
C. 1, 3 1 point
D. All of the above A. Urine output decreases to 1.5 liters per day
Situation: Nurse Seo Woo-jin is managing the care of Cha B. Urine specific gravity increases to 1.030
Eun-jae, a 34-year-old female patient who was admitted C. Serum sodium levels decrease to 125 mEq/L
with severe headache and altered mental status. After D. Erika reports severe headaches
thorough diagnostic testing, Cha Eun-jae has been 24. During a follow-up visit, Erika mentions frequent
diagnosed with Syndrome of Inappropriate Antidiuretic headaches. What potential complication should Nurse
Hormone (SIADH). Nurse Seo Woo-jin needs to monitor Anneliese consider?
her condition closely, manage her symptoms, and *
educate her about the treatment plan. 1 point
16. Cha Eun-jae is diagnosed with SIADH. Which of the A. Hyperglycemia
following clinical manifestations should Nurse Seo Woo-jin B. Hyponatremia
expect to find during the assessment? Select all that apply. C. Hypercalcemia
1. Elevated serum osmolality D. Hypokalemia
2. Elevated urine specific gravity 25. Nurse Anneliese is educating Erika about the signs of
3. Decreased serum osmolality water intoxication. Which symptom should she highlight?
4. Decreased urine output *
5. Decreased serum sodium 1 point
* A. Increased thirst
1 point B. Dry mucous membranes
A. 2, 3, 5 C. Sudden weight loss
B. 1, 4, 5 D. Reduced urine output
C. 2, 3, 4, 5 Situation: Nurse Yoon Seo Jung is assessing a male
D. All of the above patient, Kang Dong Joo, who has recently been
17. Cha Eun-jae’s serum sodium level is critically low at 120 diagnosed with Addison’s disease. The focus is on
mEq/L. Nurse Seo Woo-jin understands that which of the understanding clinical manifestations, appropriate
following treatments is appropriate for severe hyponatremia interventions, and management strategies.
26. Kang Dong Joo, newly diagnosed with Addison’s disease,
in SIADH? exhibits several symptoms. Which of the following clinical
*
signs should Nurse Yoon Seo Jung expect? Select all that A. Mitotane is used to stimulate adrenal hormone
apply. production.
1. Increased body or facial hair B. Mitotane can cause hyperglycemia and should be
2. Purple or red striae on the abdomen monitored closely.
3. Skin color that is tanned in appearance C. Mitotane is used to suppress ACTH production from the
4. Orthostatic hypotension pituitary gland.
5. Anorexia and weight loss D. Mitotane is a form of chemotherapy used to reduce
* cortisol levels by targeting adrenal glands.
1 point 33. Nurse Lee Mi-hyun is reviewing Kim Doo-shik’s medication
A. 3, 4, 5 regimen. Which of the following instructions should she give
B. 1, 3, 5 him regarding corticosteroid therapy?
C. 3, 4 *
D. 1, 4 1 point
27. Kang Dong Joo is experiencing signs of an Addisonian A. Take the medication at bedtime to mimic the natural
crisis. Which of the following should Nurse Yoon Seo Jung cortisol rhythm.
prioritize in the management of this crisis? B. Take the medication in the morning to align with the
* body's natural cortisol secretion.
1 point C. Take the medication with meals to reduce
A. Administer high-dose hydrocortisone intravenously as gastrointestinal irritation.
prescribed. D. Take the medication at noon to manage symptoms
B. Encourage increased oral fluid intake without medical effectively.
supervision. 34. During Kim Doo-shik’s follow-up visit, Nurse Lee Mi-hyun
C. Increase the dose of oral glucocorticoids immediately. reviews his lab results. His serum sodium is elevated, and his
D. Monitor blood glucose levels and provide immediate potassium is low. What should Nurse Lee Mi-hyun infer about
glucose supplementation. these findings?
28. Kang Dong Joo’s Addison’s disease requires ongoing *
management. Which dietary recommendation is most 1 point
appropriate for him? A. These electrolyte imbalances are typical of primary
* adrenal insufficiency.
1 point B. The electrolyte changes are consistent with Cushing’s
A. High protein and carbohydrate diet. syndrome, indicating mineralocorticoid excess.
B. Low-sodium and high-potassium diet. C. These findings suggest acute adrenal crisis.
C. Low-protein and high-fat diet. D. The lab results are indicative of a side effect of
D. High-fiber and low-carbohydrate diet. antidiabetic medication.
29. Kang Dong Joo’s laboratory results show hyperkalemia 35. Nurse Lee Mi-hyun is preparing Kim Doo-shik for a
and hyponatremia. What is the significance of these findings dexamethasone suppression test. What should she explain to
in the context of Addison’s disease? him about this test?
* *
1 point 1 point
A. They indicate an acute exacerbation of heart failure. A. The test involves intravenous administration of
B. They are consistent with the electrolyte imbalances seen dexamethasone and subsequent imaging of the adrenal
in Addison’s disease. glands.
C. They suggest a complication from diabetes mellitus. B. The test involves oral administration of dexamethasone
D. They are a sign of renal failure. at 11 PM, with a plasma cortisol level checked at 8 AM.
30. Which lifestyle adjustment should Nurse Yoon Seo Jung C. The test involves oral administration of dexamethasone,
emphasize for Kang Dong Joo to help manage his Addison’s followed by hourly plasma cortisol levels for 3 hours.
disease effectively? D. The test involves intravenous administration of
* dexamethasone, followed by plasma cortisol measurement
1 point 3 hours later.
A. “Increase your physical exercise to boost overall health.” Situation: Nurse Jeong Gu-won is caring for Do Do-Hee, a
B. “Avoid situations that may lead to physical or emotional 45-year-old female who has been admitted to the hospital
stress.” with a diagnosis of Graves' disease. Do Do-Hee has been
C. “Engage in regular high-intensity workouts to improve experiencing severe symptoms of hyperthyroidism,
endurance.” including agitation, tremors, and high fever. Upon
D. “Limit your fluid intake to prevent fluid overload.” examination, her symptoms suggest the possibility of
Situation: Nurse Lee Mi-hyun is working at a busy thyroid storm. Nurse Jeong Gu-won needs to assess her
regional hospital in the Philippines. She is assigned to condition, implement immediate interventions, and
manage the care of Kim Doo-shik, a local businessman provide patient education about managing her
who has recently been diagnosed with Cushing's hyperthyroidism effectively.
syndrome. Kim Doo-shik has been struggling with the 36. Which clinical manifestation is characteristic of Graves'
symptoms of his condition while trying to maintain his disease and would Nurse Jeong Gu-won expect to observe in
demanding job. Nurse Lee Mi-hyun needs to coordinate Do Do-Hee?
his care, address his concerns about the disease, and 1. Exophthalmos
help him manage his treatment plan effectively while 2. Irregular heart rhythm
considering his lifestyle and the local healthcare 3. Obesity
resources available. 4. Insomnia and anxiety
31. Kim Doo-shik, newly diagnosed with Cushing’s syndrome, 5. Hand tremors
is presenting with specific physical signs. Which of the 6. Increased perspiration
following manifestations should Nurse Lee Mi-hyun expect? *
Select all that apply. 1 point
1. Moon facies A. 1, 2, 4, 6
2. Thin, fragile skin B. 1, 3, 4, 5, 6
3. Hyperpigmentation of the skin C. 1, 3, 4, 5
4. Truncal obesity D. 1, 2, 4, 5, 6
5. Hirsutism 37. When planning Do Do-Hee's diet to manage
* hyperthyroidism, what should Nurse Jeong Gu-won include in
1 point her meal plan?
A. 1, 4 *
B. 1, 2, 4 1 point
C. 1, 2, 4, 5 A. Small, frequent meals, high in protein and calories
D. All of the above B. Large, bland meals
32. Kim Doo-shik has been prescribed mitotane for his C. Clear liquid diet
Cushing’s syndrome. What should Nurse Lee Mi-hyun instruct D. High-fiber and low-fat diet
him about this medication? 38. Which laboratory finding is most indicative of
* hyperthyroidism in Do Do-Hee?
1 point *
1 point considered a cornerstone of CHF management due to its
A. Elevated TSH levels ability to reduce afterload and improve cardiac output?
B. Low T3 and T4 levels *
C. Elevated T3 and T4 levels 1 point
D. Normal T3 and T4 levels A. Angiotensin-converting enzyme (ACE) inhibitors
39. Which symptom should Nurse Jeong Gu-won be most B. Diuretics
vigilant for in managing Do Do-Hee’s thyroid storm? C. Beta-blockers
* D. Calcium channel blockers
1 point 47. Rico presents with orthopnea and dyspnea. Upon lung
A. Elevated appetite and weight gain auscultation, Nurse Claudine hears fine crackles that do not
B. Severe agitation and delirium clear with coughing. What is the most likely cause of these
C. Decreased urinary frequency lung sounds?
D. Increased dryness of the skin *
40. What is the primary goal of treatment for thyroid storm? 1 point
* A. Bronchitis
1 point B. Atelectasis
A. Decrease thyroid hormone levels C. Pulmonary edema
B. Decrease thyroid gland size D. Lung cancer
C. Increase thyroid hormone production 48. Nurse Claudine is explaining the procedure of a stress test
D. Normalize thyroid function tests to Rico. Which of the following best describes how a stress
Situation: Nurse Pia is preparing to administer a unit of test is performed?
packed red blood cells (PRBCs) to Rancho, who has a *
diagnosis of severe anemia. Nurse Pia ensures that all 1 point
necessary preparations are in place before starting the A. The patient is given medication to increase heart rate
transfusion. while lying down.
41. What is Nurse Pia's first action before starting the blood B. The patient exercises on a treadmill or stationary bike
transfusion? while their heart rate, blood pressure, and ECG are
* monitored.
1 point C. The patient undergoes an echocardiogram while at rest.
A. Prime the IV tubing with normal saline. D. The patient receives an injection of a radioactive
B. Check Rancho's identification band and confirm the substance, and images of the heart are taken.
blood type with another nurse. 49. Nurse Claudine is educating Rico on how to manage his
C. Administer pre-medications as prescribed. CHF at home. Which statement by Rico indicates he has
D. Assess Rancho's baseline vital signs. understood the importance of monitoring his condition?
42. After starting the transfusion, Nurse Pia observes that Select all that apply.
Rancho begins to show signs of a transfusion reaction, 1. "I will take my medications as prescribed and not skip
including chills, back pain, and fever. What is the most doses."
appropriate initial action for Nurse Pia? 2. "I’ll check my weight every day at the same time and report
* any significant changes."
1 point 3. "I’ll avoid all physical activity to prevent fatigue."
A. Stop the transfusion immediately and disconnect the 4. "I will limit my fluid intake and avoid high-sodium foods."
blood bag. 5. "I’ll see my healthcare provider only when I feel very sick."
B. Administer acetaminophen and continue monitoring *
Rancho. 1 point
C. Apply a warm compress to Rancho’s back and continue A. 1, 2, 4
the transfusion slowly. B. 1, 2, 5
D. Notify the blood bank to report the reaction. C. 2, 3, 4
43. Nurse Pia is preparing to administer a unit of fresh-frozen D. 3, 4, 5
plasma (FFP) to Rancho. Which of the following is the correct
procedure for administering FFP? 50. Rico reports experiencing fatigue and shortness of breath
* during moderate exertion, such as walking up a flight of
1 point stairs. He has no symptoms at rest. Based on the New York
A. Infuse the FFP over 1 to 2 hours. Heart Association (NYHA) Classification, how would you
B. Use a blood warmer to heat the FFP before infusion. classify Rico's heart failure?
C. Administer the FFP with normal saline through a blood
transfusion pump. A. NYHA Class I
D. Infuse the FFP immediately upon thawing and complete B. NYHA Class II
the infusion within 30 minutes. C. NYHA Class III
44. Rancho’s blood transfusion is nearly complete, and Nurse D. NYHA Class IV
Pia notes that it has been infusing for over 4 hours. What is
the best course of action for Nurse Pia? 51. Nurse Sandiego, a senior nurse, is teaching his students
* about caring for a client immediately following
1 point transsphenoidal hypophysectomy. It would be essential for
A. Continue the transfusion until the bag is empty. him to obtain a prescription for which medication?
B. Reduce the infusion rate and continue the transfusion.
C. Stop the transfusion and dispose of the remaining blood. A. Ondansetron :( prophylactic for surgery,
D. Return the remaining blood to the blood bank. Antiemetics)
45. To ensure the safe and effective administration of the B. Methimazole: (antithyroid) for hypothyroidism
blood product, Nurse Pia needs to use the correct gauge for C. Omeprazole: ( pud peptic ulcers disease)
the IV catheter. What is the appropriate range of gauge sizes D. Methylphenidate :(adhd)
for the IV catheter used in blood transfusion?
* 52. The senior nurse conducts a lecture for nursing students.
1 point It would be appropriate for him to state which of the following
A. 16-gauge to 18-gauge procedures requires a sterile technique? Select all that apply.
B. 18-gauge to 20-gauge
C. 20-gauge to 22-gauge A. Changing the dressing for a central line: (big vein
D. 22-gauge to 24-gauge intrajugular exam)
Situation: Nurse Claudine is managing Rico’s care in the B. Inserting an indwelling urinary catheter:
hospital for congestive heart failure (CHF). Rico’s C. Removing a peripheral vascular access device: ( clean
symptoms are being monitored and adjusted as needed. procedure)
Nurse Claudine needs to ensure that all aspects of his D. Suctioning an endotracheal tube with in-line suction:
care, including medication management, lifestyle clean procedure
changes, and disease monitoring, are optimized. E. Inserting a nasogastric tube (NGT) : clean procedure
46. Nurse Claudine is reviewing Rico's treatment plan for
congestive heart failure. Which medication class is 53. He is now teaching the class on acid-base imbalances. It
would be correct for the class to identify which of the
following would cause respiratory acidosis? Select all that
apply. A. You will need to take daily showers or baths with
chlorhexidine.
A. Aspirin overdose : metabolic acidosis/ hydrogen ion B. It is important to clean common surfaces with warm
B. Pneumothorax: soapy water.
C. Opioid overdose C. You will need to have repeat stool testing to determine if
D. Anxiety: respiratory alkalosis you are still infectious.
E. Renal disease: metabolic acidosis/ bicarbonate D. Check with your primary healthcare provider prior to
taking any medications.
54. He then comes back to the hospital and cares for
assigned clients. He should initially intervene when the client 61. Nurse Beca is reviewing arterial blood gas (ABG) results
reports: for a client. The ABG report shows the following values: pH
7.59 [7.35-7.45], PaCO2 30 mmHg [35-45 mm Hg], HCO-3 24
A. nausea during an infusion of amphotericin: anti fungal mEq/L [22-28 mEq/L], PaO2 85 mmHg [80-100 mm Hg].
medicine Nurse Beca interprets this ABG result as
B. palpitations after receiving rapid-acting insulin. (
70/100dcl) A. Metabolic acidosis
C. drowsiness after receiving fentanyl.: cause drowsiness, B. Respiratory acidosis
opioid, C. Metabolic alkalosis
D. itching in the perineal area while receiving intravenous D. Respiratory alkalosis
dexamethasone.: common expected, not life
threatening,infuse slow 62. Nurse Suzu has attended a staff education program about
indwelling urinary catheter-associated infections (CAUTI).
55. He is now then teaching a group of students a potential Which nursing intervention is most effective in preventing a
cause of metabolic alkalosis. ( Kidney) It would indicate a CAUTI in hospitalized clients?
correct understanding if a student stated which condition
could cause this acid-base imbalance? A. Implementing strict sterile technique during catheter
insertion and maintenance.
A. Hyperventilation: respiratory alkalosis B. Using antibacterial indwelling urinary catheters for all
B. Urinary retention: no acid base clients requiring urinary catheterization.
C. Opioid toxicity: respiratory acidosis C. Limiting the duration of indwelling urinary catheter
D. Excessive vomiting: (all alk alk, out many acid: use and promptly removing them when no longer
alkalosis tira) needed.
D. Administering prophylactic antibiotics to all clients with
Metabolic acidosis: diarrhea indwelling urinary catheters in place.
56. Which of the following interventions is helpful in reducing 63. Marie, the newly hired nurse, is preparing to administer
the effects of GERD? medications to assigned clients. Which prescription should
the nurse clarify with the doctor?
A. Lie down after eating.
B. Wear a girdle. A. Warfarin to a client with an international normalized ratio
C. Elevate the head of the bed on 4-6 inch blocks. 2 (INR) of 1.8 seconds
pillows B. Digoxin to a client with a serum potassium level of
D. Increase fluid intake just before bedtime.: gerd the food 3.1 mEq/L (normal 0.5-2)
should go down C. Enoxaparin to a client with a platelet count of 155,000
mm3( 150-400 mm3 normal platelet)
57. The ER nurse is made aware that a 7-year-old client with D. Lisinopril to a client with a serum creatinine level of 0.6
an avulsion fracture to the left tibia is 20 minutes out. She mg/dL( hypertension, congestive heart failure)
knows to expect which of the following?
A. A fracture that pulls a part of the bone from the Creatinine 0.6 1.2 normal
tendon or ligament.
B. A fracture with which the whole cross-section of the bone 64. The assigned nurse is performing a continuing education
is fractured.: complete fracture seminar. It would be correct for her to identify which factor
C. A fracture that results from an underlying disease or increases the older adult's risk for illness?
disorder, not physical trauma or stressors.: pathologic
fracture: bone cancer: osteoporosis A. Increased lymph tissue ( Dec)
D. A fracture that affects only one side of the bone.: green B. Increased autoimmune responses( rheumatoid
strict arthritis, collagen disorder
C. Increased circulation of lymphocytes( dec
58. You have developed a care plan for a client with a D. Increased T and B cell production ( Dec if Geria)
self-care deficit related to bathing and hygiene. You should
initially plan to: 65. The nurse supervises a student nurse auscultating lung
sounds on a group of clients. Which statement by the student
A. ask the family to assist the client with baths. nurse would require follow-up?
B. provide the client with supervised bed baths.
C. assess the client for their self-care strengths and A. "Wheezes arise from the small airways and usually do
weaknesses. not clear with coughing."
D. provide assistive devices to the client while they bathe. B. "A pleural friction rub causes loud, rough, scratching
Not initially sounds usually during inspiration."
C. "Thick, tenacious secretions that clear with
59. A nurse is caring for an 80-year-old woman with a coughing cause crackles."
long-standing history of asthma. The nurse is preparing to D. "Fluid or secretions in large airways typically cause
give a dose of theophylline( bronchodilator)to the patient. He coarse crackles."
knows that the most critical sign to assess before giving this
dose is: 66. Mario, the med-surg head nurse, is caring for a client with
severe liver cirrhosis who is prescribed albumin infusion. He
A. Temperature: irrelevant understands that the primary purpose of albumin
B. Blood Pressure administration in this client population is to do which of the
C. Urinary Output following?
D. Pulse: if greater than 20 mcg values of blood can
cause cardiac arrhythmia A. Increase platelet counts
B. Correct electrolyte imbalances
60. San Lorenzo ward nurse is providing discharge C. Enhance oncotic pressure
instructions to a client with hepatitis A.( Fecal- oral) Which of D. Stimulate erythropoiesis
the following instructions should she include?
67. He cares for a client who reports dumping syndrome
following gastric bypass surgery. To alleviate the symptoms A. that had a myocardial infarction two days ago and the
of dumping syndrome, he should recommend that the client troponin is elevated. ( 2 WEEEKS expected,)
Select all that apply. B. with infective endocarditis that wants to leave against
medical advice.
A. take a dose of their prescribed proton pump inhibitor C. that has arterial insufficiency and is reporting leg pain
immediately before meals. after walking in the hall.
B. stay upright for 30 minutes following eating. D. recovering from cardiac catheterization that has
C. eat high-fiber foods to decrease late dumping developed atrial fibrillation.
syndrome.
D. increase their intake of simple carbohydrates to prevent 75. Which nursing intervention would be his priority for a
spikes in blood sugar. patient receiving 3% saline maintenance fluids?
E. eat five to six small meals a day to avoid overloading
the stomach. A. Monitor serum HCO3-
B. Monitor urine sodium
68. Mario is now caring for a client requiring intubation via C. Assess blood pressure( fluid volume overload)
endotracheal tube (ETT). He anticipates that the doctor will D. Collect 24-hour urine output
prescribe which medication immediately prior to intubation?
76. He is helping the nursing aid pass meal trays. When
A. propofol( sedation) providing a meal tray for a client diagnosed with
B. baclofen( multiple sceloris, back pain) pheochromocytoma, which dietary item should he remove?
C. phenytoin( anti epilepsy
D. haloperidol( antipsychotics schizophrenia, delirium) A. Macaroni and cheddar cheese
B. Watermelon slices
69. He is assessing a client receiving mechanical ventilation. C. Caffeine free cola
Which clinical data is most important to review before D. Baked chicken
weaning the client off the ventilator?
77. The PALMER queen is reviewing leadership and
A. Chest x-ray management concepts with a student nurse. Which of the
B. Sputum culture following statements by the student nurse would require
C. Lung sounds follow-up?
D. Arterial blood gas (ABG)
A. “The Laissez-faire leadership style is a passive
70. He observes the newly hired registered nurse prepare to leadership approach.”
administer neutral protamine hagedorn (NPH) insulin to a B. “A registered nurse may delegate accountability to a
client. Which action by the newly hired nurse requires nursing aide”
follow-up? The newly hired nurse: C. “The rights of delegation include task, circumstance,
person, direction, supervision.”
A. asks the client which site the insulin was last injected. D. “The nurse practice act defines roles and responsibilities
B. checks the client's blood glucose levels prior to of nursing professionals.”
administering the insulin injection.
C. shakes the insulin vial before withdrawing insulin. ( 78. Nurse Gloria reviews a client's medical record taking
Gently shake prescribed isoniazid for pulmonary tuberculosis. Which
D. reminds the client to report symptoms of clammy skin laboratory data is most important to monitor?
and disorientation.
A. PT and PTT
71. He evaluates a student's ability to appropriately apply B. CBC
personal protective equipment (PPE). It would indicate C. BUN
effective teaching if the student dons PPE in which order? D. Liver enzymes
Place the steps in the appropriate order.
1. Gown 79. She is teaching a group of students about fluids and
2. Gloves electrolytes. It would be correct for the nurse to state that the
3. Goggles role of calcium is to Select all that apply.
4. Mask
A. promote blood clotting.
A. 1234 B. increase bone density.
B. 4321 C. promote healthy dentition.
C. 1432 D. regulate fluid balance.
D. 3421 E. maintain neuromuscular health.
80. She cares for a client with a hyphema. She should plan to
72. His student is developing a care plan for a client with take which action?
Bell's palsy. Which problem should the student prioritize in the
care plan? A. Shield the affected eye.( And raise bed 30 degree)
B. Place the client supine.
A. Risk for infection C. Apply a cold compress to the eye.
B. Risk for disturbed sensory perception: physiologic D. Request a prescription for aspirin.
C. Risk for disturbed body image
D. Risk for ineffective tissue perfusion 81. Her patient is seeking guidance on secondary prevention
strategies to prevent cancer. Which strategies would be most
73. He read his client’s chart and saw a prescribed 3 g/hr of appropriate for her to include in the client's educational plan?
intravenous magnesium sulfate via continuous infusion. The Select all that apply.
label reads 20 g of magnesium sulfate in 1000 mL of normal
saline. How many mL/hr should he set the pump to deliver the A. Eliminate alcohol intake
prescribed dose? B. Pap smears
C. Rehabilitation programs
A. 152 D. Colonoscopies
B. 148 E. Regular cancer screenings
C. 150
D. 154 82. She is caring for a client brought in after being found
walking around a neighborhood without shoes, confused and
D disoriented. She should initially:
S
V A. obtain vital signs.
B. perform a mental status exam.
74. He has become aware of the following client situations. C. attempt to locate the client's family.
He should first follow up with which client? A client: D. request an order for a psychiatry consultation.
83. Which of the following should she include when teaching A. establish continuous cardiac monitoring.
a 65-year-old male client with COPD about exercise? Select B. obtain the serum peak level prior to infusion.( Troponin
all that apply. level)
C. initiate continuous electroencephalography (EEG)
A. Instruct the client to avoid sudden position changes monitoring.
that may cause dizziness. D. insert an indwelling urinary catheter.
B. Recommend that the client restrict fluid until after
exercising is finished. 92. Anna, the newly hired nurse, cares for a client with venous
C. Instruct the client to push a little further beyond his thromboembolism (VTE) prescribed a heparin infusion and
fatigue with each session. warfarin. Which statement, if made by Anna, requires
D. Instruct the client to avoid exercising in very cold or follow-up?
very hot temperatures.
E. Encourage the client to exercise if he feels ill or weak. A. "The infusion should be discontinued once the INR
F. Recommend consuming a high-carb, low protein diet. is 4.0."
B. "Protamine sulfate should be available if the aPTT gets
84. She attends to a client with shortness of breath, bilateral too high."
lung crackles, weak pulses, and frothy pink sputum. Which of C. "Hypotension and tachycardia may suggest an internal
the following orders should she question for this client? hemorrhage."
D. "Both medications increase the client's risk of bleeding.
A. Supplemental oxygen via nasal cannula or mask "
B. Losartan ( blocks angiotensin 2,) 93. Anna is caring for a client with thick, tenacious mucus due
C. Fowler’s position to cystic fibrosis who is prescribed acetylcysteine. What is the
D. Diltiazem( negative inotropic effects) primary therapeutic effect of acetylcysteine in this situation?
85. She is now caring for a client who has pulmonary A. Reduction of airway inflammation by inhibiting
tuberculosis (TB). Which infection control measure should leukotriene synthesis.
she implement? B. Promotion of bronchodilation by blocking muscarinic
receptors.
A. Restrict visitors who are pregnant C. Enhancement of mucociliary clearance by reducing
B. Remove any portable fans in the room mucus viscosity.
C. Wear a dosimeter badge during client care D. Prevention of oxidative damage to lung tissue by
D. Place the client further away from the nursing station scavenging free radicals.
86. She is caring for a client with atrial fibrillation. Which of 94. She is caring for a client scheduled for a bronchoscopy.
the following client findings requires immediate follow-up by The client asks what to expect during the recovery period.
her? Which response her is most appropriate?
A. Irregular QRS complexes on telemetry reading A. "You may experience some sore throat and
B. Irregular peripheral pulse coughing for a few days after the procedure."
C. Reports of intermittent palpitations B. "You'll need to avoid eating or drinking anything for at
D. Blurred vision least 24 hours following the procedure."
C. "You'll be discharged immediately after the procedure
87. She is caring for a client with a kidney injury with a serum and can resume your normal activities."
potassium level of 6.1 mEq/L (mmol/L). Which of the D. "Expect to have difficulty breathing for several days after
following actions is her priority? the procedure."
A. Obtain a prescription for a diuretic to increase urine 95. She is caring for a female client who is receiving
output prescribed isotretinoin. Which laboratory data is essential
B. Check the client's sodium level prior to the initiation of this therapy?
C. Place the client on a cardiac monitor
D. Encourage oral fluid intake A. Pregnancy test ( teratogenic)
B. C-Reactive Protein
88. Nurse Lindo is preparing a client for angiography using C. BUN and Creatinine
contrast media. Which of the following side effects should D. Prothrombin time (PT)
Lindo educate the client is not an expected side effect?
A. Sudden nausea
B. A headache lasting several days
C. A feeling of facial flushing 96. She receives a client who has just returned from a circular
D. Sudden urge to urinate skin punch biopsy to confirm a skin cancer diagnosis. She
89. He is teaching at an interdisciplinary conference focused should prioritize observing the site for:
on age-related changes. Which of the following are expected
in the older adult? Select all that apply. A. Dehiscence
B. Infection
A. Fatty tissue is redistributed C. Bleeding
B. Skin becomes less elastic D. Swelling
C. Cardiac output increases
D. Muscle mass increases 97. She is reviewing newly prescribed medications for
E. Hormone production increases assigned clients. Which of the following prescribed
F. Visual and hearing acuity diminishes medications should she question?
90. He is caring for a child with rickets.( Vit d def) Which A. Levothyroxine for a client with a myxedema coma
dietary intervention should he anticipate in the client's plan of B. Hydrochlorothiazide for a client with
care? hyperparathyroidism(
C. Hydrocortisone for a client with adrenal insufficiency
A. Increasing dietary intake of vitamin D-rich foods. D. Regular insulin for a client with diabetic ketoacidosis
B. Limiting dietary intake of calcium-rich foods.( Partner vit
d) 98. She is caring for a client who sustained a cervical spinal
C. Encouraging consumption of caffeine-containing cord injury. Which priority vital sign should she obtain?
beverages.
D. Promoting a high-protein diet. A. Respiratory rate
B. Blood Pressure
91. He is preparing to administer prescribed intravenous C. Pulse
phenytoin to a client with epilepsy. Prior to starting the D. Temperature
infusion, he should:
99. She is providing discharge instructions to a client D. is being treated for diabetes insipidus, and the family
prescribed phenazopyridine.( Pri: fire) Which of the following member reports the client has developed confusion.
instructions should she include?
6. Nurse Sarah is assessing a client receiving peritoneal
A. Discontinue this medication if urinary discoloration dialysis. Which laboratory result should immediately be
occurs reported to the doctor?
B. Take this medication on an empty stomach
C. This medication may increase the amount of urine you A. WBC 19,000 mm3
produce B. Hemoglobin 9 g/dL
D. Urine may have a reddish or orange coloration after C. Calcium 8.6 mg/dL
taking this medication D. Serum pH 7.33
100. She cares for a client who is at risk of developing
pressure ulcers. Which of the following would she recognize
as accurate statements regarding pressure ulcers? Select all 7. Nurse Sarah is assessing a client with systemic lupus
that apply. erythematosus (SLE). Which laboratory data is essential for
her to monitor to determine if the client is experiencing a
A. In a stage II pressure ulcer, part of the dermis and complication?
epidermis are lost.
B. In a stage I pressure ulcer, there is a loss of integrity of A. urine analysis
the epidermis only.
C. In a stage III pressure ulcer, there is a deep tissue injury B. hemoglobin A1C (HbA1C)
that can expose fat. C. thyroid-stimulating hormone (TSH)
D. In a stage IV pressure ulcer, the base of the wound is D. ammonia
covered by eschar. 8. She has received the following prescriptions for newly
E. Stage III involves extensive tissue damage and can lead admitted clients. Which prescription should she administer
to bone and muscle involvement. first?
A. Aspirin to a client experiencing an acute myocardial
infarction
B. Lisinopril to a client with essential hypertension
C. Risperidone to a client with schizophrenia
D. Levodopa-carbidopa to a client with Parkinson’s disease
NP4 9. The occupational health nurse is conducting an in-service
on reducing back injuries. Which of the following statements,
https://docs.google.com/forms/d/e/1FAIpQLSf0a2yQg5GvFC9 if made by a participant, would indicate a correct
qhowQk4ssit_hlE193kVJtvr86gBDwW5Qsg/viewscore?views understanding of the conference?
core=AE0zAgBCaeU5b41muL6CozObUcMXqxcdj1l6E-SYvg
W7trGn0niDTpVUWfMzj0kU1iXZymA A. "I should keep my legs straight while lifting."
B. "Heavy objects should be held away from my body."
C. "I shouldn't twist while lifting an object."
1. Nurse Dagul is assessing a client with drooping of their left D. "I should keep a narrow base of support."
eyelid. He documents this finding as 10. The nurse is assessing a patient with syndrome of
inappropriate antidiuretic hormone (SIADH). Which of the
A. mydriasis. following laboratory tests require careful monitoring?
B. ptosis.
C. presbyopia. A. Potassium
D. hyphema. B. Sodium
C. Glucose
2. He is now caring for a client following a stroke. The client D. Magnesium
can understand instructions but is unable to express himself
verbally. He anticipates that this stroke has impacted which SIADH :has many ADH
lobe of the client's brain?
11. John is caring for a client with heart failure. Which
A. Frontal lobe medication should he clarify with the doctor?
B. Parietal lobe
C. Temporal lobe A. lisinopril
D. Occipital lobe B. Prednisone STEROID, SODIUM RETENTION fluid volume
3. He then receives a client with myocardial infarction (MI), C. Hydralazine
who is receiving tissue plasminogen activator (tPA), he should D. carvedilol
plan to prioritize which of the following?
12. John is providing asthma education to a teen that has just
A. Observe for neurological changes been diagnosed with asthma. Which of the following
B. Monitor for any signs of renal failure statements by the client would indicate a need for further
C. Observe for signs of bleeding teaching? Select all that apply.
D. Check the client’s food diary
A. “When I am having an asthma attack, I should call 911
4. Dags is now planning to perform a dressing change for a first.”
client with a stage three pressure injury. He should initially B. “When I am having an asthma attack, my airway is
perform which action? constricting, and it can become dangerous.”
C. “I should try to identify what causes me to have an
A. Gather all the necessary equipment asthma attack and avoid those activities.”
B. Use non-sterile gloves to remove the old dressing D. “I’ve really been wanting to get a dog and my asthma
C. Documenting the characteristics of the wound will not stop me.”
D. Administer prescribed oral pain medication E. "I should inhale quickly when using my albuterol inhaler."
F. "I will take theophylline prior to exercising."
5. He cares for assigned clients. He should initially follow-up
on the client who:
Fish safe for asthma
A. has chronic back, neck, and shoulder pain and is crying. Theophylline: asthma
B. is being treated for pneumonia and reports a persistent
cough with thick, yellow mucus. 13. John is teaching a client with hypercalcemia appropriate
C. had a colostomy placed three hours ago and has an dietary measures. Which food selections by the client would
edematous stoma with scant bloody drainage. require follow-up by him? Select all that apply.
22. Yulois teaching a client about newly prescribed
A. Broccoli tamsulosin. Which of the following statements should the
B. 2% milk nurse include?
C. whole wheat pasta
D. Bananas A. “This medication may turn your urine reddish/orange.”
E. seafood B. “You will urinate more often with this medication.”
C. “Change positions slowly while you take this
medication.”
14. John plans to care for a client experiencing a D. “Avoid calcium-containing foods while on this
hyperglycemic-hyperosmolar state (HHS). He should medication.”
anticipate which prescriptions from the doctor?
23. Yulo is teaching a client about newly prescribed
A. 0.9% saline infusion tamsulosin. Which of the following statements should he
B. Glargine insulin include?
C. Sodium polystyrene
D. Sodium bicarbonate A. “This medication may turn your urine reddish/orange.”
B. “You will urinate more often with this medication.”
15. John is teaching a client who has hypertension about the C. “Change positions slowly while you take this
newly prescribed medication, furosemide. Which of the medication.”
following should he include in the teaching? D. “Avoid calcium-containing foods while on this
medication.”
A. Limit intake of bananas, cantaloupe, and potatoes.
B. Avoid taking the medication with grapefruit juice. 24. Yulo is caring for a client in labor who is positive for the
C. Take this medication in the early part of the day. human immunodeficiency virus (HIV). He should obtain a
D. A nagging cough can occur as a side effect of the prescription for which medication?
medication.
A. valacyclovir ( herpes zoster)
16. Carlos cares for assigned clients. Which of the following B. zidovudine
clients should he identify is at the highest risk for falling? C. amphotericin b( anti fungal
D. metronidazole
A. 88-year-old admitted with a chest tube secondary to
pneumothorax and has a history of dementia 25. Angelika, the nurse manager, has received a complaint
B. 44-year-old admitted with heart failure, has a peripheral from a client's family member. She should take which initial
IV, and receiving IV furosemide action?
C. 33-year-old admitted with cholecystitis, has a peripheral
IV, and is receiving IV hydromorphone A. Tell the night charge nurse to ensure the night shift nurse
D. 28-year-old admitted with bacteremia is receiving performs the assigned duties appropriately
intravenous fluids via central line and is diaphoretic B. Speak with the night shift nurse regarding the complaint
and discuss the care provided
17. Carlos is preparing a staff in-service regarding C. Assess the complaint and clarify the details with the
sensorineural hearing loss. It would be appropriate for him to family member and client
identify which factors cause this type of hearing loss? Select D. Take note of the complaint and place it in the applicable
all that apply. employee's file
A. Presbycusis 26. Angelika is positioning a client following a liver biopsy.
B. Ototoxic substance Which position is best suited for this client?
C. Foreign body
D. Exposure to loud noise A. On the left side with a pillow under the ribs.
E. Edema B. Supine with a pillow under the client's knees.
C. Face down with a pillow under the hips.
18. Carlos is preparing to insert an indwelling urinary catheter. D. On the right side with a pillow under the biopsy site.
Which action may be delegated to the nursing aide?
27. Nurse Angelika is conducting a teaching session with the
A. Set up the sterile field parents of a child newly diagnosed with asthma. The priority
B. Palpate the bladder for distention topic for her to cover is:
C. Explain the procedure to the client
D. Place the urinary catheter kit at the bedside A. how to use a peak flow meter.
19. The unit charge nurse, Carlos, knows which of the B. signs and symptoms of an asthma attack.
following are internal disasters? Select all that apply. C. the need to stay current with immunizations.
D. community resources available for asthma management.
A. A loss of electrical power to the facility
B. The sudden cessation of internal communication 28. Which of the following statements should she use to best
C. toxic chemical spill in the lobby of the facility describe a very low-calorie diet?
D. A serious life-threatening medication error
E. Train crash in neighboring town A. “This is a long-term treatment measure that assists
obese people who can’t lose weight.”
20. Which is an intrinsic risk factor that increases the risk of B. “A very low-calorie diet contains very little protein.”
patients developing pressure ulcers? C. “This diet can be used only when there is close medical
supervision.”
A. Shearing D. “This diet consists of solid food that is pureed to
B. Friction facilitate digestion and absorption.”
C. Impaired tissue perfusion
D. Pressure 29. Angelika is now caring for a child with nocturnal enuresis
that was not responsive to non-pharmacological
21. Yulo is caring for a client with suspected retinal modifications. She anticipates the doctor to provide which
detachment. Which of the following manifestations would not medication?
be consistent with a diagnosis of retinal detachment?
A. urecholine ( pns)
A. Seeing “floaters” in the field of vision B. desmopressin
B. A sense of having a curtain drawn over the eyes C. Prazosin
C. Flashes of light D. finasteride
D. Intense pain in the affected eye
30. Angelika is caring for a client at risk of developing tumor 38. Nurse Hidilyn is caring for a client who is postoperative
lysis syndrome (TLS). Which of the following prescriptions and at risk for venous thromboembolism (VTE). Which of the
would prevent TLS? following medications would prevent this complication?
A. Intravenous (IV) hydration A. Enoxaparin
B. Broad-spectrum antibiotics B. Verapamil
C. High dose corticosteroids C. Tranexamic acid
D. Histamine-2 receptor antagonists D. Aspirin
39. Hidilyn is caring for a client who is recovering from carotid
31. Nurse Chloe cares for assigned clients. She should endarterectomy. Which assessment would she recognize as a
immediately follow-up on the client who: sign that the client experienced hypoglossal nerve injury?
A. is being treated for pneumonia and develops A. Tongue deviation
restlessness. B. Inspiratory stridor
B. is receiving intravenous fluids for influenza and C. Tracheal deviation
dehydration and reports a headache. D. Severe headache
C. has a chest drainage system in place for a hemothorax
and tidaling is present in the water seal chamber. 40. The RN cares for a client who sustained a stroke
D. is being treated for pleurisy and is experiencing impacting the occipital lobe. Which of the following
inspiratory chest pressure assessment findings would support this diagnosis?
32. Chloe educates a family with a child with phenylketonuria A. homonymous hemianopia
(PKU). It would be appropriate for her to recommend that the B. impaired proprioception
child avoid which foods? Select all that apply. C. expressive aphasia
D. impulsivity
A. pork tenderloin
B. grapefruit slices 41. Obiena is educating a client newly diagnosed with
C. cheese omelets hypertension about sodium and its role in blood pressure.
D. steamed carrots Which of the following statements about sodium are true?
E. ice cream Select all that apply.
33. Chloe in the mental health unit is preparing to establish a A. Sodium cannot be completely eliminated from the diet.
new group therapy session. Which client would be most B. There is no sodium in fresh fruits and vegetables.
appropriate for group therapy? A client: C. Canned vegetables should be avoided.
D. The body needs some sodium as it plays an important
A. in the acute phase of schizophrenia. role in water balance.
B. with bipolar I disorder experiencing a mixed episode of E. Reduce daily sodium intake to 2,000 mg
depression and mania.
C. with post-traumatic stress disorder having difficulty 42. Obiena is caring for a client who is 75 years old with
sleeping because of night terrors. coronary artery disease and chronic obstructive pulmonary
D. experiencing delirium tremens associated with alcohol disease (COPD). Based on the client's age and medical
withdrawal. problems, which immunization should he recommend for this
client?
34. Chloe has provided discharge instructions to a client who
was prescribed niacin.( Vasodilate)Which of the following A. hepatitis A vaccine
statements, if made by the client, would indicate effective B. hepatitis b vaccine
teaching? C. pneumococcal vaccine
D. live attenuated influenza vaccine
A. "This medication may increase my blood pressure."
B. "I may experience flushing while taking this medication." 43. He is discharging a client following knee arthroplasty.
C. "This medication may raise my total cholesterol." Which of the following information should the nurse include in
D. "I may feel fatigued and tired after taking this discharge teaching?
medication."
A. “After this procedure, you will use a wheelchair to get
35. Chloe, RN, is caring for a patient who is receiving enteral around.”
feedings. What is the most effective method to verify initial B. “You will need to resume your prescribed
tube placement is correct? anticoagulants.”
A. Obtain chest and abdominal x-rays. C. “Placing a pillow under your knee will help with the pain.”
B. Aspirate the contents to assess pH range. D. “You may ice the site for one hour at a time.”
C. Mark tubing at the exit site and record the length of 44. RN Obiena is caring for a client immediately following
tubing that protrudes. scleral buckling surgery for a retinal detachment of the right
D. Insert 20-30 mL of air into the tube while auscultating the eye. Which of the following actions would be appropriate for
epigastrium. him to take? Select all that apply.
*
36. Hidilyn is observing a student nurse feed a client requiring 1 point
aspiration precautions. She should intervene if the student: A. place the client in a prone position.
B. approach the client from the left side.
A. asks the client to remain sitting upright for at least 30 to C. instruct the client to perform deep breathing and
60 minutes after a meal. coughing exercises.
B. reminds the client to tilt their head backward when D. instruct client to avoid bending down.
eating and drinking. E. orientate the client to the environment.
C. avoids mixing foods of different textures in the same F. obtain a prescription for a stool softener.
mouthful.
D. places salt and pepper on the client's food at their 45. Obiena inserts an indwelling urinary catheter for a female
request. patient prior to surgery. Which of the following would require
immediate intervention by him?
37. Hidilyn is caring for a client who is of the Islamic faith, *
who has expired. She should take which appropriate action? 1 point
A. The patient states she feels the need to urinate.
A. Prepare the client for cremation B. Patient reports a pinching sensation as the catheter is
B. Position the client facing Mecca advanced.
C. Keep the client's eyelids open C. The student nurse notes resistance when inflating the
D. Keep the client uncovered balloon.
D. The student separates the labia majora and labia minora 53. Elle cares for a client who sustained full-thickness thermal
with non-dominant hand. burns to 30% of their total body surface area (TBSA). Which of
46. When performing a transcultural assessment, the nurse the following initial laboratory values would be expected?
must: *
* 1 point
1 point A. Potassium 5.6 mEq/L
A. Determine which questions to ask the client based on B. Hematocrit 30%
the potential health effects of culture-based practices. C. BUN 14 mg/dL
B. Wait until the nurse-patient relationship is established D. Glucose 89 mg/dL
before asking questions. 54. She has attended a staff education program about
C. Ask all questions for completeness of the assessment. incident reporting. It will indicate effective understanding if
D. Include all questions as part of an admitting she states that the primary purpose of incident reporting is to:
assessment. *
47. Nurse Simone supervises a student nurse preparing a 1 point
client for a magnetic resonance imaging (MRI) test. Which of A. implement corrective measures needed to prevent
the following actions by the student nurse would require recurrence.
follow-up by her? The student: B. collect data about errors and compare it to different time
* periods.
1 point C. communicate the error(s) to other departments within
A. asks the client if they have claustrophobia. the facility.
B. instructs the client to apply earplugs before the exam. D. notify the individual involved of the deviation from the
C. moves the nitroglycerin patch from the torso to the standard of care.
back. 55. RN Elle is planning to care for a client with a newly
D. tells the client that they will not have any exposure to diagnosed fractured pelvis. Which action would lessen the
radiation. risk of fat embolism syndrome (FES)?
48. Simone is performing a physical assessment on an adult *
client. She should assess for tactile fremitus by: 1 point
* A. Request a prescription for enoxaparin.
1 point B. Alternate with the application of ice and heat.
A. placing the thumbs on the client's spine at the level of C. Educate the client on pelvic immobilization.
the ninth ribs. D. Encourage passive range of motion of the lower legs.
B. asking the client to breathe slowly and deeply through an 56. It will indicate effective understanding if the nurse
open mouth while auscultating lung sounds. classifies which immunization as live-attenuated?
C. asking the client to say “ninety-nine” while palpating the *
intercoastal spaces beginning at the lung apex. 1 point
D. tapping the chest over the distal interphalangeal joint A. pneumococcal
with the middle finger of the opposite hand. B. diphtheria, tetanus, and pertussis (DTaP)
49. Simone prepares a client for a positron emission C. varicella
tomography (PET) scan. Which laboratory data is necessary D. hepatitis B
to obtain before this test? 57. A client is scheduled to undergo a computed tomography
* scan with iodine-based contrast dye. Which of the following
1 point medications may cause interaction and should be withheld
A. Urine specific gravity for 24 hours before the procedure?
B. Liver function tests *
C. Blood glucose 1 point
D. Creatinine kinase A. Labetolol
50. She is evaluating a patient’s response to peripheral pain. B. Metformin
Which technique should she use to perform this evaluation? C. Levodopa
* D. Ondansetron
1 point 58. Emmett, RN, is caring for a client newly admitted to the
A. Pressure on the patient’s mid-back mental health unit with bulimia nervosa. Which client
B. Sternal rub statement requires immediate follow-up?
C. Squeezing the sternocleidomastoid muscle *
D. Pressing on the patient’s nail bed 1 point
A. “These sores in my mouth hurt."
B. “When can I weigh myself?”
C. “I hate my life and wish it was over.”
D. “I feel really dizzy right now.”
51. Nurse Elle is administering IV magnesium to a client with 59. RN Emmett is caring for a client who has been taken to
a magnesium level of 1.5 mEq/L. You check on them halfway the emergency department after a severe car accident. They
through the infusion, and they report that their face feels require immediate life-saving surgery; however, they are
flushed. What is your priority nursing intervention? unconscious and unable to consent to the operation. Which of
* the following is the best course of action?
1 point *
A. Slow down the infusion rate. 1 point
B. Notify the primary healthcare provider (PHCP). A. Ask a friend who was with the client to sign the consent
C. Reassess the client when the infusion finishes. form.
D. Stop the infusion. B. Attempt calling a family member to obtain consent.
52. Elle supervises a nursing student administering a purified C. Call the on-staff nursing supervisor and request a court
protein derivative (PPD) skin test. Which action by the student order for the surgery.
requires follow-up by Elle? D. Immediately transport the client to the operating
* department without obtaining consent.
1 point
A. Inserts the needle, bevel up at a 15-degree angle 60. Emmett supervises a new graduate caring for a child
B. Instructs the client that the test will be read in 48-72 immediately following surgery to correct an imperforate anus
hours (anorectal malformation). Which action by the new graduate
C. Selects a site 3 to 4 finger widths below the antecubital requires follow-up? The new graduate:
space *
D. Administers the test using a 20-gauge needle, 2 inches 1 point
long A. positions the child side-lying prone position with the hips
elevated.
B. obtains a rectal temperature.
C. auscultates bowel sounds in all four abdominal 68. The ABCDEs of melanoma identification include which of
quadrants. the following? Select all that apply.
D. secures the indwelling urinary catheter bag to the frame
of the bed. *
1 point
61. Nurse Vivian is caring for a client who is receiving A. Asymmetry: one half does not match the other half
prescribed metoclopramide for gastroparesis. Which of the B. Birthmark: cafe au lait spot that does not fade
following findings require immediate notification to the C. Color: pigmentation is not uniform
doctor? D. Diameter: greater than 6 mm
* E. Evolving: any change in size, shape, color, elevation, or
1 point any new symptom such as bleeding, itching, or crusting
A. Muscle rigidity of the neck
B. Hyperactive bowel sounds 69. Warner in a community-based setting is teaching clients
C. Frequent diarrhea strategies for preventing cardiovascular disease. Which of the
D. Abdominal distention following interventions should he include? Select all that
62. Vivian is teaching a group of clients about varicose veins apply.
and home care management. Which of the following should *
Vivian include in the teaching session? 1 point
* A. Encouraging regular physical exercise
1 point B. Promoting a diet high in saturated fats
A. When you are sitting, keep your legs lower than your C. Advising smoking cessation
heart D. Recommend no more than 5 alcoholic drinks a day
B. Wear compression stockings during the day E. Educating about the importance of regular blood
C. Participate in activities that have you stand for long pressure monitoring
periods F. Instructing on the importance of regular cholesterol
D. Take a low-dose aspirin to prevent the development of screenings.
new varicose veins ( dvt)
70. Warner observes a client connected to a chest drainage
63. Vivian is assessing a client with Paget’s disease. Which of system ambulate to the bathroom. The client's chest tube
the following would be an expected finding? becomes disconnected from the client. Warner should take
* which action?
1 point *
A. Bone deformities 1 point
B. Berry aneurysm A. Reinsert the tube after instructing the client to perform
C. Heberden’s nodes the Valsalva maneuver.
D. Janeway lesions B. Hold direct pressure over the insertion site with a
transparent dressing.
64. She is caring for a client who is admitted to the C. Place a sterile dressing taped on three sides over the
emergency department with suspected rhabdomyolysis. insertion site.
Which of the following interventions would she anticipate D. Place the client back in bed in a low Fowler's position
implementing initially for this client? with the knees bent.
*
1 point 71. Warner is assisting in discharging a client who was
A. Administering intravenous fluids treated for digoxin toxicity. While reinforcing teaching on ways
B. Placing the client on a low-potassium diet to prevent future toxicity, he should recommend the client
C. Administering corticosteroids have an adequate dietary intake of which electrolyte?
D. Initiating prophylactic antibiotic therapy *
1 point
65. Vivian is teaching a client with Parkinson's disease about A. chloride
dietary considerations. She understands that this client is at B. potassium
highest risk for: C. phosphorus
* D. sodium
1 point
A. constipation and drooling. 72. Warner performs triage. Which client should he prioritize
B. drooling and a loss of appetite. care for? A client with:
C. loose stools and choking. *
D. dysphagia and aspiration. 1 point
A. hemophilia reporting knee and ankle stiffness with
66. Student nurse Warner is preparing a client for a dizziness.
bronchoscopy procedure. Which pre-procedure intervention B. chronic obstructive pulmonary disease (COPD) reporting
should he prioritize to ensure the client's safety? a productive cough.
* C. chronic pericarditis reporting intermittent chest pain
1 point during inspiration.
A. Administering benzodiazepines and opioids for sedation. D. pain over the cheek radiating to the teeth, tenderness to
B. Ensuring the client has been nothing by mouth (NPO) percussion over the sinuses.
for 4-8 hours before the procedure.
C. Monitoring for bleeding and infection following the 73. Warner is reviewing septic shock with a group of nursing
procedure. students. It would be correct if the student identifies which
D. Verifying that the bronchoscope is properly sterilized causes increased lactate production?
before use. *
67. Warner teaches a group of students about phenytoin. 1 point
Which of the following statements would indicate A. Decreased anaerobic glycolysis
understanding? B. Elevated serum bicarbonate levels
* C. Improved tissue oxygenation
1 point D. Inadequate tissue perfusion
A. "Phenytoin is a selective serotonin reuptake inhibitor
(SSRI) commonly used for epilepsy." 74. Warner is caring for a client with Raynaud’s disease who
B. "The therapeutic range of phenytoin is 10-20 mcg/mL, has just been prescribed ephedrine. What is Warner’s most
and levels above this range can lead to toxicity." appropriate action?
C. "Phenytoin is contraindicated in clients with Parkinson's
disease due to potential exacerbation of symptoms." A. Provide dietary instructions to the client.
D. "Phenytoin is primarily metabolized by the liver and has B. Question and discuss the prescription with the
no significant drug interactions." physician.
C. Instruct the client regarding adverse effects.
D. Administer the medication initially to the client. A. occupational therapy.
B. speech therapy.
75. He is conducting health screenings for hypothyroidism at C. smoking cessation.
the community health fair. Which client is at the highest risk D. group psychotherapy.
for this condition? A client who is:
* 83. Callahan assembles a team of nurses to care for a client
1 point in cardiac arrest. He is assigning various roles to each nurse
A. is underweight, anxious, has a rapid pulse and reports and is demonstrating which management style?
persistent diarrhea.
B. is overweight, reports perspiration while playing sports, A. Authoritative
and reports feeling cold all the time. B. Situational
C. is obese, has high blood pressure, and has frequent C. Democratic
reports of thirst. D. Laissez-faire
D. is obese, has periorbital edema, and reports a decrease
in motivation. 84. He supervises a student nurse assisting a client with a
left-sided weakness in performing activities of daily living.
76. Head nurse Windham is preparing to administer an enema Which action by the student nurse requires him to intervene?
to a client. Prior to administering this medication, she should The student nurse:
position this client:
* A. puts the client's affected (weaker) arm in the shirt's
1 point sleeve first.
A. trendelenburg's position. B. places shoes with velcro straps on the client's feet.
B. semi-Fowler’s position. C. places the wheelchair as close to the bed as possible on
C. left lateral position. the client's affected (weaker) side.
D. right lateral with the head of the bed lowered. D. places the hairbrush in the client's unaffected (stronger)
hand.
77. Head nurse Windham is conducting a health screening at
a local health fair. Which of the following should she 85. Due to a recent flood, the only staff that were able to make
recognize as risk factors for developing colorectal cancer? it to work were two nursing assistants and one licensed
Select all that apply. practical nurse with the nurse manager. Knowing the different
* nursing delivery systems, which system should the nurse
1 point manager implement to care for the 20 clients admitted in their
A. Ulcerative colitis ward?
B. Body Mass Index (BMI) of 21
C. Human Immunodeficiency Virus (HIV) infection A. Primary nursing
D. Low-fiber diet B. Team nursing
E. Excessive alcohol consumption C. Functional nursing
F. African American ethnicity D. Case management
78. Windham is performing medication administration for four 86. RN Cruella is caring for a client with hyperkalemia. Which
clients. Which client and medication should be administered of the following treatments would she recognize as
first? appropriate options for treating this electrolyte imbalance?
1 – prednisone 10mg PO daily for asthma exacerbation Select all that apply.
2 – acetaminophen 500 mg PO x 1 dose for fever
3 – magnesium oxide 250 mg PO daily for chronic alcoholism A. Spironolactone( potassium sparring)ka will not be out
4 – glargine insulin 15 unites SubQ daily for diabetes mellitus B. Sodium polystyrene
* C. Regular insulin
1 point D. Hemodialysis
A. Client one E. Magnesium sulfate
B. Client two
C. Client three
D. Client four
79. Windham cares for the following assigned clients. She
should initially follow-up with the client who:
A. is repeatedly washing their hands.
B. B. talking over others during group therapy.
C. C. yelling and shouting at others.
D. D. is voluntarily admitted and requesting discharge. 87. Cruella is preparing to administer furosemide
intravenously to a client. Prior to administering this
80. Intravenous therapies often consist of electrolyte medication, she should perform which action?
replacement therapies. Select the electrolyte that is
accurately paired with one of its functions. A. Obtain the client's weight
B. Administering furosemide intravenously rapidly to ensure
A. Sodium: The control and management of circulating a quick onset of action
blood volume. C. Verifying the client's blood pressure after administering
B. Bicarbonate: The regulation of extracellular fluid. the medication
C. Chloride: The regulation of plasma protein. D. Assessing the client's serum electrolytes
D. Calcium: The metabolism of fats, carbohydrates, and
proteins. 88. Nurse Cruella is caring for a client with a hiatal hernia who
is being discharged today. Cruella talks to them regarding
81. Instructor Callahan cares for a client who has influenza. methods to manage symptoms and promote overall
Which of the following prescriptions may be prescribed by the well-being associated with their condition. Which of the
doctor? following statements from the client indicate that teaching is
successful?
A. Valacyclovir
B. Oseltamivir A. “I need to wear loose-fitting clothes.”
C. Azithromycin B. “After a meal, I must lie down to avoid dumping
D. Omeprazole syndrome.”
C. “I need to eat three large meals a day.”
82. Callahan is caring for a client with Buerger's disease. He D. “I can go to my favorite Indian restaurant anytime of the
plans on suggesting that the client receive a referral for: week.”
89. While scheduling a client for thoracentesis, Cruella D. Increase the intake of green leafy vegetables
understands which of the following is the most preferred
position for the procedure? 97. Nurse Artie is teaching a health promotion class to a
group of community members on preventing chronic
A. Sitting up, leaning over a bedside table, and feet obstructive pulmonary disease (COPD). Nurse Artie should
supported on the ground or stool. emphasize that a significant risk factor for COPD is:
B. The head of the bed flat with the patient lying on the
unaffected side. A. environmental pollution.
C. Prone position with both arms extended above the head. B. endobronchial infections.
D. The head of the bed elevated 45 degrees, and the patient C. cigarette smoking.
lying on the affected side. D. reactive airway disease.
90. Cruella has attended a staff education program about 98. Nurse Artie has attended a staff education program about
disseminated intravascular coagulation (DIC). Which of the sources of negligent lawsuits. It would indicate effective
following clients is at risk for DIC? Which of the following understanding if Nurse Artie identified which of the following
clients is at risk for DIC? A client: is a source of a negligent lawsuit?
A. with iron deficiency anemia receiving parenteral iron A. The nurse documents care under another nurse's
sucrose infusion. username and password
B. being treated for gram-negative sepsis with intravenous B. The nurse takes pictures of a client's medical record and
antibiotics. distributes them online
C. with atrial fibrillation receiving prescribed rivaroxaban to C. The nurse does not notify the physician of a client's
reduce their risk for stroke. critical laboratory result
D. taking a daily aspirin to reduce their risk for acute D. The nurse treats their spouse in the acute care facility
coronary syndrome. with prescribed medications
91. Baroness is explaining the different aspects of ostomy 99. Religious and cultural rituals/practices often surround
care to a client with a newly created ileostomy. Which death. Which of the following populations prefer cremations
statement from the client indicates an understanding of the rather than burying the remains of the deceased person?
Baroness’ teaching?
A. Hindus
A. “I need to cut the pouch to fit the stoma, allowing B. Islam
one-sixteenth of an inch of room around it.” C. Mormons
B. “I must avoid eating spinach, parsley, and yogurt.” D. Eastern orthodox
C. “I need to drink at least 800 mL of water daily.”
D. “I can eat a large meal during dinner.” 100. Artie has recently finished education about vascular
perfusion. He knows which of the following clients is at
92. Baroness is caring for a client who is admitted to the greatest risk for experiencing impaired vascular perfusion?
hospital with suspected osteomyelitis. Which of the following
laboratory tests should she anticipate being ordered to aid in A. A 76-year-old female client with a history of alcohol
the diagnosis and monitoring of this condition? abuse.
B. A 76-year-old female client with a history of radon gas
A. Erythrocyte sedimentation rate (ESR) exposure.
B. Serum potassium levels C. A 64-year-old male client with a history of cigarette
C. Serum creatinine levels smoking.
D. Prothrombin time (PT) D. A 64-year-old male client with hypotension.
93. Nurse Baroness is caring for a client with chronic
obstructive pulmonary disease (COPD) with a chronically
increased red blood cell count (RBC). Baroness understands
that this finding is likely from:
A. Increased carbon dioxide levels
B. Low blood oxygen levels
C. Insensible water loss
D. Decreased fluid intake